Item: 1 of 27 ~ 1 • M k -<:J 1>- Jil ~· !:';-~ QIO: 3565 ..L a r Previous Next Labfli!l tues Notes Calculat o r

• 1 & & An 8-year-old boy is brought to the hospital because of weakness and abdominal pain. A careful hist01·y reveals that he had a skin 2 • 2 weeks ago. Lately his has been dark brown, and he has not urinated during the past 8 hours. • 3 Laboratory tests show: · 4 Hemoglobin: 13.0 g/dL • 5 Hematocrit: 40% • 6 WBC count: 13,000/mm3 Blood urea nitrogen: 18 mg/dL • 7 : 1.5 mg/dL · 8 Antistreptolysin 0 titer: 184 U . 9

• 10 What is the best therapy for this pat ient ? • 11 : • 12 A . Doxycycline • 13 B. H yd roch loroth iazid e • 14 c . Mannitol • 15 • 16 D . Penicillin V

• 17 E . Supportive therapy • 18 • 19 • 20 • 21 • a s 8 Lock Suspend End Block Item: lof27 ~ . , . M k <:] t> al ~· ~ QIO: 3565 .l. ar Previous Next lab 'lifllues Notes Calculator

1 • The correct answer is E. 590/o chose this. • 2 The history of infection and the high antistreptolysin 0 titer are clear ind icators of a postinfectious g lomerulonephritis. This is believed to be • 3 mediated by immune complex deposition in the g lomerulus. It usually occu rs 2-4 weeks after a skin infection or 1- 2 weeks after an upper respiratory infection (commonly from Streptococcus, serotype 12}. Poststreptococcal g lomerulonephritis (PSGN} g radually resolves within 1 - 2 • 4 weeks, with d isappearing within 6 months. Because this process is mediated by an autoimmune process, antibiotics have little utility, • 5 and most patients experience a complete recovery with supportive therapy alone. Supportive therapy generally involves management of fluid overload and volume status, includ ing loop d iuretics and water and sodium restriction . • 6 Immune complex Hematuria Glomerulus Glomerulus () Anti-streptolysin 0 Autoimmune disease Titer Hypervolemia Antibiotics

• 7 Streptococcus Diuretic loop diuretic Upper respiratory tract infection Autoimmunity Infection Serotype Sodium • 8 A is not correct. J Ofo chose this • • 9 Doxycycline, a tetracycline antibiotic, works by bind ing to the 305 ribosome subunit and prevents attachment of aminoacyl-tRNA. It is effective • 10 against Rickettsia and Chlamydia due to its ability to accumulate intracellularly. Toxicities associated with doxycycline include Gl d istress, teeth d iscoloration, inhibition of bone g rowth in children and photosensitivity. Tetracyclines are contraindicated in pregnancy. This · 11 patient would not benefit from antibiotics because postinfectious g lomerulonephritis is mediated by an autoimmune process. • 12 Doxycycline Tetracycline Ribosome Antibiotics Photosensitivity Tetracycline antibiotics Rickettsia Glomerulonephritis Chlamydia infection Chlamydia (genus) 30S Autoimmunity Aminoacyl-tRNA Pregnancy Autoimmune disease Bone • 13 • 14 B is not correct. 6 0/o chose this • Hydroch lorothiazide is a thiazide d iuretic that inhibits NaCI reabsorption in the early d istal tubule. It is used primarily to control hypertension and • 15 occasionally to alleviate calcium imbalances. Toxicities associated with hydrochlorothiazide can be remembered by the mnemonic hyperGLUC • 16 (hyperGlycemia, hyperlipidemia, hyperUricemia and hyperCalcemia). Postinfectious g lomerulonephritis in children should be managed with supportive care, and most cases will resolve on their own . • 17 Hydrochlorothiazide Thiazide Diuretic Distal convoluted tubule Hypertension Sodium chloride Calcium Tubule Glomerulonephritis • 18 C is not correct. 4 0fo chose this • • 19 Mannitol is an osmotic d iuretic that is used for a variety of applications, includ ing reduction of increased intracranial/intraocu lar pressu re. • 20 Toxicities associated with mannitol include pulmonary edema and dehydration. The use of mannitol is contraindicated in patients that are an uric or have congestive heart failure. This patient's condition will resolve with supportive care, thus mannitol is not ind icated . • 21 • Diuretic Heart failure Mannitol Pulmonary edema Anuria Conqestive heart failure Osmosis Edema Dehydration Osmotic diuretic 6 s 0 lock Suspend End Block Item: lof27 ~ . , . M k <:] t> al ~· ~ QIO: 3565 .l. ar Previous Next lab 'lifllues Notes Calculator g p y y p g y 1 patient would not benefit from antibiotics because postinfectious g lomerulonephritis is mediated by an autoimmune process. . 2 Doxycycline Tetracycline Ribosome Antibiotics Photosensitivity Tetracycline antibiotics Rickettsia Glomerulonephritis Chlamydia infection Chlamydia (genus) 30S • 3 Autoimmunity Aminoacyl-tRNA Pregnancy Autoimmune disease Bone . 4 B is not correct. 60/o chose this. • 5 Hydroch lorothiazide is a thiazide d iuretic that inhibits NaCI reabsorption in the early d istal tubule. It is used primarily to control hypertension and occasionally to alleviate calcium imbalances. Toxicities associated with hydrochlorothiazide can be remembered by the mnemonic hyperGLUC • 6 (hyperGlycemia, hyperlipidemia, hyperUricemia and hyperCalcemia). Postinfectious g lomerulonephritis in children should be managed with . 7 supportive care, and most cases will resolve on their own. Hydrochlorothiazide Thiazide Diuretic Distal convoluted tubule Hypertension Nephron Sodium chloride Calcium Tubule Glomerulonephritis • 8 C is not correct. 4 0fo chose this • • 9 Mannitol is an osmotic d iuretic that is used for a variety of applications, includ ing reduction of increased intracranial/intraocu lar pressu re . • 10 Toxicities associated with mannitol include pulmonary edema and dehydration . The use of mannitol is contraindicated in patients that are an uric · 11 or have congestive heart failure. This patient's condition will resolve with supportive care, thus mannitol is not ind icated. Diuretic Heart failure Mannitol Pulmonary edema Anuria Congestive heart failure Osmosis Edema Dehydration Osmotic diuretic • 12 0 is not correct. 240/o chose this • • 13 Pen icillin V is an oral form of the prototype B-lactam antibiotics. It binds pen icillin -bind ing proteins, blocks transpeptidase cross-linking of • 14 peptidoglycan and activates autolytic enzymes. Pen icillin V is effectively used to treat streptococcal infections, such as pharyng itis, however it has • 15 no proven benefit in reducing poststreptococcal g lomerulonephritis because this condition is an autoimmune process . Phenoxymethylpenicillin Peptidoglycan Penicillin Pharyngitis Penicillin binding proteins Glomerulonephritis Antibiotics Streptococcus Autoimmunity • 16 Autolysis (biology) DD-transpeptidase Enzyme Protein Autoimmune disease • 17

• 18 Bottom Line: • 19 Postinfectious g lomerulonephritis in children should be managed with supportive care. Most cases will resolve on their own . • 20 Glomerulonephritis • 21 6 s 0 lock Suspend End Block Item: 1 of 27 ~ 1 • M k -<:J 1>- Jil ~· !:';-~ QIO: 3565 ..L a r Previous Next Labfli!l tues Notes Calculat o r

1 & & FA17 p 564.1 . 2 • 3 I\ephrltic syndrome= Inflammatory process. When glomeruli are inmlved, leads to hematuria and RBC casts in urine. Associated with a~otemia, oliguria, hypertension (due to salt retention), · 4 . • 5 Acute L~ l -glomerul i enlarged and h) percellular rJ. lost frequently seen in children. Occurs • 6 post streptococca I IF' - (.. sta rry sky") granular appearance - 2-4 weeks after group A streptococcal . 7 glomerulonephritis ("lumpy-bumpy") [lJ due to lgC, lg\ I, and C3 infection of pharynx or skin. Resolves · 8 deposition along CB I and mesangium. spontaneous!). Type Ill h) persensiti' it) . 9 EM-subepithelial immune complex (IC) reaction. humps. Presents with peripheral and periorbital edema, • 10 cola-colored urine, hypertension . • 11 Positi,·e strep titers/serologies, l complement • 12 b ·cls (C3) due to consumption . • 13 Rapidly progressive LM and IF' - crescent moon shape~- Crescents Poor prognosis. Rapidly deteriorating renal • 14 (crescentic) consist of fibrin and plasma proteins (eg, C3b) function (days to weeks). • 15 glomerulonephrit is with glomerular parietal cells, monocytes, • 16 macro phages. Several d isease processes may result in this • 17 pallern, in particular: . 18 • -type II llematuria/hemoptysis. • 19 hypersensiti,·ity reaction; antibodies to Treatment: emergent plasmapheresis. • 20 CBM and alveolar basement membrane • 21 -+ linear IF' • • r-+ _____ • ___ a_ __:_ ••• .:a.L _ _ L ___ _::.L.-: ... 1\ \ t ... _ ... _ ... _, nn., ,,,,....' '- A'Tr"' n ___ _.: =------'-- ,_,,....., a s 8 Lock Suspend End Block Item: 1 of 27 ~ 1 • M k -<:J 1>- Jil ~· !:';-~ QIO: 3565 ..L a r Previous Next Labfli!l tues Notes Calculat o r

1 & & FA17 p566.1 • 2 I\ephrOtic syndrome-massi'e prOteinuria {> 3.5 glday) with hypoalbuminemia, resulting • 3 edema, hyperlipidemia. Frothy urine" ith fatly casts. Due to podocyte damage disrupti ng · 4 glomerular filtration charge barrier. \llay be 1° {eg, direct sclerosis of podocytes) or 2° {systemic • 5 process [eg, diabetes] secondarily damages podocytes). Associated with hypercoagulable state {eg, • 6 thromboembolism) due to antithrombin { T) Ill loss in urine and t risk of infection {due to loss of • 7 immunoglobulins in urine and soft tissue compromise by edema). Se,·ere nephritic syndrome may present with nephrotic S}lldrome features {nephritic-nephrotic · 8 syndrome) if damage to CB I is severe enough to damage charge barrier. . 9 Minimal change LM-normal glomeruli {lipid may be seen in \!l ost common cause of nephrotic S)lldrome • 10 disease (lipoid PCT cells). in children. Often 1° (idiopathic) and ma) be • 11 nephrosis) IF 8 . triggered by recent infection, immunization, • 12 EM-effacement of foot processes t'J . immune stimulus. Rarely, may be zoto 0 • 13 lymphoma (eg, cytokine-mediated damage). I response to corticosteroids . • 14 disease has excellent • 15 Focal segmental LM-segmental sclerosis and hya linosis : . Mo~t common cause of nephrotic sy ndrome in glomerulosclerosis IF - often 8, but may be<±> for nonspeci Fi e focal African Americans and Hispanics. Can be ]0 • 16 deposits of Ig r, C3, Cl. {idiopathic) or zo to other conditions {eg, H IV • 17 EM -effacement of foot process sim ilar to infecti on, sickle cell disease, heroin abuse, • 18 minimal change disease . massive obesity, interferon treatment, ch ronic • 19 kidney disease due to congenital malfom1ations). • 20 1° disease has inconsistent response to steroids . ~lay progress to chronic renal disease . • 21 • T , , •·rr ...... ' . . r • n a s 8 Lock Suspend End Block • 2 • 3 · 4 • 5 • 6 • 7 FA17 p 132.4 · 8 Streptococcus Gram $ cocci in chainsG · . roup i\ strep j ¥1\ ES (major criteria for ac ute rheumatic pyogenes (group A . 9 cause: fe,cr): streptococci) • Pyogenic-pharyngiti s, cellulitis, impetigo Joints-polyarthritis • 10 ("honey-crusted" lesions), er) sipclas • -carditis • 11 • Toxigenic- sca rlet b ·cr, toxic shock- like ~ od u l es (subcutaneous) • 12 syndrome, necroti zing fasci it is Err thema marginatum • 13 • Immunologic-rheumati c fever, Sydenham chorea • 14 glomeruloneph ritis Pharyngitis can result in rh eumatic "phcvcr" Bacitracin sensitive, ~ - hem o l yti c, pyrrolicl onyl and glomerulonephritis. • 15 arylamidase (PYR) $. Hyaluronic acid capsnlc Impetigo usually precedes glomerulonephrit·is. • 16 c inh ibits phagocytosis. An tibodies to M protein Scarlet fever- blanching, sandpaper-like body • 17 enhance host defenses against S (Jyogenes but rash, stra wberry tongue, and circumoral • 18 can gi,·e rise to rheumati c fever. pallor in the setting of group A streptococcal • 19 ASO titer or anti-DNasc B antibodies indicate pharyngitis (eryth roge nic toxin $ ). recent S pyogenes infection . • 20 • 21 I • a s 8 Lock Suspend End Block Item: 2 of 27 ~ 1 • M k -<:J 1>- Jil ~· !:';-~ QIO: 1763 ..L a r Previous Next Labfli!llues Notes Calculat o r

1 & & A 13-year-old boy is brought to the emergency department with periorbital edema, hype•-tension, and tea-colored urine. His parents say that • 2 he had a sore throat about 3 weeks ago. Urinalysis shows RBCs with casts. A positive antistreptolysin 0 titer and decreased levels of • 3 complement are also noted. · 4 What electron microscopy (EM) findings would be expected in this patient's glomeruli? • 5 • 6 A. Dense deposits within the basement membrane • 7 · 8 B. Granular subendothelial deposits . 9 C. Mesangial deposits • 10 D. Podocyte effacement • 11 E. Subepithelial humps • 12 • 13 • 14 • 15 • 16 • 17 • 18 • 19 • 20 • 21 • a s 8 Lock Suspend End Block Item: 2 of 27 ~ 1 • M k -<:J 1>- Jil ~· !:';-~ QIO: 1763 ..L a r Prev ious Next Labfli!llues Notes Calculat or

1 & & Th e correct a nswer is E. 580/o chose this. 2 Acute poststreptococcal glomerulonephritis is an autoimmune disease most frequently seen in • 3 children. Under light microscopy, the glomeruli appear enlarged and hypercellular (resulting from leukocyte infiltration and proliferation of endothelial and mesangial cells). Under electron microscopy, · 4 subepithelial immune complexes are observed in the subepithelium of the glomerulus, as indicated in • 5 this image by the white arrowheads. This condition normally manifests 3-6 weeks after a streptococcal infection (postpharyngitic) with peripheral and periorbital edema, dark urine, and • 6 proteinuria. These symptoms are caused by circulating antistreptococcal antibody-antigen complexes • 7 that deposit in the glomerular basement membrane, leading to complement activation and glomerular damage. The classic findings are RBCs and casts in the urine (which cause the · 8 characteristic tea-colored urine), a positive antistreptolysin 0 titer, and decreased levels of . 9 complement. Basement membrane • 10 Autoimmune disease Proteinuria Glomerular basement membrane Glomerulus {kidney) Glomerulonephritis Glomerulus Mesangium Anti-streptolysin 0 Electron microscope Edema • 11 Urine Autoimmunity Endothelium Immune complex Light microscopy Streptococcus Optical microscope • 12 Mesangial cell • 13 Image copyright ©2011 Scholtze eta!; licensee BioMed Central Ltd • 14

• 15 A i s not correct. 90fo c hose this • • 16 Dense deposits within the basement membrane indicated in this image by the arrows) are characteristic of membranoproliferative glomerulonephritis (MPGN) type II, a rare disease most often • 17 occurring in children aged 5-15 and occasionally associated with a late comorbidity of visual • 18 impairment. It is a result of excessive activation of the alternative complement pathway, resulting in the deposition of multiple complement components in the glomerulus. It is thought to be caused by • 19 C3 nephritic factor, an autoantibody directed against the C3 convertase of the alternative • 20 complement pathway. It can result in nephritic or nephrotic syndrome. As a whole, MPGN I and II account for 4%-7% of cases of nephrotic syndrome in children and adults . • 21 • Memb1 ano1 rol ferative qlomerulonephritis Nephrotic syndrome Alternative complement pathway a s 8 Lock Suspend End Block Item: 2 of 27 ~ 1 • M k -<:J 1>- Jil ~· !:';-~ QIO: 1763 ..L a r Prev ious Next Labfli!llues Notes Calculat or

& & 1 A i s not correct. 90/o ch ose this. 2 Dense deposits within the basement membrane indicated in this image by the arrows) are glomerulonephritis (MPGN) type II, a rare disease most often • 3 characteristic of membranoproliferative occurring in children aged 5-15 and occasionally associated with a late comorbidity of visual · 4 impairment. It is a result of excessive activation of the alternative complement pathway, resulting in • 5 the deposition of multiple complement components in the glomerulus. It is thought to be caused by C3 nephritic factor, an autoantibody directed against the C3 convertase of the alternative • 6 complement pathway. It can result in nephritic or nephrotic syndrome. As a whole, MPGN I and II I . 7 account for 4%-7% of cases of nephrotic syndrome in children and adults. Memb anop o ferative glomerulonephritis Nephrotic syndrome Alternative complemenl pathway · 8 Basement membrane Glomerulonephritis Glomerulus Glomerulus (kidney) Nephritis Nephritic syndrome . 9 Autoantibody Nephron Comorbidity CJ-convertase Visual impairment • 10 Image copyright© 2008 Majoni and • 11 Smith; licensee BioMed Central Ltd • 12 • 13 B is not correct. 180/o c hose this . posits (ind icated by the white stars in this image), for • 14 Granular subendothelial de US MLE Step I purposes, are found in d iffuse proliferative g lomerulonephritis and • 15 membranoproliferative g lomerulonephritis type I. A high-yield d isease associated with • 16 all three is systemic erythematosus {SLE ). Glomerular injury in SLE is primarily mediated by formation of immune complexes composed primarily of anti-double­ • 17 stranded DNA (anti-dsDNA) bound to DNA, which are deposited into the mesangial, . 18 subendothelial, and/or subepithelial spaces. Occasionally, the anti-dsDNA binds directly to the basement membrane. The International Society of Nephrology (ISN) • 19 classification system divides glomerular disorders associated with SLE into six different • 20 patterns (or classes) based upon kidney biopsy histopathology. (1) Minimal mesangial (class I) involves mesangial deposits that are only identified by • 21 • immunofluorescence {IF) and/or electon microscopy (EM); there are no abnormal a s 8 Lock Suspend End Block Item: 2 of 27 ~ 1 • M k -<:J 1>- Jil ~· !:';-~ QIO: 1763 ..L a r Previous Next Labfli!llues Notes Calculat o r

1 & B is not correct. 1 80/o ch ose t his. & Granular subendothelial deposits (indicated by the white stars in this image), for 2 USMLE Step I purposes, are found in diffuse proliferative glomerulonephritis and • 3 membranoproliferative glomerulonephritis type I. A high-yield disease associated with · 4 all three is systemic lupus erythematosus (SLE) . Glomerular injury in SLE is primarily mediated by formation of immune complexes composed primarily of anti-double­ • 5 stranded DNA (anti-dsDNA) bound to DNA, which are deposited into the mesangial, • 6 subendothelial, and/or subepithelial spaces. Occasionally, the anti-dsDNA binds directly to the basement membrane. The International Society of Nephrology (ISN) . 7 classification system divides glomerular disorders associated with SLE into six d ifferent · 8 patterns (or classes) based upon kidney biopsy histopathology. (1) Minimal mesangial lupus nephritis (class I) involves mesangial deposits that are only identified by . 9 immunofluorescence (IF) and/ or electon microscopy (EM ); there are no abnormal • 10 findings on light microscopy (LM ). (2) Mesangial proliferative lupus nephritis (class II) involves mesangial hypercellularity found on LM and few isolated deposits found on IF • 11 or EM. (3) Focal lupus nephritis (class III) is defined by less than 50% of glomeruli are • 12 affected on LM, whereas (4 ) diffuse lupus nephritis (class IV) is defined by more than Image copyright ©2011 Scholtze et al; licensee BioMed 50% are affected. (5) Lupus membranous nephropathy (class V) involves diffuse Central Ltd • 13 thickening of the glomerular capillary wall on LM and subepithelial deposits on IF or • 14 EM. Lastly, (6) advanced sclerosing lupus nephritis (class VI) involves diffuse sclerosing of more than 90% of g lomeruli . lupus erythematosus Membranoproliferative glomerulonephritis Lupus nephritis Immunofluorescence Membranous glomerulonephritis • 15 Systemic Basement membrane Glomerulonephritis Kidney disease Immune complex Nephritis Kidney Biopsy Histopathology Glomerulus Capillary Mesangium • 16 Glomerulus {kidney) Nephrology Lupus erythematosus Endothelium DNA Light microscopy • 17 C is not correct. 80/o ch ose t his • . 18 Mesangial deposits (shown in the these images as fine granular, spherical, or • 19 hemispherical in shape and varying in size) are usually seen in IgA nephropathy, which is the most common cause of primary glomerulonephritis throughout the world . • 20 It manifests with hematuria concurrently with an upper respiratory illness • 21 (synpharyngitic) compared with poststreptococcal glomerulonephritis, which manifests • a s 8 Lock Suspend End Block Item: 2 of 27 ~ 1 • M k -<:J 1>- Jil ~· !:';-~ QIO: 1763 ..L a r Previous Next Labfli!llues Notes Calculat o r

& 1 C is not correct . s oto ch ose this . & 2 Mesangial deposits (shown in the these images as fine granular, spherical, or hemispherical in shape and varying in size) are usually seen in IgA nephropathy, • 3 which is the most common cause of primary glomerulonephritis throughout the world. · 4 It manifests with hematuria concurrently with an upper respiratory illness • 5 (synpharyngitic) compared with poststreptococcal glomerulonephritis, which manifests 3-6 weeks after group A 13-hemolytic streptococcal pharyngitis. Mesangial deposits are • 6 found via electron microscopy and immunofluorescence, whereas light microscopy . 7 primarily displays mesangial hypercellularity. Streptoco :ca pharyngitis Hematuria IgA nephropathy Immunofluorescence Pharyngitis · 8 Glomerulonephritis Electron microscope Immunoglobulin A Streptococcus Kidney disease . 9 Ught microscopy Microscopy Mesangium Optical microscope • 10

• 11 • 12 Images copyright© 2013 Yahata eta/.; licensee BioMed • 13 Central Ltd. • 14 D is not co rrect . J Ofo c hose this . • 15 Podocyte effacement (shown in this image) is seen in minimal change disease (MCD), which • 16 manifests with symptoms of nephrotic synd rome, including massive proteinuria {>3.5 g/day), • 17 edema, hypoalbuminemia, hypertriglyceridemia, and frothy urine. It is thought that overproduction of interleukin- 13, secondary to aberrant T-cell signaling, affects the visceral epithelial cells in the . 18 glomerular capillary wall, resulting in marked proteinuria and foot process fusion. MCD occurs more • 19 frequently in hematologic malignancies such as Hodgkin lymphoma and in some patients precedes the discovery of the lymphoma by several months or years . • 20 Nephrotic syndrome Hypoalbuminemia Minimal change disease Interleuk1n 13 Proteinuna Hypertriglyceridemia • 21 Podocyte Edema T cell Epithelium Hodgkin's lymphoma Urine Lymphoma Glomerulus Glomerulus (kidney) • a s 8 Lock Suspend End Block • 3 Images copyright© 2013 Yahata eta/.; licensee BioMed Central Ltd. · 4

• 5 0 is n o t correct . J Ofo ch ose this. • 6 Podocyte effacement (shown in this image) is seen in minimal change disease (MCD), which manifests with symptoms of nephrotic syndrome, including massive proteinuria (>3.5 g/day), • 7 edema, hypoalbuminemia, hypertriglyceridemia, and frothy urine. It is thought that overproduction · 8 of interleukin-13, secondary to aberrant T-cell signaling, affects the visceral epithelial cells in the proteinuria and foot process fusion. MCD occurs more . 9 glomerular capillary wall, resulting in marked frequently in hematologic malignancies such as Hodgkin lymphoma and in some patients precedes • 10 the discovery of the lymphoma by several mont hs or years. Hypoalbuminemia Minimal change disease Interleukin 13 Proteinuria Hypertriglyceridemia • 11 Nephrotic syndrome Podocyte Edema T cell Epithelium Hodgkin's lymphoma Urine Lymphoma Glomerulus Glomerulus (kidney) • 12 Malignancy Capillary Cancer Tumors of the hematopoietic and lymphoid tissues • 13 • 14 • 15 • 16 • 17 Bottom Line : • 18 A potential sequela of streptococcal infection, most commonly occurring in children, is the development of poststreptococcal glomerulonephritis (PSGN). a condition marked by the subepithelial deposition of antigen-antibody complexes and a "lump-bumpy" appearance on electron • 19 microsopy (EM). IgA nephropathy is "synpharyngitic" whereas PSGN is, as the name implies, "postpharyngitic." An important distinction to • 20 make is the time from pharyngitis to hematuria when differentiating between these two entities . Sequela Hematuria IgA nephropathy Pharyngitis Glomerulonephr;tis Immunoglobul n A Streptococcus Immune complex Kidney disease Infection • 21 • a s 8 Lock Suspend End Block Item: 2 of 27 ~ 1 • M k -<:J 1>- Jil ~· !:';-~ QIO: 1763 ..L a r Previous Next Labfli!llues Notes Calculat o r

1 & & FA17 p 564.1 2 Nephritic syndrome l\ephrltic syndrome= Inflammatory process. When glomeruli are inmlved, leads to hematuria • 3 and RBC casts in urine. Associated with a~otemia, oliguria, hypertension (due to salt retention), · 4 proteinuria. • 5 Acute u.~ -glomeruli enlarged and h) percellular fJ. lost frequently seen in children. Occurs • 6 poststre ptococca I IF- ("starry sky") granular appearance - 2-4 weeks after group A streptococcal . 7 glo me rulo ne phritis ("lumpy-bumpy") (lJ due to lgC, lg\1, and C3 infection of pharynx or skin. Resolves · 8 deposition along GB I and mesangium. spontaneousl). Type Ill h) persensiti' it) E~1-subepithelial immune complex (IC) reaction. . 9 humps. Presents with peripheral and periorbital edema, • 10 cola-colored urine, hypertension. • 11 Positi,·e strep titers/serologies, l complement • 12 b ·cls (C3) due to consumption . • 13 Rapidly progressive LM and IF- crescent moon shape ~- Crescents Poor prognosis. Rapidly deteriorating renal • 14 (crescentic) consist of fibrin and plasma proteins (eg, C3b) function (days to weeks). glomerulo ne phritis with glomerular parietal cel ls, monocytes, • 15 macro phages . • 16 Several d isease processes may result in this • 17 pallern, in particular: . 18 • Goodpasture syndrome-type II llematuria/hemoptysis. • 19 hrpersensitivity reaction; antibodies to Treatment: emergent plasmapheresis. • 20 CBl\1 and alveolar basement membrane -+ linear IF • 21 • r.r:~nnlom::~tmi~ with nolv::~nuiiti~ (\.VPoPnPr) PR~-A I (;Afr-Ao (;A P::~1wi-immnnP (no l olr.~ a s 8 Lock Suspend End Block Item: 2 of 27 ~ 1 • M k -<:J 1>- Jil ~· !:';-~ QIO: 1763 ..L a r Previous Next Labfli!llues Notes Calculat o r

1 & & FA17 p 566.1 2 Nephrotic syndrome t\ephrO ti c syndrome-massive prOteinuri a (> 3.5 glday) with hypoalbuminemia, resulting • 3 edema, hyperlipidemia. Frothy urine " ith fa tty casts. Due to podocyte damage disrupting · 4 glomerular filtration charge barrier. \llay be 1° (eg, direct sclerosis of podocytes) or 2° (systemic • 5 process [eg, diabetes] secondarily damages podocytes). Associated with hypercoagulable state (eg, • 6 thromboembolism) due to antithrombin ( T) Ill loss in urine and t risk of infection (due to loss of . 7 immunoglobulins in urine and soft tissue compromise by edema). Se,·ere nephritic syndrome may present with nephroti c S}lldrome features (nephritic-nephrotic · 8 syndrome) if damage to CBl\ I is severe enough to damage charge barrier. . 9 Minimal change LM- normal glomeruli (lipid may be seen in \llost common cause of nephrotic S)lldrome • 10 disease (lipoid PCT cells). in children. Often 1° (idiopathic) and ma) be • 11 nephrosis) we. triggered by recent infec tion, immunization, • 12 EM -effacement of foot processes t'J . immune stimulus. Rarely, may be zot o • 13 lymphoma (eg, cytokine-mediated damage). 1° disease has excellent response to corticostero ids. • 14 Focal segmental -segmental sclerosis and hya li nosis : . se of nephrotic sy ndrome in • 15 LM Most common cau glomerulosclerosis IF - often 8 , but may be<±> for nonspeci fi e focal African Americans and Hispanics. Can be ]0 • 16 deposits of Ig r, C3, Cl. (idiopathic) or zo to other conditions (eg, H IV • 17 EM -effacement of foot process similar to infecti on, sickle cell disease, heroin abuse, . 18 minimal change disease . massive obesity, interferon treatment, chronic I • 19 kidn ey disease due to congenital malformations). • 20 1° disease has inconsistent response to steroids . ~lay progress to chronic renal disease . • 21 t ______. r1 o _ 1 -- · · • •• --L ----··- T'.r -•·rr. ___ __ ·•• -- _ l r-n,r.l · ~ \,f I - a s 8 Lock Suspend End Block Item: 3 of 27 ~ 1 • M k -<:J 1>- Jil ~· !:';-~ QIO: 1782 ..L a r Previous Next Labfli!llues Notes Calculat o r

1 & & A 30-year-old female is brought to your em ergency department by EMS after neighbors called when they saw the patient collapse at her 2 mailbox. Blood pressure is 104/54 mm Hg, pulse 108/min, and oxygen saturation is 92%. On physical exam she awakens to painful stimuli, • 3 bowel sounds are present with moderate, nonlocalized abdominal pain. The remainder of the exam is unrevealing. Initial laboratory analysis reveals the following: · 4 Na+, 140 mEq/L • 5 K+, 3.6 mEq/L • 6 c1-, 116 mEq/L HC03-, 15 mEq/L • 7 Arterial blood gas pH, 7.32 · 8 Pco2, 30 mm Hg . 9 • 10 Which of the following is most likely to explain t hese laboratory abnormalities? • 11 : • 12 A . Bulimia • 13 B. Diabetic ketoacidosis • 14 • 15 C. Diarrheal illness

• 16 D. Furosemide use • 17 E. Heroin overdose • 18 • 19 • 20 • 21 • a s 8 Lock Suspend End Block Item:3of27 ~ . , . M k <:] t> al ~· ~ QIO: 1782 .l. ar Previous Next lab 'lifllues Notes Calculator

1 • The correct answer is C. 550fo chose this. 2 Diarrhea (including traveler's d iarrhea) causes a non-anion gap metabolic acidosis. Bicarbonate lost in the gastrointestinal tract is exchanged for 3 c l-, lead ing to a hyperchloremic non-anion gap acidosis. Because Cl- is counted in normal electrolyte measurements, there is not a significant anion gap. Lab data would reveal a low pH with a low Pco . • 4 2 Traveler's diarrhea Metabolic acidosis Electrolyte Diarrhea PH Anion gap Human gastrointestinal tract Gastrointestinal tract Anion Chloride Bicarbonate Acidosis

• 5 Metabolism Chlorine • 6 A is not correct. 9 0fo chose this. • 7 The loss of hydrochloric acid due to vomiting in patients with bulimia leads to a transient increase in pH. The kidney responds by excreting • 8 sodium and/or potassium bicarbonate. As the vomiting persists, the alkalemic and dehydrated patient tries to conserve sodium through activation of the ren in -angiotensin-aldosterone system. Therefore, although HC03- excretion persists (because of the alkalosis), it is primarily • 9 excreted with potassium rather than sodium. Compensating by conserving sodium and excreting HC03- at the expense of potassium soon fails. • 10 In the presence of severe hypokalemia, the kidney beg ins to exchange H+ in the d istal convoluted tubule in order to conserve potassium . Therefore, acidic urine in the setting of volume contraction and alkalosis is primarily due to the loss of potassium. · 11 Distal convoluted tubule Potassium bicarbonate Hypokalemia Hydrochloric acid Bulimia nervosa Vomiting Bicarbonate Kidney Urine Potassium Sodium Alkalosis

• 12 Renin-angiotensin system Dehydration Nephron Acid

• 13 B is not correct. 220/o chose this. • 14 Diabetic ketoacidosis leads to an anion gap metabolic acidosis. This patient does have a metabolic acidosis, but her anion gap is 9 (Na+ - [c l- + HCo - )), which is a normal anion gap. Thus this is not the best answer. • 15 3 Diabetic ketoacidosis Metabolic acidosis Anion gap Ketoacidosis Acidosis Diabetes mellitus Anion Metabolism • 16 0 is not correct. S O/o chose this . • 17 Loop d iuretics such as furosemide that are used to treat heart failure can lead to metabolic alkalosis. The development of the metabolic alkalosis • 18 has three components: (1) volume depletion, (2} electrolyte imbalance (specifically lead ing to hypochloremia and hypokalemia), and (3} secondary hyperaldosteronism. Volume contraction leads to increased Na+ reabsorption and an increased HC03- threshold. Diuretic- induced • 19 hypochloremia and hypokalemia lead to the persistence of the alkalosis. A d rop in intravascular volume causes activation of the ren in ­ • 20 angiotensin-aldosterone system and an increase in aldosterone levels. With persistent hypokalemia, H+ is preferentially excreted instead of K+ at the d istal convoluted tubule in exchange for Na+. • 21 • Oic:t~l ronvolutP.c1 tuhuiP. Furoc:P.mic1P. Hvnorhlon~mi~ MP.t~holir ::llk::lloc:ic: Hvnok::liP.mi~ Alc1oc:tP.ronP. HvnP.r~lc1oc:tP.ronic:m FIP.rtrolvtP. I non c1iurP.tir OiurP.tir 6 s 0 lock Suspend End Block Item:3of27 ~ . , . M k <:] t> al ~· ~ QIO: 1782 .l. ar Previous Next lab 'lifllues Notes Calculator

. . ~ ...... ~- . . . . ~ ... 1 Therefore, acidic urine in the setting of volume contraction and alkalosis is primarily due to the loss of potassiu m . 2 Distal convoluted tubule Potassium bicarbonate Hypokalemia Hydrochloric acid Bulimia nervosa Vomiting Bicarbonate Kidney Urine Potassium Sodium Alkalosis Renin-angiotensin system Dehydration Nephron Acid 3 • 4 B is not correct. 220/o c hose t his. Diabetic ketoacidosis leads to an anion gap metabolic acidosis. This patient does have a metabolic acidosis, but her anion gap is 9 (Na+ - [c l- + • 5 HC03-)), which is a normal anion gap. Thus this is not the best answer. • 6 Diabetic ketoacidosis Metabolic acidosis Anion gap Ketoacidosis Acidosis Diabetes mellitus Anion Metabolism

• 7 0 is not correct. SO/o c hose t his. • 8 Loop d iuretics such as furosemide that are used to treat heart failure can lead to metabolic alkalosis. The development of the metabolic alkalosis has three components: (1) volume depletion, (2} electrolyte imbalance (specifically lead ing to hypochloremia and hypokalemia), and (3} • 9 secondary hyperaldosteron ism. Volume contraction leads to increased Na+ reabsorption and an increased HCo3- threshold. Diuretic-induced • 10 hypochloremia and hypokalemia lead to the persistence of the alkalosis. A d rop in intravascular volume causes activation of the ren in ­ angiotensin-aldosterone system and an increase in aldosterone levels. With persistent hypokalemia, H+ is preferentially excreted instead of K+ at · 11 the d istal convoluted tubule in exchange for Na+. • 12 Distal convoluted tubule Furosemide Hypochloremia Metabolic alkalosis Hypokalemia Aldosterone Hyperaldosteronism Electrolyte loop diuretic Diuretic • 13 Hypovolemia Alkalosis Renin-angiotensin system Water-electrolyte imbalance Heart failure Nephron Blood plasma Metabolism • 14 E is not correct. 60/o c hose t his . • 15 Heroin overdose would lead to a respiratory acidosis due to narcotic- induced hypoventilation. Alveolar hypoventilation leads to hypercapnia. This increase in turn decreases the ratio of HCo3- relative to the Pco2 and decreased pH. Hypoventilation will not lead to a metabolic acidosis; • 16 compensation, instead, resu lts in elevated plasma HC03- levels. • 17 Hypercapnia Respiratory acidosis Metabolic acidosis Heroin Acidosis Hypoventilation Blood plasma Pulmonary alveolus Opioid overdose Metabolism Drug overdose • 18 • 19 Bottom Li ne : • 20 Diarrhea leads to excessive HC03- excretion, resu lting in a non-anion gap metabolic acidosis. Metabolic acidosis Diarrhea Acidosis Excretion Metabolism • 21 • 6 s 0 lock Suspend End Block Item: 3 of 27 ~ 1 • M k -<:J 1>- Jil ~· !:';-~ QIO: 1782 ..L a r Previous Next Labfli!llues Notes Calculat o r

& & 1 FA17p561.1 2 Acid-base physiology 3 pH p~ IHC01 I COMPENSATORY RESPONSE · 4 Metabolic acidosis l l l l lyper.·entilation (immediate) • 5 Metabolic alkalosis t t t llypo"entilation (immediate) • 6 Respiratory acidosis l t t t renal (1-IC03- J reabsorption (delayed) . 7 Respiratory alkalosis t l l l renal [HC03- J reabsorption (delayed) · 8 Kc): t l = 1" disturbance; l t =compensatory response. . 9 • 10 . [I ICO ·] Henderson-Hasselbalch equahon: pi I = 6. 1 +log _ P~Oz • 11 0 03

• 12 Predicted respira tory compensation for a simple metabolic acidosis can be calculated using the • 13 Winters formula. If measured Pco2 >predicted Pco2 - concomitant respiratory acidosis; if • 14 measured Pco2 < predicted Pco2 - concomitant respimtory alkalosis: • 15 Pco2 = 1.5 IH C03- 1+ 8 :!: 2 • 16

• 17 FA17 p 561 .2 . 18 Acidosis and alkalosis • 19 • 20 Check arterial pH • 21 pH< 7.35 pH> 745 • a s 8 Lock Suspend End Block Item: 4 of 27 ~ 1 • M k -<:J 1>- Jil ~· !:';-~ QIO: 1798 ..L a r Previous Next Labfli!llues Notes Calculat o r

1 & & A middle-aged, white man crashes his car into a tree while driving drunk. At the scene, he was exsanguinating and hypotensive. The 2 emergency department team pe1forms acute resuscitation requiring 5 L saline and 6 U packed RBCs and subsequently admits him to the his urine output is inadequate. Blood urea nitrogen:creatinine ratio is 13:1. Urinalysis 3 trauma intensive care unit. On hospital day 2, demonstrates numerous casts, as shown in the image. . 4 • 5 • 6 . 7 · 8 . 9 • 10

• 11 • 12 . 13 • 14 .. •• • 15 Image courtesy of Dr. Serban Nicolescu, A Imina Laboratory • 16 The condition affecting this patient's kid neys can be caused by a variety of mechanisms. Which of the following is a common cause of his kidney • 17 pathology? . 18 : • 19 A. Amyloidosis • 20 B. Ascending urinary tract infection • 21 • a s 8 Lock Suspend End Block 2 3 . 4 • 5 • 6 . 7 · 8 . 9 • 10 .. • • Image courtesy of Dr. Serban Nicolescu, A Imina Laboratory • 11 • 12 The condition affecting this patient's kid neys can be caused by a variety of mechanisms. Which of the following is a common cause of his kidney • 13 pathology? • 14 : • 15 A. Amyloidosis

• 16 B. Ascending urinary tract infection • 17 C. Crush injury . 18 • 19 D. Diabetes mellitus • 20 E. NSAID toxicity • 21 • a s 8 Lock Suspend End Block Item: 4 of 27 ~ 1 • M k -<:J 1>- Jil ~· !:';-~ QIO: 1798 ..L a r Prev ious Next Labfli!llues Notes Calculat or

1 & & 2 Th e correct a n s wer is C. 6 30/o chose this. This man was found exsanguinating and hypotensive. He quickly passed from prerenal to intrinsic kidney injury. As urinalysis indicates, he is 3 suffering from (ATN) secondary to renal ischemia. Muddy brown casts (seen in the image) on urinalysis are 4 pathognomonic for ATN. (caused by crush injury in this case) is another common cause of ATN. It is a direct result of muscle cell renal tubules. Thus, it does not typically pass through the prerenal stage. Other toxins, such as • 5 necrosis and deposition of in the chemotherapeutic agents and aminoglycoside antibiotics, may also cause this condition . • 6 Rhabdomyo ~sos Aminog ycoside Myoglobin Acute tubular necrosis Urinalysis Pathogo omo 1c Ischemoa Necrosis Hypotension Kidney Antibiotics Myocyte • 7 Cr sh syn · ome To n · 8 A is not correct. 5 0/o chose this. . 9 Amyloidosis results in subendothelial deposits in the kidney, leading to nephrotic syndrome. The kidney is t he most commonly involved organ in systemic amyloidosis and without t reat ment may progress to end-stage renal disease. Amyloidosis is associated with chronic conditions such as • 10 multiple myeloma, tuberculosis, and rheumatoid arthritis. Testing for amyloidosis is done with the Congo red stain, demonstrating apple-green • 11 birefringence. Urinalysis in patients with end-stage renal disease would show waxy casts . Congo red Nephrotic syndrome Multiple myeloma Rheumatoid arthritis Tuberculosis Amyloidosis Urinalysis Birefringence Endothelium Kidney • 12 End-stage renal disease Arthritis Kidney disease • 13 B is not correct. 60/o c hose this . • 14 is often the result of an ascend ing urinary tract infection. This infection generally affects the renal cortex, with relative sparing of • 15 the vessels and glomeruli. Urinalysis in these patients often will reveal WBC casts . Urinary tract infection Pyelonephritis Renal cortex Urinalysis Glomerulus Urinary system Glomerulus (kidney) Kidney • 16 • 17 D is not correct. 60fo chose this . Diabetes causes chronic kidney damage. The first presentation is microscopic proteinuria, which gradually moves to gross proteinuria, possibly • 18 hematuria, and . On biopsy, diabetes can manifest as renal papillary necrosis with the presence of Kimmelstiei-Wilson nodules . • 19 Renal papillary necrosis Hematuria Proteinuria Glycosuria Diabetes mellitus Kidney Biopsy Necrosis Nephrotoxicity Kidney disease

• 20 E is n o t correct . 200/o ch ose this. • 21 Nonsteroidal anti-inflammatory drugs {NSAIDs) can induce several forms of kidney injury, including acute interstitial nephritis (AIN), acute • ' . ' , ... , 1' .. . ' ' • I .o. -r•o' a s 8 Lock Suspend End Block Item:4of27 ~ . , . M k <:] t> al ~· ~ QIO: 1798 .l. ar Previous Next lab 'lifllues Notes Calculator g I y p JJ; 1 Urinary tract infection Pyelonephritis Renal cortex Urinalysis Glomerulus Urinary system Glomerulus (kidney) Kidney 2 0 is not correct. 60/o c hose t his. 3 Diabetes causes chronic kidney damage. The first presentation is microscopic proteinuria, which g radually moves to g ross proteinuria, possibly 4 hematuria, and g lycosu ria. On biopsy, d iabetes can manifest as renal papillary necrosis with the presence of Kimmelstiei-Wilson nodules. Renal papillary necrosis Hematuria Proteinuria Glycosuria Diabetes mellitus Kidney Biopsy Necrosis Nephrotoxicity Kidney disease • 5 • 6 E is not correct. 200/o c hose t his • Nonsteroidal anti-inflammatory d rugs {NSA!Ds} can induce several forms of kidney inj ury, includ ing acute interstitial nephritis (AIN}, acute • 7 kidney injury (AKI), and acute tubular necrosis (ATN} . • 8 AIN is an inflammatory d isorder of the kidney often caused by d rug therapy, especially NSA!Ds and 13 -lactam d rugs (penicillins and • 9 cephalosporins). AIN is characterized clinically by a decrease in renal function evidenced by an increase in serum creatinine. The typical triad of • 10 symptoms for AIN includes fever, rash, and eosinophila. Not all of these symptoms needs to be present, however. Urinalysis often will show WBC casts and (key to the d iagnosis of AIN}. NSAID-induced AIN is unique in that it may also manifest with nephrotic syndrome, in which · 11 the patient presents with proteinuria, hypoalbuminemia, and edema, in addition to the typical features of AIN .

• 12 ATN is a possibility with NSA!Ds. NSA!Ds inhibit the production of prostagland ins lead ing to constriction of the afferent arterioles. Excessive • 13 NSAID use can lead to ATN via the mechanism of prerenal failure. Although a possibility, the stem of this question does not support the use of NSA!Ds. The mechanism causing the ATN is more likley due to crush injury () . • 14 Nonsteroidal anti-inflammatory drug Nephrotic syndrome Hypoalbuminemia Interstitial nephritis Myoglobinuria Acute tubular necrosis Proteinuria Urinalysis • 15 Acute kidney injury Creatinine Nephritis Prostaglandin Edema Renal function Kidney Necrosis Anti -inflammatory Afferent arterioles Blood plasma • 16 Penicillin Cephalosporin Rash Serum (blood) Fever Serum creatinine Interstitial fluid Crush syndrome • 17 • 18 Bottom Line : • 19 The most common cause of acute tubular necrosis is prerenal kidney injury. Rhabdomyolysis is the second most common. Muddy brown casts • 20 are characteristic. Rhabdomyolysis Acute tubular necrosis Kidney Necrosis • 21 • 6 s 0 lock Suspend End Block Item: 4 of 27 ~ 1 • M k -<:J 1>- Jil ~· !:';-~ QIO: 1798 ..L a r Previous Next Labfli!llues Notes Calculat o r

1 & & FA17 p 572.2 2 Acute tubular necrosis !\ lost common cause of ac ute J..idn ey injur) in hospitalized pati ents. Spontaneously resolves in 3 many cases. Can be fatal, especially during initial oliguric phase. f FENa. 4 Key findin g: granular ("muddy brown") casts • 5 3 stages: • 6 l. Inciting event . 7 2 . .\l[aintenancc phasc- oliguric; lasts 1-3 weeks; risk of hyperkalemia, metabolic acidosis, uremta · 8 3. Recovery phase- polyuric; BU and serum creatinine fall : risk of hypokaJemia . 9 Can be caused by ischemic or nephrotoxic injury: • 10 Ischemic-2° to l renal blood Oo\\ (eg, h} potension, shock, sepsis, hemorrhage, IlF). Resuhs • 11 in death of tubular cells that may slough into tu bular lumen : (PCT and thick ascending limb ghly susceptible to injury). • 12 are hi ephrotoxic-2° to injury rcsuhing from toxic substances (cg, aminoglycosides, radiocontrast • 13 agents, lead, cisplatin, eth rlene glycol), crush injury (myoglobinuria}, . PCT is • 14 particularly susceptible to injury. • 15 • 16 • 17 FA17 p 571 .2 . 18 Acute kidney injury Acute kidney injury is defin ed as an abrupt decline in renal functi on as measured by f creatinine • 19 (acute renal failure) and f BU N or by oliguria/anuria. • 20 Prerenal azotemia Due to l RBF (eg, hypotension) - l CFR. a+/1120 and BUt retained by kidn ey in an att empt to • 21 • consen e volume - f BU '/creatinine ratio (BU l is reabsorbed, creatinine is not) and l FE" •. a s 8 Lock Suspend End Block Item: 4 of 27 ~ 1 • M k -<:J 1>- Jil ~· !:';-~ QIO: 1798 ..L a r Previous Next Labfli!llues Notes Calculat o r

1 & &

2 FA17 p 571 .2

3 Acute kidney injury Acute kidney injury is defined :~san abrupt decline in ren:~ l function as measured by t creatinine 4 (acute renal failure) and t BUN or by oliguria/anuria. • 5 Prerenal azotemia Due to l RBF (eg, hypotemion) - l CFR. 'a+f1 J10 and BUN retained by kidn ey in an attempt to • 6 consene \Oiume - t BUt /creatinine ratio (BUN is reabsorbed, creatinine is not) and l FE\la· • 7 Intrinsic renal failure Cenerallr due to acute tubular necrosis or ischemia/toxins; less commonlr due to acute · 8 glomerulonephritis (eg, RPC1 , hemol) tic uremic S) ndrome) or acute interstitial nephritis. . 9 In ATt , patchy necrosis - debris obstructing tubule and fluid backAo"· across necrotic tubule - l CFR. Urine has epithelial/granular casts. BUN reabsorption is impaired - l BUi\/creatinine • 10 ratio and t FENa· • 11 Postrenal azotemia Due to outflow obstruction (stones, BPI I, neoplasia, congenital anomalies). De,·elops only with • 12 bilateral obstruction . • 13 Pre renal Intrinsic renal Postrenal • 14 • 15 > 500 < 350 < 350 (mOsm/kg) • 16 Urine Na+ (mEq/l) < 20 >40 >40 • 17 2% < 1% (mild) • 18 FENa > 2% (severe) • 19 Serum BUN/Cr > 20 < 15 Varies • 20 • 21 • a s 8 Lock Suspend End Block Item: 5 of 27 ~ 1 • M k -<:J 1>- Jil ~· !:';-~ QIO: 1800 ..L a r Previous Next Labfli!l tues Notes Calculat o r

1 & & A 59-year-old woman with type 2 diabetes mellitus comes to her primary care physician for a routine visit. Her creatinine level has been 2 slowly increasing over the last decade due to poor adherence to her medical regimen. 3 4 If her renal disease were to progress, she would be at risk for developing which of the following conditions?

• 5 : • 6 A. Hypokalemia

• 7 B. Hypophosphatemia · 8 C. Hypotension . 9 D. Metabolic alkalosis • 10 • 11 E. Osteomalacia

• 12 F. Polycythemia • 13 • 14 • 15 • 16 • 17 • 18 • 19 • 20 • 21 • a s 8 Lock Suspend End Block Item:5of27 ~ . , . M k <:] t> al ~· ~ QIO: 1800 .l. ar Previous Next lab 'lifllues Notes Calculator

1 • The correct answer is E. 520/o chose this. 2 Osteomalacia occu rs in renal failure due to the kidney's inability to maintain its normal vitamin D production. Ch ron ic renal failure is a common complication in nonadherent d iabetic patients. Ch ron ic renal failure resu lts in the progressive loss of renal function, eventually lead ing to end­ 3 stage renal d isease. In chronic renal failure, the kidneys are unable to keep up their normal excretory, metabolic, and endocrine functions. The 4 abnormalities include accumulation of toxins, underproduction of hormones (vitamin D and erythropoietin), and increased release of ren in . Symptoms and clinical abnormalities associated with chronic renal failure include edema, hyperkalemia, metabolic acidosis, hyperphosphatemia, 5 hypocalcemia, renal osteodystrophy, hypertension, pulmonary edema, congestive heart failure, uremia, anemia, nausea, vomiting, peripheral • 6 neuropathy, pruritus, and decreased libido. The pathogenesis of osteomalacia in patients with chronic renal d isease is caused by failure of the kidney to turn 25-{0H}D into the active form 1,25-{0H}2D. Without active vitamin D, there is impaired mineralization of bone, lead ing to renal • 7 osteodystrophy . • 8 Hyperkalemia Renal osteodystrophy Hypocalcaemia Erythropoietin Uremia Osteomalacia Hyperphosphatemia Metabolic acidosis Vitamin D Pulmonary edema • 9 Peripheral neuropathy Itch Edema Renin Anemia Heart failure Congestive heart failure Kidney disease Hypertension libido Nausea Kidney Acidosis Renal function Vomiting Endocrine system Ossification Diabetes mellitus Pulmonary hypertension Vitamin Excretion Metabolism Bone Pathogenesis • 10 Hormone · 11 A is not correct. 140/o chose this • • 12 In end-stage renal d isease, the kidneys are unable to regulate and excrete potassium, lead ing to hyperkalemia, not hypokalemia. Card iac • 13 arrhythmias are a common complication of hyperkalemia. • 14 Hyperkalemia Hypokalemia Kidney disease End-stage renal disease Kidney Cardiac arrhythmia Potassium

• 15 B is not correct. 100/o chose this •

• 16 Hyperphosphatemia, not hypophosphatemia, is a common complication of chronic kidney d isease. Retention of phosphate can lead to secondary hyperparathyroidism. Secondary hyperparathyroidism contributes to the development of bone d isease, such as osteitis fibrosa cystica, in chronic • 17 renal d isease patients. • 18 Osteitis fibrosa cystica Chronic kidney disease Kidney disease Hyperparathyroidism Secondary hyperparathyroidism Kidney Phosphate Bone Bone disease • 19 C is not correct. JOfo chose this • Hypotension is not caused by renal failure. Excess retention of sodium and water leads to fluid overload and resu lts in hypertension, congestive • 20 heart failure, and pulmonary edema . • 21 Heart failure Pulmonary edema Hypotension Congestive heart failure Hypertension Edema Hypervolemia Sodium Kidney • 6 s 0 lock Suspend End Block Item: 5 of 27 ~ 1 • M k -<:J 1>- Jil ~· !:';-~ QIO: 1800 ..L a r Prev ious Next Lab fli!ltues Not es Calculat or . . . . 1 & & In end-stage renal disease, the kidneys are unable to regulate and excrete potassium, leading to hyperkalemia, not hypokalemia. Cardiac 2 arrhythmias are a common complication of hyperkalemia. Hyperkalemia Hypokalemia Kidney disease End-stage renal disease Kidney Cardiac arrhythmia Potassium 3 4 B i s n o t correct. 100/o chose this. Hyper phosphatemia, not hypophosphatemia, is a common complication of chronic kidney disease. Retention of phosphate can lead to secondary 5 hyperparathyroidism. Secondary hyperparathyroidism contributes to the development of bone disease, such as osteitis fibrosa cystica, in chronic • 6 renal disease patients. hyperparathyroidism Kidney Phosphate Bone Bone disease • 7 Osteitis f osa cystica Chronic kodney disease Kidney disease Hyperparathyroidism Secondary · 8 C is not correct. JOfo chose this. Hypotension is not caused by renal failure. Excess retention of sodium and water leads to fluid overload and results in hypertension, congestive . 9 heart failure, and pulmonary edema . • 10 Heart failure Pulmonary edema Hypotension Congestive heart failure Hypertension Edema Hypervolemia Sodium Kidney

• 11 D is not correct. 11Ofo chose this. • 12 Metabolic acidosis, not alkalosis, occurs due to a decrease in acid secretion and a decrease in bicarbonate production . Metabolic acidosis Acidosis Alkalosis Bicarbonate Metabolism Secretion • 13 F is not correct. 60/o c hose t his . • 14 Anemia, not polycythemia, will result in renal failure as the kidneys become unable to produce erythropoietin. Some renal cancers can lead to • 15 overexpression of erythropoietin and lead to polycythemia, but renal failure leads to anemia. • 16 Erythropoietin Polycythemia Anemia Kidney • 17

• 18 Bottom Line:

• 19 Osteomalacia occurs due to the kidney's inability to maintain its normal vitamin D production in renal failure, a common complication in • 20 nonadherent diabetic patients . Osteomalacoa v;tamon D Kidney Diabetes mellitus Vitamin • 21 • a s 8 Lock Suspend End Block Item: 5 of 27 ~ 1 • M k -<:J 1>- Jil ~· !:';-~ QIO: 1800 ..L a r Previous Next Labfli!l tues Notes Calculat o r

1 & & FA17 p 571 .3 2 Consequences of renal Inability to make uri ne and excrete nitrogenous 2 forms of renal fai lure: ac ute (eg, AT I) and 3 failure wastes. chronic (eg, hypertension, diabetes mellitus, 4 Consequences ( ~1.\0 HU:\GER}: congenital anomalies). 5 \1etabolic Acidosis • 6 Dyslipidemia (cspeciall} t trigl)ccridcs) . 7 Hyperkalemia Uremia-clinical syndrome marked b) · 8 t BUt : . 9 'ausea and anorexia • 10 • Pericarditis • 11 • Asterixi s • 12 Encephalopathy Platelet dysfunction • 13 :\fa+/H20 retention (HP, pulmonary edema, • 14 hypertension} • 15 Growth retardation and developmental del ay • 16 Erythropoietin failure (a nemia) • 17 Renal osteodys trophy . 18 • 19 FA17 p 436.1 • 20 Osteomalacia/rickets Defecti,·e mineralization of osteoid • 21 (osteomalacia) or cartilaginous gro" th plates • ' . . , , ... ' . • • a s 8 Lock Suspend End Block Item: 6 of 27 ~ 1 • M k -<:J 1>- Jil ~· !:';-~ QIO: 4116 ..L a r Previous Next Labfli!llues Notes Calculat o r

1 & & A 16-year-old Caucasian male presents to the emergency department for blood in his urine. The patient states that he has had a red tinge to 2 his urine after he recovered from a gastrointestinal infection. He is concerned this time because there is more blood than before. Upon further remembers that he recently recovered from an upper respiratory infection about 2- 3 days prior. The patient's 3 questioning the patient complete review of symptoms is negative, and physical examination reveals no other problems. Laboratory tests are pe1formed as well as urinalysis. 4 The urinalysis reveals mild hematuria without proteinuria. 5 • 6 Immunofluorescent staining of the tissue would most likely show which of the following?

• 7 : · 8 A. Cl complement in the afferent arteriole

. 9 B. C3 complement in the epithelium • 10 C. IgA deposits in the mesangial matrix • 11 D. IgG deposits in the endothelium • 12

• 13 E. IgM deposits in the basement membrane • 14

• 15

• 16

• 17

• 18 • 19

• 20

• 21 • a s 8 Lock Suspend End Block Item: 6 of 27 ~ 1 • M k -<:J 1>- Jil ~· !:';-~ QIO: 4116 ..L a r Prev ious Next Labfli!llues Notes Calculat or

1 & & Th e correct an swer is C. 640/o ch ose this. 2 This patient has IgA nephropathy, also known as Berger disease. This disease is characterized 3 histologically by mesangial deposits of IgA (as shown in the image) and clinically by nephritic syndrome. It commonly occurs concurrently with or just after an infection, particularly a respiratory 4 or gastrointestinal infection or pharyngitis. It is more common among Asians and Caucasians. The 5 development of hematuria with infection is a repeat event in the lives of these patients. IgA nephropathy generally occurs 2- 3 days after an upper respiratory infection, whereas 6 poststreptococcal glomerulonephritis occurs several weeks after a streptococcal infection. The

0 7 difference in time frame is a huge clue and a distingushing point between the two disease processes. Hemat ia Nephritic syndrome IgA nephropathy Pharyngitis Immunoglobu n A Glomerulonephritis Mesangium o8 Kidney disease Upper respiratory tract infection Streptococcus Nephritis Histology Gastrointestinal tract . 9 Image copyright ©2013 Zhuo eta/; • 10 licensee BioMed Central Ltd. • 11 A i s not correct. JO/o chose this. • 12 Cl complement component is found in patients with lupus glomerulonephritis and membranoproliferative glomerulonephritis. Mesangial • 13 proliferative lupus nephritis can manifest with moderate mesangial hypercellularity. Although membranous lupus nephritis can show thickened

0 14 glomerular basement membrane, it is unlikely this patient would have these findings with his clinical presentation. Membranoproliferative glomerulonephritis lupus nephritis Mesangial proliferative glomerulonephritis Glomerular basement membrane Basement membrane • 15 Glomerulonephritis Glomerulus (kidney) Mesangium Systemic lupus erythematosus Glomerulus Nephritis lupus erythematosus 0 16 B is not correct. 100/o c hose this. 0 17 C3 complement component is found in patients with focal segmental glomerulosclerosis (FSGS) and poststreptococcal glomerulonephritis • 18 (PSGN). Patients with PSGN have hypercellular glomeruli due to WBC infiltration and endothelial and mesangial proliferation. This patient's biopsy • 19 specimen has neither of these signs. Renal biopsy of patients with FSGS usually reveals hyaline deposits in some glomeruli . Focal segmental glomerulosclerosis Renal biopsy Glomerulonephritis Hyaline Biopsy Glomerulosclerosis Mesangium Glomerulus Glomerulus (kidney) Endothelium • 20 D is not correct . 120/o ch ose this • • 21 • In(; nPnn.;it.; in thP mp.;;mnium ;,nrl ;,lnnn thP. h;,sP.mPnt mPmhr;,np ;,rp nftPn fnunrt in nn.;t.;trpntnrnrr;,l nlnmP.rulonP.nhriti.; (PS(;N) ;,nrl for;,l a s 8 Lock Suspend End Block Item: 6 of 27 ~ . , . M k <:] t> al ~· ~ QIO: 4116 .l. ar Previous Next lab 'lifllues Notes Calculator -- ... _... - ... _.,. - - ... - - ... - ... - • 1 Membranoproliferative glomerulonephritis lupus nephritis Mesangial proliferative glomerulonephritis Glomerular basement membrane Basement membrane 2 Glomerulonephritis Glomerulus (kidney) Mesangium Systemic lupus erythematosus Glomerulus Nephritis lupus erythematosus 3 B is not correct. 100/o c hose t his. 4 C3 complement component is found in patients with focal segmental g lomerulosclerosis {FSGS} and poststreptococcal g lomerulonephritis {PSGN}. Patients with PSGN have hypercellular g lomeruli due to WBC infiltration and endothelial and mesangial proliferation. This patient's biopsy 5 specimen has neither of these signs. Renal biopsy of patients with FSGS usually reveals hyaline deposits in some g lomeruli. 6 Focal segmental glomerulosclerosis Renal biopsy Glomerulonephritis Hyaline Biopsy Glomerulosclerosis Mesangium Glomerulus Glomerulus (kidney) Endothelium • 7 0 is not correct. 1 20/o c hose t his. • 8 lgG deposits in the mesangium and along the basement membrane are often found in poststreptococcal g lomerulonephritis {PSGN} and focal segmental g lomerulosclerosis {FSGS}, along with lgM and C3. Patients with PSGN have hypercellular g lomeruli due to WBC infiltration and • 9 endothelial and mesangial proliferation. This patient's biopsy specimen has neither of these signs. Renal biopsy of patients with FSGS usually • 10 reveals hyaline deposits in some g lomeruli. Mesangium Focal segmental glomerulosclerosis Basement membrane Renal biopsy Immunoglobulin M Glomerulonephritis Immunoglobulin G Biopsy Hyaline · 11 Glomerulosclerosis Glomerulus Glomerulus (kidney) Endothelium Streptococcus • 12 E is not correct. 11 Ofo c hose t his . • 13 lgM deposits are usually found in focal segmental g lomerulosclerosis. This d isease presents with heavy proteinuria. Light microscopy usually • 14 shows hyaline deposits in some g lomeruli not limited to the mesangial matrix. • 15 Focal segmental glomerulosclerosis Proteinuria Immunoglobulin M Hyaline Glomerulus light microscopy Mesangium Glomerulus (kidney) Glomerulosclerosis Optical microscope • 16 • 17 • 18 Bottom Li ne : • 19 Berger d isease, or lgA nephropathy, is characterized histologically by deposits of lgA in the mesangial matrix. lgA nephropathy generally occu rs 2- 3 days after an upper respiratory infection, whereas poststreptococcal g lomerulonephritis occu rs several weeks after a streptococcal • 20 infection. The d ifference in time frame is a d istinguishing point between the two d isease processes. • 21 IgA nephropathy Immunoglobulin A Kidney disease Glomerulonephritis Upper respiratory tract infection Streptococcus Mesangium • 6 s 0 lock Suspend End Block Item: 6 of 27 ~ 1 • M k -<:J 1>- Jil ~· !:';-~ QIO: 4116 ..L a r Previous Next Labfli!llues Notes Calculat o r

1 & & FA17 p 564.1 2 Nephritic syndrome l\ephrltic syndrome= Inflammatory process. W hen glomeruli are inmlved, leads to hematuria 3 and RBC casts in urine. Associated with a~otemia, oliguria, hypertension (due to salt retention), 4 proteinuria. 5 Acute u.~ -glomeruli enlarged and h) percellular fJ. lost frequently seen in children. Occurs 6 poststre ptococca I IF- ("starry sky") granular appearance - 2-4 weeks after group A streptococcal

0 7 glo me rulo ne phritis ("lumpy-bumpy") (lJ due to lgC, lg\1, and C3 infection of pharynx or skin. Resolves o8 deposition along GB I and mesangium. spontaneousl). Type Ill h) persensiti' it) E~1-subep i thelial immune complex (IC) reaction. . 9 humps. Presents with peripheral and periorbital edema, • 10 cola-colored urine, hypertension. • 11 Positi,·e strep titers/serologies, l complement • 12 b ·cls (C3) due to consumption . • 13 Rapidly progressive LM and IF - crescent moon shape ~- Crescents Poor prognosis. Rapidly deteriorating renal

0 14 (c rescentic) consist of fibrin and plasma proteins (eg, C3b) function (days to weeks). glomerulo ne phritis with glomerular parietal cel ls, monocytes, • 15 macro phages. 0 16 Several d isease processes may result in this 0 17 pallern, in particular: . 18 • Goodpasture syndrome-type II llematuria/hemoptysis. • 19 hrpersensitivity reaction; antibodies to Treatment: emergent plasmapheresis. • 20 CBl\1 and alveolar basement membrane -+ linear IF • 21 • r.r:~nnlom::~tmi~ with nolv::~nuiiti~ (\.VPoPnPr) PR~-A I (;Afr-Ao (;A P::~1wi -immnnP (no lolr.~ a s 8 Lock Suspend End Block Item: 7 of 27 ~ 1 • M k -<:J 1>- Jil ~· !:';-~ QIO: 3514 ..L a r Previous Next Labfli!l tues Not es Calculat o r

1 & & A 66-year-old man presents for his annual physical complaining of a 1 -month history of"red pee." He has a 15-year history of poorly 2 controlled diabetes mellitus and routinely misses doses of his oral antihyperglycemic medications. He describes no urologic problems except tract infections as a young child. He drinks a "six-pack per night" and has a remote smoking history of one pack 3 two uncomplicated urina•·y per day for a couple of years while in college. He and his wife both recently retired from jobs in a textile mill. MRI of his pelvis is performed and is 4 shown in the image. 5

6

0 7 o8 . 9 • 10

• 11

• 12

• 13

0 14

• 15

0 16

0 17 • 18 • 19

• 20

• 21 • a s 8 Lock Suspend End Block 2 3 4 5 6

0 7 o8

0 9

0 10

0 11

0 12

0 13 What is the most concerning element of this patient's history that is specifically suggestive of the cause of his disease?

0 14 : • 15 A. Excessive alcohol consumption

0 16 B. Industrial toxin exposure 0 17 C. Poorly controlled diabetes 0 18

0 19 D. Smoking history • 20 E. Urinary tract infections as a child

0 21 • a s 8 Lock Suspend End Block Item: 7 of 27 ~ 1 • M k -<:J 1>- Jil ~· !:';-~ QIO: 3514 ..L a r Prev ious Next Lab fli!ltues Not es Calculat or

1 & & Th e correct an sw er i s B. 5 1 Ofo ch ose this. 2 The MRI of the pelvis is remarkable for dark thickening of his bladder (orange arrow in stem image), 3 which, given his history of hematuria, is concerning for transitional cell carcinoma. Exposure to aniline dyes is the strongest risk factor for bladder cancer; another risk factor is smoking. Aniline 4 dyes, which are found in synthetic dyes in the textile industry where he worked, are known to be 5 carcinogens that concentrate in the urinary system and that are stored in the bladder, thereby causing a significant risk for transitional cell carcinoma. The image here shows sheets of tumor cells 6 completely replacing the normal urothelium- pathology characteristic of transitional cell carcinoma. 7 -ran•'tion ce carcinoma Aniline Hematuria Bladder cancer Ur'nary system Pelvos '1ago etoc resonance maging · 8 Urinary bladder Neoplasm Carcinogen Carcinoma Epithelium Cancer Risk factor Dye . 9 • 10 Image courtesy of Wikipedia • 11

• 12 A i s not correct. 8 0fo c hose t his. • 13 This patient does have a history of excessive alcohol consumption; however, there is no strong evidence that suggests alcohol consumption is linked to the development of bladder cancer. He has other exposures in his past, such as industrial toxin exposure and smoking, that are more • 14 concerning for the development of bladder cancer. • 15 Bladder cancer Urinary bladder Cancer Alcoholic beverage Alcohol Toxin Tobacco smoking

• 16 C is not correct. 1 30/o ch ose this. • 17 Diabetes may be a moderate risk factor for the development of transitional cell carcinoma of the bladder. Although it is a risk factor for the development of transitional cell carcinoma, it is not as significant to developing this disease as this patient's prolonged exposure to aniline dyes . • 18 Transitional cell carcinoma Aniline Diabetes mellitus Urinary bladder Carcinoma Risk factor Epithelium • 19 D is n ot correct . 230/o ch ose this • • 20 Smoking is a risk factor for bladder cancer because of the carcinogenic compounds damaging the urothelium. This patient has a remote exposure • 21 of smoking so this would not be the most concerning part of his history for the development of bladder cancer. Studies have shown that smoking • a s 8 Lock Suspend End Block Item: 7 of 27 ~ . , . M k <:] t> al ~· ~ QIO: 3514 .l. ar Previous Next lab 'lifllues Notes Calculator

,-. -~. , ... -., 1

2 A is not correct. SO/o c hose t his. 3 This patient does have a history of excessive alcohol consumption; however, there is no strong evidence that suggests alcohol consumption is linked to the development of bladder cancer. He has other exposu res in his past, such as industrial toxin exposu re and smoking, that are more 4 concerning for the development of bladder cancer. 5 Bladder cancer Urinary bladder Cancer Alcoholic beverage Alcohol Toxin Tobacco smoking 6 C is not correct. 130/o c hose t his. 7 Diabetes may be a moderate risk factor for the development of transitional cell carcinoma of the bladder. Although it is a risk factor for the development of transitional cell carcinoma, it is not as significant to developing this d isease as this patient's prolonged exposu re to aniline dyes . • 8 Transitional cell carcinoma Aniline Diabetes mellitus Urinary bladder Carcinoma Risk factor Epithelium • 9 0 is not correct. 230/o c hose t his . • 10 Smoking is a risk factor for bladder cancer because of the carcinogenic compounds damaging the urothelium. This patient has a remote exposu re · 11 of smoking so this would not be the most concerning part of his history for the development of bladder cancer. Studies have shown that smoking cessation will decrease the risk for bladder cancer, but will not eliminate the risk of cancer. • 12 Urothelium Bladder cancer Smoking cessation Carcinogen Urinary bladder Risk factor Cancer Tobacco smoking • 13 E is not correct. 50fo c hose t his . • 14 Uncomplicated urinary tract infections (UT!s} in childhood, although rare in men, are often ben ign and are unlikely to ind icate any risk for the • 15 development of transitional cell carcinoma. Ch ron ic UT!s, however, may contribute to bladder cancer due to the chronic inflammatory changes that may resu lt. This patient has on ly had two UT!s, so this is unlikely to be a factor in his bladder cancer. • 16 Transitional cell carcinoma Bladder cancer Urinary tract infection Urinary system Urinary bladder Carcinoma Cancer Benignity Benign tumor • 17 • 18 Bottom Line : • 19 Transitional cell carcinoma of the bladder is strongly associated with exposu re to aniline dyes, commonly used in textile industry products . • 20 Other risk factors for developing transitional cell carcinoma include smoking, d iabetes, and use of the d rug cyclophosphamide. • 21 Transitional cell carcinoma Cyclophosphamide Aniline Diabetes mellitus Urinary bladder Carcinoma Epithelium • 6 s 0 lock Suspend End Block Item: 7 of 27 ~ - , . M k <:] t> al ~· ~ QIO: 3514 .l. ar Previous Next lab 'lifllues Notes Calculator

1 FA17 p 569.2 2 Transitional cell Most common tumor of urinary trac t system 3 carcinoma (can occur in renal calyces, renal pelvis, 4 ureters, and bladder) rJ Ill Cs an be uggested 5 by painless hematuria (no casts). Associated with problems in your Pee SAC: 6 Phenacetin, Smoking, Aniline dyes, and 7 Cyclophosphamide . • 8 • 9 • 10 · 11 • 12 FA17 p 219.2 • 13 Carcinogens • 14 TOXIN ORGAN IMPACT • 15 Aflatoxins (Aspergillus) Liver Hepatocellular carcinoma • 16 Alkylating agents Blood Leukemia/lymphoma • 17 Aromatic amines Bladder Transitional cell carcinoma • 18 (eg, benzidine, • 19 2-naphthylamine) • 20 Arsenic Liver Angiosarcoma • 21 Lung Lung cancer 6 s 0 lock Suspend End Block Item: 8 of 27 ~ 1 • M k -<:J 1>- Jil ~· !:';-~ QIO: 1784 ..L a r Previous Next Labfli!llues Notes Calculat o r

1 & & A 64-year-old man presents to his primary care physician for a routine physical. He is in good health, smokes half a pack of cigarettes daily, 2 and has no concomitant medical problems. However, he was diagnosed with hypertension 20 years ago and has been extremely nonadherent . The physician notes a blood pressure of 160/100 mm Hg and 1+ pitting edema to his knees. The physician also 3 in taking his daily medication notes bilateral bruits when auscultating the abdomen. 4

5 Which of the following will require careful monitoring of renal function with use? 6 : 7 A. ~-Blocker · 8 B. Angioplasty . 9 enzyme inhibitor • 10 C. Angiotensin-converting

• 11 0. Diuretics • 12 E. Surgical management • 13 • 14

• 15

• 16

• 17

• 18 • 19

• 20

• 21 • a s 8 Lock Suspend End Block Item: 8 of 27 ~ 1 • M k -<:J 1>- Jil ~· !:';-~ QIO: 1784 ..L a r Prev ious Next Labfli!llues Notes Calculat or

& & 1 Th e correct an sw er i s C. 590/o ch ose this. 2 This patient is most likely suffering from bilateral renal artery stenosis, indicated on physical exam by renal bruits. Stenosis of the renal arteries leads to a decrease in 3 Normal Kidneys Bilateral Renal Artery perfusion of the kidney, as shown in the flowchart. The result is a drop in Stenosis 4 intraglomerular pressure and glomerular filtration rate (GFR). The underperfused 5 kidneys respond by upregulating the renin-angiotensin-aldosterone system. • • Angiotensin II causes vasoconstriction of both the afferent and efferent arterioles; 6 because the vasoconstrictive effects are greater on efferent arterioles, GFR is 7 increased. Angiotensin-converting enzyme (ACE) inhibitors lower systemic blood ------• pressure and prevent the vasoconstrictive effects of angiotensin II to increase GFR. As Prevents •asoconst11ctJ.e effects and deafases renal ~rlwon 8 a result, this may both decrease renal perfusion and hinder the body's attempt to . 9 maintain effective GFR, which can lead to an increase in serum creatinine concentration. In a patient with bilateral renal artery stenosis the kidneys are • • 10 dependent on high levels of angiotensin to maintain function. Blocking this effect may l t.e.et of angiOtensin II l GfR • 11 further decrease kidney perfusion and function. Thus, renal function should be carefully monitored in patients with bilateral (not unilateral) renal stenosis who are • • 12 Ren1n release f as aresult ofloss of placed on ACE inhibitors. feedback inh1bi1ion ·---- • 13 Renal artery stenosis ACE inhibitor Renal artery Angiotensin Angiotensin-converting enzyme • 14 Creatinine Enzyme Vasoconstriction Renal function Renin-angiotensin system Angiotensin II Arteriole Kidney Glomerulus Blood pressure Blood plasma Glomerulus (kidney) Artery Stenosis Perfusion Physical examination Efferent arteriole • 15 ose this • • 16 A i s not correct. 1 20/o c h 13-Biockers, excellent antihypertensive medications, work through decreased cardiac output and decreased renin secretion. Although it is true • 17 that 131-blockade in the juxtaglomerular cells of the kidney leads to reduced activation of the renin-angiotensin-aldosterone system, this • 18 sympathetic input plays only a minor role in renal perfusion and blockage and so does not preclude usage of a 13-blocker in a patient with renal artery stenosis. 13-Biockers are therefore safe to use in renal artery stenosis but are contraindicated in asthma because of bronchoconstriction . • 19 ACE inhibitors, in contrast, reduce efferent arteriole tone (which relies more significantly on ACE function than adrenergic signaling), leading to a • 20 more noteworthy drop in GFR that can be harmful in renal artery stenosis. Rena artery stenosis Efferent arteriole cardiac output Renin Antihypertensive drug Renal artery Juxtaglomerular cell ACE inhibitor • 21 Arteriole • n-...:- ---1 ...... : ... -··-•-- r ...... : ... n ...... s...... : ...... v:..1 ...... r ...... : ... A:-....: ...... _ .... __ ... ..~ ...... ,. _ _ , .. n ...... l & ...... : ...... a s 8 Lock Suspend End Block Item:8of27 ~ . , . M k <:] t> al ~· ~ QIO: 1784 .l. ar Previous Next lab 'lifllues Notes Calculator

"" I 1?, I I ' 1 • Renal artery stenosis ACE inhibitor Renal artery Angiotensin Angiotensin-converting enzyme 2 Creatinine Enzyme Vasoconstriction Renal function Renin-angiotensin system Angiotensin II Arteriole Kidney Glomerulus Blood pressure Blood plasma Glomerulus (kidney) Artery Stenosis Perfusion Physical examination Efferent arteriole 3 4 A is no t correct . 1 20/o c hose t his. 13-Biockers, excellent antihypertensive medications, work through decreased cardiac output and decreased ren in secretion. Although it is true 5 that 13 1-blockade in the juxtaglomerular cells of the kidney leads to reduced activation of the ren in -angiotensin -aldosterone system, this 6 sympathetic input plays on ly a minor role in renal perfusion and blockage and so does not preclude usage of a 13-blocker in a patient with renal 7 artery stenosis. 13-Biockers are therefore safe to use in renal artery stenosis but are contraindicated in asthma because of bronchoconstriction . ACE inhibitors, in contrast, reduce efferent arteriole tone (which relies more significantly on ACE function than adrenergic signaling), lead ing to a 8 more noteworthy d rop in GFR that can be harmful in renal artery stenosis. • 9 Arteriole Renal artery stenosis Efferent arteriole Cardiac output Renin Antihypertensive drug Renal artery Juxtaglomerular cell ACE inhibitor Renin-angiotensin system Stenosis Perfusion Kidney Contraindication Asthma Adrenergic Renal function • 10 B is no t correct . 50/o c hose t his. · 11 Angioplasty is a minimally invasive procedure that involves placing intravascular stents in the renal artery, thereby restoring blood flow to the • 12 kidney. This form of therapy is the primary treatment for renal stenosis in symptomatic patients. Patency rates after angioplasty are strongly • 13 dependent on the size of the vessel treated and the quality of inflow and outflow through the vesseL Renal artery stenosis Angioplasty Renal artery Stenosis Stent Kidney laparoscopic surgery Blood vessel Minimally-invasive procedures Blood flow • 14 0 is no t correct . 2 1 Ofo c hose t his . • 15 In patients with hypertension caused by bilateral renal artery stenosis, both kidneys will be underperfused, so both will retain sodium and water • 16 by activating the ren in -angiotensin -aldosterone system. Diuretics can counteract this effect and control blood pressu re; therefore they are • 17 appropriate in this clinical scenario . Renal artery stenosis Renal artery Hypertension Renin-angiotensin system Stenosis Diuretic Blood pressure Kidney Sodium • 18 E is no t correct . 30/o c hose t his . • 19 Surgery is another therapeutic option for renal stenosis. It is ind icated particularly when angioplasty cannot be performed, as in completely • 20 occluded renal vessels. • 21 Angioplasty Renal artery stenosis Stenosis Kidney • 6 s 0 lock Suspend End Block Item:8of27 ~ . , . M k <:] t> al ~· ~ QIO: 1784 .l. ar Previous Next lab 'lifllues Notes Calculator

1 • sympathetic input plays on ly a m inor role in renal perfusion and blockage and so does not preclude usage of a 13-blocker in a patient with renal artery stenosis. 13-Biockers are therefore safe to use in renal artery stenosis but are contraindicated in asthma because of bronchoconstriction . 2 ACE inhibitors, in contrast, reduce efferent arteriole tone (which relies more significantly on ACE function than adrenergic signaling), lead ing to a 3 more noteworthy d rop in GFR that can be harmful in renal artery stenosis. Arteriole Renal artery stenosis Efferent arteriole Cardiac output Renin Antihypertensive drug Renal artery Juxtaglomerular cell ACE inhibitor 4 Renin-angiotensin system Stenosis Perfusion Kidney Contraindication Asthma Adrenergic Renal function 5 B is no t correct . 50fo c hose t his. 6 Angioplasty is a minimally invasive procedure that involves placing intravascular stents in the renal artery, thereby restoring blood flow to the 7 kidney. This form of therapy is the primary treatment for renal stenosis in symptomatic patients. Patency rates after angioplasty are strongly 8 dependent on the size of the vessel treated and the quality of inflow and outflow through the vesseL Renal artery stenosis Angioplasty Renal artery Stenosis Stent Kidney laparoscopic surgery Blood vessel Minimally-invasive procedures Blood flow • 9 0 is no t correct . 2 1 Ofo c hose t his . • 10 In patients with hypertension caused by bilateral renal artery stenosis, both kidneys will be underperfused, so both will retain sodium and water · 11 by activating the ren in -angiotensin -aldosterone system. Diuretics can counteract this effect and control blood pressu re; therefore they are • 12 appropriate in this clinical scenario . Renal artery stenosis Renal artery Hypertension Renin-angiotensin system Stenosis Diuretic Blood pressure Kidney Sodium • 13 E is no t correct . 30fo c hose t his . • 14 Surgery is another therapeutic option for renal stenosis. It is ind icated particularly when angioplasty cannot be performed, as in completely • 15 occluded renal vessels. • 16 Angioplasty Renal artery stenosis Stenosis Kidney • 17 • 18 Bo tto m Line : • 19 In bilateral renal artery stenosis, the kidneys are dependent on high levels of angiotensin to maintain renal perfusion and function. Although ACE inhibitors are not absolutely contraindicated in bilateral renal artery stenosis, they should be used with caution in these patients . • 20 Renal artery stenosis Renal artery Stenosis ACE inhibitor Perfusion Angiotensin Contraindication Kidney • 21 • 6 s 0 lock Suspend End Block Item: 8 of 27 ~ 1 • M k -<:J 1>- Jil ~· !:';-~ QIO: 1784 ..L a r Previous Next Labfli!llues Notes Calculat o r

1 & & FA17p577.1 2 Angiotensin­ Captopril, enalapril, lisi nopril, ramipri l. 3 converting enzyme 4 inhibitors 5 MECHANISM Inhibit ACE - l AT II - l CFR b) pre,cnting 6 constriction of efferent arterioles. f renin due 7 to loss of negati,·e feedback. Inhibition of CE also prevents inacti\'ation of brad) kinin, a 8 potent vasodilator. . 9 CliNICAL USE Hypertension, HF (lmortality), proteinuria, In chronic kidney disease (eg, diabetic • 10 diabetic nephropathy. Pre,entunfmorable nephropathy), l intraglomerular pressure, • 11 heart remodeling as a result of chronic slowing CBYl thickening. • 12 hypertension . • 13 ADVERSE EFFECTS Cough, Angioedema (due to f bradykinin; Captopril's CATCllll. • 14 contraindicated in Cl esterase inhibitor • 15 deficiency), Teratogen (fetal renal ma lfo rmations), f C reatinine (l CFR), • 16 llyperka lemia, and llypotension . Used wi th • 17 caution in bilateral renal artery stenosis . 18 because ACE inhibitors will furt her l CFR • 19 - renal failure . • 20 • 21 • FA17 p 553.2 a s 8 Lock Suspend End Block Item: 8 of 27 ~ 1 • M k -<:J 1>- Jil ~· !:';-~ QIO: 1784 ..L a r Previous Next Labfli!llues Notes Calculat o r

1 & & FA17 p 553.2 2 Changes in glomerular dynamics 3 Effect GFR RPF FF (GFR/ RPF) 4 Afferent arteriole constriction l l 5 Efferent arteriole constriction T l f 6 T plasma protem concentration l l 7 l plasma protein concentration f f Constriction of ureter l l 8 Dehydration l f . 9 • 10

• 11 FA17 p 290.2 Hype rtension Defined as persistent systolic BP ~ 140 mm Hg and/or d iastolic BP ~ 90 mm Hg • 12 RISK FACTORS salt intake, excess alcohol intake, fam ilr history; • 13 t age, obesity, diabetes, physical inactivity, excess African American> Caucasian> Asian . • 14 FEATURES 90% of hypertension is 1° (essential) and related lo f CO or t TPR. Remaining 10% mostly zot o • 15 renal/renovascular disease (eg, fibromuscular dysplasia (which has characteristic "string of beads" • 16 appearance of renal artery · ], ;~t h erosclerosis) and 1° hyperaldosteronism . • 17 Hype rte nsive urg e ncy -seve re ( ~ 1 80/~ 120 mm llg) hypertens ion without acute end-organ . 18 damage. • 19 Hype rtensive e me rge ncy-severe hypertension with evidence of acute end-organ damage (eg, encephalopathy, stroke, re tinal hemorrhages and e'udates, papilledema, MJ, HF, aortic dissection, • 20 kidney injury, microangiopathie hemolytic anemia, eclampsia). • 21 • a s 8 Lock Suspend End Block Item: 9 of 27 ~ 1 • M k -<:J 1>- Jil ~· !:';-~ QIO: 5 059 ..L a r Previous Next Labfli!l tues Notes Calculat o r

1 & & A 34-year-old man is rushed to the emergency department after experiencing a head-on collision with another car. In the field the patient 2 was tachycardic and hypotensive, with a pulse of 145/min and blood pressure of 65 mm Hg by palpation. His Glasgow Coma Scale score was taken to the operating room for surgical treatment of internal bleeding. The following day while 3 7 . At the hospital he is immediately recovering in the intensive care unit, the patient has a creatinine level of 2 .8 mg/dl. 4

5 Which laboratory result would indicate a prerenal etiology of his acute renal failure? 6 : 7 A. Blood urea nitrogen :creatinine rat io < 15 8 B. Fractional excretion of sodium > 2% . 9 >500 mOsm/ kg • 10 C. Urine osmolality

• 11 0. Urine osmolality <350 mOsm/kg • 12 E. Urine sodium >20 m Eq/L • 13 • 14

• 15

• 16

• 17

• 18 • 19

• 20

• 21 • a s 8 Lock Suspend End Block Item:9of27 ~ . , . M k <:] t> al ~· ~ QIO: 5059 .l. ar Previous Next lab 'lifllues Notes Calculator

1 • The correct a nswer is C. 550/o c hose t his. 2 Acute renal failure (ARF} is defined as an abrupt decline in renal function with increased creatinine (Cr} and blood urea nitrogen levels (BUN} 3 over a period of several days. There are three main categories of ARF: prerenal azotemia, intrinsic renal failure, and postrenal azotemia. In prerenal azotemia, decreased renal perfusion (often in the setting of hypotension) causes a decreased g lomerular filtration rate (GFR}. In 4 response, the kidney acts to retain salt and water in an attempt to increase blood pressu re and perfusion . Prerenal azotemia is d iagnosed by a 5 urine osmolality of >500 mOsm/kg, urine sodium level <20 m Eq/L, fractional excretion of sodium (FeNa) <1%, and a BUN :Cr ratio >20. In contrast, intrinsic renal failure is generally due to acute tubular necrosis, which leads to the obstruction of renal tubules with necrotic debris and 6 subsequent fluid backflow with decreased GFR. In this setting, the damaged renal tubules are unable to retain salt and water. Thus intrinsic renal 7 failure is d iagnosed by a urine osmolality <350 mOsm/kg, urine sodium level > 40 m Eq/L, FeNa >2%, and a BUN :creatinine ratio of <15. Finally, postrenal azotemia is caused by outflow obstruction (eg, kidney stones, ben ign prostatic hyperplasia, neoplasia) and can on ly occu r when such 8 obstruction is bilateraL In this setting there is also backflow of fluid with decreased GFR. Postrenal azotemia is d iagnosed by a urine osmolality 9 <350 mOsm/kg, urine sodium level > 40 m Eq/L, FeNa >2% (severe) or FeNa >1% (mild}. The postrenal BUN :Cr ratio can be quite variable. Blood urea nitrogen Renal function Azotemia Urea Acute tubular necrosis Creatinine Hypotension Acute kidney injury Urine Glomerulus Sodium Kidney 0 10 Blood pressure Osmolality Necrosis Urine osmolality Glomerulus (kidney) Renal tubules Nitrogen Perfusion oll A is no t correct . 1 50/o c hose t his. 0 12 A BUN :Cr ratio < 15 is consistent with intrinsic renal failure, not prerenal azotemia. Intrinsic renal failure is generally due to acute tubular 0 13 necrosis or ischemia/toxins; less commonly due to acute g lomerulonephritis. Azotemia Acute tubular necrosis Glomerulonephritis Acute proliferative glomerulonephritis Necrosis Kidney 0 14

0 15 B is no t correct . 100/o c hose t his.

0 16 FeN a > 2% is consistent with either intrinsic renal failure or severe forms of postrenal azotemia, but not prerenal azotemia. Intrinsic renal failure is generally due to acute tubular necrosis or ischemia/toxins; less commonly due to acute g lomerulonephritis. Postrenal azotemia is due to 0 17 bilateral outflow obstruction like stones, ben ign prostatic hyperplasia (BPH}, neoplasia, or congenital anomalies. Acute tubular necrosis Azotemia Glomerulonephritis Necrosis Neoplasm Acute proliferative glomerulonephritis Congenital disorder Kidney 0 18

0 19 0 is no t correct . 1 50/o c hose t his. Urine osmolality <350 mOsm/kg is consistent with either intrinsic renal failure or postrenal azotemia, but not prerenal azotemia. Intrinsic renal 0 20 failure is generally due to acute tubular necrosis or ischemia/toxins; less commonly due to acute g lomerulonephritis. Postrenal azotemia is due to 0 21 • bilateral outflow obstruction like stones, ben ign prostatic hyperplasia (BPH}, neoplasia, or congenital anomalies. • 6 s 0 lock Suspend End Block Item:9of27 ~ . , . M k <:] t> al ~· ~ QIO: 5059 .l. ar Previous Next lab 'lifllues Notes Calculator . . 1 A BUN :Cr ratio < 15 is consistent with intrinsic renal failure, not prerenal azotemia. Intrinsic renal failure is generally due to acute tubular 2 necrosis or ischemia/toxins; less commonly due to acute g lomerulonephritis. 3 Azotemia Acute tubular necrosis Glomerulonephritis Acute proliferative glomerulonephritis Necrosis Kidney 4 B is not correct. 100/o c hose t his. 5 FeN a > 2% is consistent with either intrinsic renal failure or severe forms of postrenal azotemia, but not prerenal azotemia. Intrinsic renal failure is generally due to acute tubular necrosis or ischemia/toxins; less commonly due to acute g lomerulonephritis. Postrenal azotemia is due to 6 bilateral outflow obstruction like stones, ben ign prostatic hyperplasia {BPH}, neoplasia, or congenital anomalies. 7 Acute tubular necrosis Azotemia Glomerulonephritis Necrosis Neoplasm Acute proliferative glomerulonephritis Congenital disorder Kidney 8 0 is not correct. 1 50/o c hose t his. 9 Urine osmolality <350 mOsm/kg is consistent with either intrinsic renal failure or postrenal azotemia, but not prerenal azotemia. Intrinsic renal failure is generally due to acute tubular necrosis or ischemia/toxins; less commonly due to acute g lomerulonephritis. Postrenal azotemia is due to 0 10 bilateral outflow obstruction like stones, ben ign prostatic hyperplasia {BPH}, neoplasia, or congenital anomalies. oll Benign prostatic hyperplasia Acute tubular necrosis Azotemia Glomerulonephritis Urine Necrosis Acute proliferative glomerulonephritis Neoplasm Congenital disorder Kidney Osmolality Urine osmolality 0 12 E is not correct. 50/o c hose t his. 0 13 A urine sodium level of >20 m Eq/ L is consistent with either intrinsic renal failure or postrenal azotemia, not prerenal azotemia. Intrinsic renal 0 14 failure is generally due to acute tubular necrosis or ischemia/toxins; less commonly due to acute g lomerulonephritis. Postrenal azotemia is due to

0 15 bilateral outflow obstruction like stones, ben ign prostatic hyperplasia {BPH}, neoplasia, or congenital anomalies. Benign prostatic hyperplasia Acute tubular necrosis Azotemia Glomerulonephritis Urine Sodium Neoplasm Necrosis Acute proliferative glomerulonephritis 0 16 Congenital disorder Kidney

0 17

0 18 Bottom Line : 0 19 Prerenal azotemia is d iagnosed by a urine osmolality >500 mOsm/kg, urine sodium level <20 m Eq/ L, fractional excretion of sodium <1%, and 0 20 a blood urea nitrogen/creatine ratio > 20. 0 21 Azotemia Sodium Urine Urine osmolality Osmolality • 6 s 0 lock Suspend End Block Item: 9 of 27 ~ 1 • M k -<:J 1>- Jil ~· !:';-~ QIO: 5 059 ..L a r Previous Next Labfli!l tues Notes Calculat o r

& & 1 FA17 p 571 .2 2 Acute kidney injury Acute kidney injury is de fin ed as an abrupt decline in renal functi on as measured by t creatinine 3 (acute renal failure) and t BU or by oliguria/anuria. 4 Prerenal azotemia Due to l RBF (eg, hypotension) - l CFR. la•/ 11 20 and BU1 retained by kidn ey in an allemptto 5 consen e volume - t BU '/creati nine ratio (BU l is reabsorbed, creatinine is not) and l FE'Ia· 6 Intrinsic renal failure Cenerallr due to acute tubular necrosis or ischemia/toxins; less commonly due to acute 7 glomerulonephritis (eg, RPCN, hcmol} lie uremic S) nd rome) or acute interstitial nephritis. ross necrotic tu bule 8 In ATN, patchy necrosis - debr is obstructing tubule and fluid backflow ac - l C FR. Urine has epithelial/granular c~1 s t s . BUt': reabsorption is impaired - l BUi\/crcatininc 9 rati o and t FE, a· • 10 Postrenal azotemia Due to outfl ow obstructi on (stones, BPI I, neoplasia, congenital anomalies). De\'elops only with • 11 bilateral obstructio n . • 12 Pre renal Intrinsic renal Postrenal • 13 • 14 Urine osmolality > 500 < 350 < 350 (mOsm/kg) • 15 Urine Na+ (mEq/l) <20 > 40 > 40 • 16 < I% < 1% (mild) • 17 FEN a >2% > 2% (severe) . 18 Serum BUN/Cr > 20 < 15 Varies • 19 • 20 • 21 FA17 p 572.2 • a s 8 Lock Suspend End Block Item: 9 of 27 ~ 1 • M k -<:J 1>- Jil ~· !:';-~ QIO: 5 059 ..L a r Previous Next Labfli!l tues Notes Calculat o r

1 & FA17 p 572.2 &

2 Acute tubular necrosis ~ l os t common cause of ac ute kidney injur) in hospitalized pati ents. Spontaneously resolves in 3 manCy cases. an be fatal, especially during initial oliguric phase. t FENa. 4 Key find ing: gra nular ( muddy brown") casts 3 stages: 5 I. Inciting e\·cnt 6 2 . .\rlaintenance phase- oliguric; lasts 1-3 " eeks; risk of hyperkalemia, metabolic acidosis, 7 urem1 a 8 3. Reco,·ery phase-polyuric; BU N" and serum creatinine fall; risk of hypokalemia 9 Can be caused by ischemic or nephrotoxic injury: Ischemic- 2° to l renal blood Aow (eg, h) potension, shock, sepsis, hemorrhage, H F'). Results • 10 in death of tubular cells that may slough into tubular lumen : (PCT and thick ascending limb • 11 are highl) susceptible to injury). • 12 • 1 ephrotoxic-2° to in jury resulti ng from toxic substances (eg, aminoglycosides, radiocontrast • 13 agents, lead, cisplatin, eth ylene glrcol), crush injurr {myoglobinuria), hemoglobinuria. PCT is • 14 particularly susceptible to injury. • 15 • 16

• 17 FA17 p 571.3 . 18 Consequences of renal Inability to make urine and excrete nitrogenous 2 forms of renal fai lure: acute (eg, Kr ) and • 19 failure wastes . chronic (eg, hypertension, diabetes mellitus, • 20 Consequences (.\1.\0 IJ U:\GER}: congenital anomalies). \ fetabolic Acidosis • 21 • a s 8 Lock Suspend End Block Item: 10 of 27 ~ 1 • M k -<:J 1>- Jil ~· !:';-~ QIO: 5 055 ..L a r Previous Next Labfli!l tues Not es Calculat o r

1 & & A 22-year-old man with no significant past medical history presents to the emergency depa•-tment with hemoptysis. He has never had an 2 episode like this before. He does recall having a cold approximately 3 weeks earlier and has been experiencing increased fatigue and he denies any current symptoms of rhinorrhea, sore throat, nausea, vomiting, or diarrhea. X-ray of the chest 3 breathlessness since then, but shows diffuse opacities in both lower lung fields, and urinalysis reveals RBCs in the urine. 4

5 Which of the following is the most likely diagnosis? 6 : 7 A. Acute poststreptococcal glomerulonephritis 8 B. Alport syndrome 9 • 10 C. Berger disease

• 11 0. Goodpasture syndrome • 12 E. Minimal change disease • 13 • 14 • 15 • 16 • 17 • 18 • 19 • 20 • 21 • a s 8 Lock Suspend End Block Item: 10 of 27 ~ 1 • M k -<:J 1>- Jil ~· !:';-~ QIO: 5055 ..L a r Prev ious Next Lab fli!ltues Not es Calculat or

1 & Th e co rrect a n sw er i s D. 6 30/o chose this. &

2 Goodpasture syndrome is a rare autoimmune d isorder in which antibodies directed against glomerular basement membrane (GBM) antigen cause a rapidly progressive glomerulonephritis. The 3 antibodies can also deposit in the alveolar basement membranes, resulting in hemoptysis. It is most 4 often seen in young men. Of note, Goodpasture syndrome can follow viral infections of the pulmonary system, thought to cause pulmonary damage leading to newly exposed epitopes, which sets off an 5 immunologic response. As in this case, the patient likely had a preceding viral pneumonia (which 6 would explain the x-ray findings). This condition can be diagnosed by the detection of the anti-GBM antibodies in the blood, in combination with a kidney biopsy that demonstrates a linear staining 7 pattern on immunofluorescence (like that shown here). Because this condition may rapidly lead to a 8 compromised airway or declining kidney funct ion, immediate airway protection, plasmapheresis (to remove the autoantibodies from circulat ion), and treatment with corticosteroids and 9 cyclophosphamide is indicated. 10 Hemoptysis Plasmapheresis Cyclophosphamide Autoimmune disease Immunofluorescence Image copyright© 2010 Patel et al; • 11 Glomerular basement membrane Basement membrane Rapidly progressive glomerulonephritis Glomerulonephritis licensee BioMed Central Ltd. Antibody Autoantibody Corticosteroid Pneumonia Biopsy Goodpasture syndrome Immune system Antigen • 12 Autoimmunity Fine-needle aspiration Pulmonary alveolus . 13 Epitope Glomerulus Kidney Renal function Respiratory tract Renal biopsy X-ray Anti-glomerular basement membrane antibody Glomerulus (kidney) Virus • 14 Viral pneumonia Cell membrane Respiratory system • 15 A i s not co rrect . 180/o c hose this • suffered from eit her impetigo or pharyngitis • 16 Poststreptococcal glomerulonephrit is typically affects child ren 2-14 years old who have recently caused by particular M types of streptococci (nephritogenic strains). Poststreptococcal glomerulonephritais is n immu ne-mediated d isease that • 17 develops 2-6 weeks after skin infection and 1- 3 weeks after streptococcal pharyngitis. The classic presentation is an acute nephritic picture with . 18 hematuria, pyuria, RBC casts, edema, hypertension, and oliguric renal failure. This patient's age and gender in combination with his symptoms of lung hemorrhage and hematuria make Goodpasture syndrome the most likely diagnosis . • 19 Streptococcal pharyngitis Pyuria Hematuria Impetigo Oliguria Glomerulonephritis Pharyngitis Hypertension Edema Streptococcus Goodpasture syndrome Nephritis • 20 Bleeding Urinary cast Skin infection Lung Immune system Autoimmunity Hemoptysis Kidney

• 21 B i s not correct . 6 0fo ch ose this • • a s 8 Lock Suspend End Block Item: 10 of 27 ~ . I • M k <:] t> al ~· ~ QIO: 5055 .l. ar Previous Next lab 'lifllues Notes Calculator

1 • hematuria, pyuria, RBC casts, edema, hypertension, and oliguric renal failure. This patient's age and gender in combination with his symptoms of • lung hemorrhage and hematuria make Goodpasture syndrome the most likely d iagnosis. 2 Streptococcal pharyngitis Pyuria Hematuria Impetigo Oliguria Glomerulonephritis Pharyngitis Hypertension Edema Streptococcus Goodpasture syndrome Nephritis 3 Bleeding Urinary cast Skin infection lung Immune system Red blood cell Autoimmunity Hemoptysis Kidney

4 B is not correct. 60/o c hose t his. 5 Alport d isease is an inherited d isorder caused by a mutation in collagen type IV, which is an important structural component in the basement membranes of the eyes, ears, and kidneys. Patients with this syndrome usually present in childhood with a combination of ocu lar defects, 6 deafness, and hematuria. Given this patient's age and clinical presentation, Goodpasture syndrome is a more likely d iagnosis than Alport 7 syndrome. Alport syndrome Hematuria Collagen Type IV collagen Goodpasture syndrome Basement membrane Genetic disorder Mutation Hearing loss Kidney 8 9 C is not correct. 11 Ofo c hose t his. Berger d isease, otherwise known as lgA nephropathy, is the most common cause of g lomerulonephritis worldwide. It is caused by the deposition 10 of lgA antibodies in the mesangium of the kidney and is often triggered by a recent upper respiratory infection. Classically, patients with Berger · 11 d isease present with new-onset frank hematuria, usually within days of an upper respiratory tract infection. Although the hematuria may persist for several days, renal function usually remains normal, and the long-term prognosis is relatively favorable. Given the patient's lung involvement, • 12 Goodpasture syndrome is a more likely d iagnosis than Berger d isease . • 13 IgA nephropathy Hematuria Mesangium Upper respiratory tract infection Glomerulonephritis Immunoglobulin A Kidney disease Goodpasture syndrome Antibody • 14 Kidney Renal function Respiratory tract lung Prognosis • 15 E is not correct. 20/o c hose t his . • 16 In the pediatric population, nephrotic syndrome {characterized by proteinuria, hypoalbuminemia, hyperlipidemia, and edema) is most commonly caused by m inimal change d isease {MCD}. MCD is definitively d iagnosed on renal biopsy, which reveals effacement of the foot process supporting • 17 the epithelial podocytes with weakening of slit-pore membranes on electron m icroscopy, but normal findings on light and immunofluorescent • 18 microscopy. The pathophysiology of this lesion is uncertain, although most agree a circulating cytokine alters capillary charge and podocyte integ rity. Treatment with steroids is ind icated based on clinical suspicion of MCD; renal biopsy is not necessary before treatment is initiated . • 19 Given this patient's age and clinical presentation, Goodpasture syndrome is a more likely d iagnosis than MCD. • 20 Podocyte Nephrotic syndrome Hypoalbuminemia Cytokine Proteinuria Minimal change disease Hyperlipidemia Renal biopsy Immunofluorescence Biopsy Edema Goodpasture syndrome Electron microscope Steroid Epithelium Capillary lesion Kidney Microscopy • 21 • 6 s 0 lock Suspend End Block Item: 10 of 27 ~ . I • M k <:] t> al ~· ~ QIO: 5055 .l. ar Previous Next lab 'lifllues Notes Calculator

1 • deafness, and hematuria. Given this patient's age and clinical presentation, Goodpasture syndrome is a more likely d iagnosis than Alport syndrome. 2 Alport syndrome Hematuria Collagen Type IV collagen Goodpasture syndrome Basement membrane Genetic disorder Mutation Hearing loss Kidney 3 C is not correct. 11 Ofo c hose t his. 4 Berger d isease, otherwise known as lgA nephropathy, is the most common cause of g lomerulonephritis worldwide. It is caused by the deposition 5 of lgA antibodies in the mesangium of the kidney and is often triggered by a recent upper respiratory infection. Classically, patients with Berger d isease present with new-onset frank hematuria, usually within days of an upper respiratory tract infection. Although the hematuria may persist 6 for several days, renal function usually remains normal, and the long-term prognosis is relatively favorable. Given the patient's lung involvement, 7 Goodpasture syndrome is a more likely d iagnosis than Berger d isease. IgA nephropathy Hematuria Mesangium Upper respiratory tract infection Glomerulonephritis Immunoglobulin A Kidney disease Goodpasture syndrome Antibody 8 Kidney Renal function Respiratory tract lung Prognosis 9 E is not correct. 20/o c hose t his. 10 In the ped iatric population, nephrotic syndrome (characterized by proteinuria, hypoalbuminemia, hyperlipidemia, and edema) is most commonly · 11 caused by minimal change d isease {MCD}. MCD is definitively d iagnosed on renal biopsy, which reveals effacement of the foot process supporting the epithelial podocytes with weakening of slit-pore membranes on electron microscopy, but normal findings on light and immunofluorescent • 12 microscopy. The pathophysiology of this lesion is uncertain, although most agree a circulating cytokine alters capillary charge and podocyte • 13 integ rity. Treatment with steroids is ind icated based on clinical suspicion of MCD; renal biopsy is not necessary before treatment is initiated. • 14 Given this patient's age and clinical presentation, Goodpasture syndrome is a more likely d iagnosis than MCD . Podocyte Nephrotic syndrome Hypoalbuminemia Cytokine Proteinuria Minimal change disease Hyperlipidemia Renal biopsy Immunofluorescence Biopsy Edema • 15 Goodpasture syndrome Electron microscope Steroid Epithelium Capillary lesion Kidney Microscopy • 16 • 17 Bottom Line : • 18 Goodpasture syndrome is a rare autoimmune d isorder in which antibodies d irected against g lomerular basement membrane antigen cause a • 19 rapid ly prog ressive g lomerulonephritis. • 20 Glomerular basement membrane Autoimmune disease Basement membrane Rapidly progressive glomerulonephritis Goodpasture syndrome Glomerulonephritis Antibody Antigen Autoimmunity Glomerulus Glomerulus (kidney) • 21 • 6 s 0 lock Suspend End Block Item: 10 of 27 ~ 1 • M k -<:J 1>- Jil ~· !:';-~ QIO: 5 055 ..L a r Previous Next Labfli!l tues Not es Calculat o r

& & 1 FA17 p 564.1 2 Nephritic syndrome l\ephrltic syndrome= InA ammatory process. When glomeruli are involved, leads to hematuria 3 and RBC casts in urine. Associated with a~o lemia, oliguria, hypertension (due to salt retention), 4 proteinuria.

5 Acute L~~ - glomeruli enlarged and h) percellular rJ. lost frequently seen in children. Occurs 6 poststre ptococca I IF- ("starry sky") granular appearance - 2-4 weeks after group A streptococcal pharynx or skin. Resolves 7 glo me rulo ne phritis ("lumpy-bumpy") Ill due to lgC, lg \ I, and C3 infection of deposition along GB I and mesangium. spontaneous!). Type Ill h) persensiti' it) 8 E~1 -s ubepi t helial immune complex (IC) reaction. 9 humps. Presents with peripheral and periorbital edema, 10 cola-colored urine, hypertension. • 11 Positi,·e strep titers/serologies, l complement • 12 b ·cls (C3) due to consumption . • 13 Rapidly progressive LM and IF- crescent moon shape [j. Crescents Poor prognosis. Rapidly deteriorating renal (crescentic) consist of fibrin and plasma proteins (eg, C3b) function (days to weeks). • 14 glo merulone phritis with glomerular parietal cel ls, monocytes, • 15 macro phages. • 16 Several d isease processes may result in this • 17 pallern, in particular: . 18 • Goodpasture syndrome-type II llemaluria/hemoptysis. hrpersensitivity reaction; antibodies to eatment: emergent plasmapheresis. • 19 Tr CBM and alveolar basement membrane • 20 -+ linear IF • 21 • Granulomatosis with polya ngiitis (Wegener) PR3-AJ CA/c-ANCA. Pauci-immune (no lg/C3 a s 8 Lock Suspend End Block Item: 10 of 27 ~ 1 • M k -<:J 1>- Jil ~· !:';-~ QIO: 5 055 ..L a r Previous Next Labfli!l tues Not es Calculat o r

1 & & FA17 p 566.1 2 Nephrotic syndrome t\ephrOtic syndrome-massive prOteinuria (> 3.5 glday) with hypoalbuminemia, resulting 3 edema, hyperlipidemia. Frothy urine" ith fatty casts. Due to podocyte damage disrupting 4 glomerular filtration charge barrier. \llay be 1° (eg, direct sclerosis of podocytes) or 2° (systemic 5 process [eg, diabetes] secondarily damages podocytes). Associated with hypercoagulable state (eg, 6 thromboembolism) due to antithrombin ( T) Ill loss in urine and t risk of infection (due to loss of in urine and soft tissue compromise by edema). 7 immunoglobulins Se,·ere nephritic syndrome may present with nephrotic S}lldrome features (nephritic-nephrotic 8 syndrome) if damage to CBl\ I is severe enough to damage charge barrier. 9 Minimal change Ll\11-normal glomeruli (lipid may be seen in \llost common cause of nephrotic S)lldrome 10 disease (lipoid PCT cells). in children. Often 1° (idiopathic) and ma) be • 11 nephrosis) we. triggered by recent infection, immunization, • 12 EM -effacement of foot processes t'J . immune stimulus. Rarely, may be zoto • 13 lymphoma (eg, cytokine-mediated damage). 1° disease has excellent response to cort icosteroids . • 14 Focal segmental -segmental sclerosis and hyalinosis : . of nephrotic syndrome in • 15 LM Most common cause glomerulosclerosis IF - often 8, but may be <±> for nonspeci fie focal African Americans and Hispanics. Can be ]0 • 16 deposits of Ig r, C3, Cl. (idiopathic) or zo to other conditions (eg, H IV • 17 EM -effacement of foot process similar to infection, sickle cell disease, heroin abuse, . 18 minimal change disease . massive obesity, interferon treatment, ch ronic • 19 kidney disease due to congenital malformations). • 20 1° disease has inconsistent response to steroids . ~ lay progress to chronic renal disease . • 21 t ______. r1 o _ 1 --·· • •• --L ----··- T'.r -•·rr. ___ __ ·•• -- _ lr-n,r .l· ~ \,f I - a s 8 Lock Suspend End Block Item: 11 of 27 ~ 1 • M k -<:J 1>- Jil ~· !:';-~ QIO: 1781 ..L a r Previous Next Labfli!llues Notes Calculat o r

1 & & A 56-year-old woman with a histo•·y of advanced cervical cancer who completed treatment with radiotherapy and chemotherapy several 2 months ago is admitted to the hospital for worsening symptoms of abdominal pain and severe continuous vomiting over a 3-day period. The patient's blood pressure is 108/75 mm Hg supine and 95/60 mm Hg standing. 3 4 Laboratory studies show: Serum pH: 7.55 5 Blood urea nitrogen: 31 mg/dL 6 Na+: 141 mEq/L K+: 3.3 mEq/L 7 Cl·: 90 mEq/L 8 HC03·: 35 mEq/L Creatinine: 1.2 mg/dL 9 Urine pH: 7.8 (normal4.5-8) 10 Urine Na•: 30 mEq/L • 11 • 12 Which of the following is the most likely cause of her serum and urine electrolytes? • 13 • 14 : A. Acute tubular necrosis • 15 B. History of chemotherapy • 16 • 17 C. Hypochloremic metabolic alkalosis

• 18 D. Lactic acidosis • 19 E. Paraneoplastic syndrome • 20 • 21 • a s 8 Lock Suspend End Block Item: llof27 ~ . , . M k <:] t> al ~· ~ QIO: 1781 .l. ar Previous Next lab 'lifllues Notes Calculator

1 • The correct answer is C. 580/o chose this. 2 After 3 days of vomiting, this patient has lost a substantial quantity of hydrogen and chloride ions and developed hypochloremic metabolic 3 alkalosis with elevated serum bicarbonate. As a resu lt, there is increased delivery of of sodium bicarbonate to the kidney, causing increased 4 concentration in the filtrate and increased urine pH {>6.5}. Some of the excess sodium is exchanged for potassium in the collecting tubule, lead ing to high urine sodium and potassium (as well as hypokalemia). This sodium wasting may mask the low urine sodium that is usually seen 5 during hypovolemia. After 3 days, the kidney is able to increase its resorption of sodium bicarbonate, and urine will become acidic ( <5.5} and 6 have higher sodium concentrations. Sodium bicarbonate Hypokalemia Hypovolemia Metabolic alkalosis PH Urine Chlorine Bicarbonate Kidney Potassium Collecting duct system Vomiting Sodium 7 Alkalosis Serum (blood) Nephron Metabolism Hydrogen Blood plasma Acid Chloride Filtration 8 A is not correct. 150/o chose this. 9 The patient is hypotensive and the blood urea nitrogen-to-creatinine ratio ind icates prerenal failure {defined as a BUN :Cr >20}. In some cases if 10 renal perfusion is particularly compromised, prerenal failure prog resses to acute tubular necrosis, which is characterized by a normal BUN :Cr ratio {10-15}, high fractional excretion of sodium, and muddy brown g ranular, epithelial cell casts. Although fractional excretion of sodium 11 cannot be calculated without a urine creatinine, the low urine sodium and acidic urine suggest a functional renal response to hypovolemia. • 12 Hypovolemia Urea Acute tubular necrosis Creatinine Epithelium Urine Hypotension Fractional sodium excretion Sodium Necrosis Perfusion Excretion Kidney

• 13 B is not correct. 160/o chose this. • 14 A history of chemotherapy would have no effect on the present state of this patient's urine. If she were to have had chemotherapy recently, a picture of tumor lysis syndrome might be expected with an elevated uric acid, elevated potassiu m, elevated phosphorous, and a decreased • 15 calcium on serum chemistry profile. • 16 Blood test Tumor lysis syndrome Chemotherapy Uric acid Blood plasma Urine Neoplasm Calcium Serum (blood) Potassium lysis Hypocalcaemia

• 17 0 is not correct. 4 0fo chose this. • 18 Lactic acidosis normally occu rs in the setting of shock. There is reduced tissue oxygenation, and as the oxygen requirements of cells are not met, they beg in to metabolize anaerobically, lead ing to a build-up of lactic acid and a subsequent metabolic acidosis. However, this patient is not in • 19 shock because her systolic blood pressu res are >90 mm Hg, which is sufficient to perfuse her peripheral tissues. Therefore, she does not need to • 20 excrete hydrogen ions to counteract a metabolic acidosis . lactic acidosis Metabolic acidosis lactic acid Acidosis Anaerobic organism Metabolism Oxygen saturation (medicine) Oxygen lactic acid fermentation • 21 • /\n~ornhi r rocnir::atinn Lhtrlrn,on Oorf1 •cinn Lhtrlrnnit U"r'\ 6 s 0 lock Suspend End Block Item: llof27 ~ . , . M k <:] t> al ~· ~ QIO: 1781 .l. ar Previous Next lab 'lifllues Notes Calculator

1 B is not correct. 1 60/o c hose t his. 2 A history of chemotherapy would have no effect on the present state of this patient's urine. If she were to have had chemotherapy recently, a picture of tumor lysis syndrome might be expected with an elevated uric acid, elevated potassium, elevated phosphorous, and a decreased 3 calcium on serum chemistry profile. 4 Blood test Tumor lysis syndrome Chemotherapy Uric acid Blood plasma Urine Neoplasm Calcium Serum (blood) Potassium lysis Hypocalcaemia

5 0 is not correct. 4 0fo c hose t his. 6 Lactic acidosis normally occu rs in the setting of shock. There is reduced tissue oxygenation, and as the oxygen requirements of cells are not met, they beg in to metabolize anaerobically, lead ing to a build-up of lactic acid and a subsequent metabolic acidosis. However, this patient is not in 7 shock because her systolic blood pressu res are >90 mm Hg, which is sufficient to perfuse her peripheral tissues. Therefore, she does not need to 8 excrete hyd rogen ions to counteract a metabolic acidosis. lactic acidosis Metabolic acidosis lactic acid Acidosis Anaerobic organism Metabolism Oxygen saturation (medicine) Oxygen lactic acid fermentation 9 Anaerobic respiration Hydrogen Perfusion Hydronium 10 E is not correct. JOfo c hose t his. 11 Cervical cancer is not associated with any common paraneoplastic effects. Hypercalcemia of malignancy is associated with cancers such as breast • 12 cancer, multiple myeloma, Hodgkin lymphoma, squamous cell carcinoma of the lung, and ovarian cancer. It occu rs via three maj or mechanisms: • 13 osteolytic metastases with local release of cytokines (osteoclast activating factors), tumor secretion of parathyroid hormone-related protein (PTHrP}, and tumor production of 1,25-dihyd roxyvitamin D (calcitriol) . • 14 Osteoclast Parathyroid hormone-related protein Multiple myeloma Calcitriol Hypercalcaemia Squamous-cell carcinoma Ovarian cancer Cervical cancer Malignancy

• 15 Paraneoplastic syndrome Metastasis Cytokine Breast cancer Neoplasm Vitamin 0 lymphoma Protein Parathyroid gland Cancer Osteolysis Hodgkin's lymphoma • 16 Carcinoma Squamous epithelial cell lung Cervix • 17

• 18 Bottom Line : • 19 Prolonged episodes of vomiting can resu lt in hypochloremic metabolic alkalosis. Du ring the first 3 days, the kidney is unable to increase its • 20 resorption of sodium bicarbonate, lead ing to high urine sodium and pH. After 3 days, the kidney is able to compensate . Sodium bicarbonate Metabolic alkalosis Chlorine Kidney Urine Sodium Vomiting Alkalosis Bicarbonate Hypochloremia Metabolism • 21 6 s 0 lock Suspend End Block Item: 11 of 27 ~ 1 • M k -<:J 1>- Jil ~· !:';-~ QIO: 1781 ..L a r Previous Next Labfli!llues Notes Calculat o r

1 & & FA17p561.1 2 Acid-base physiology 3 pH P predicted Pco2 - concomitant respiratory acidosis; if • 14 measured Pco2 < predicted Pco2 - concomitant rcspimtory alkalosis: • 15 Pco2 = 1.5 I HC0 3-J + 8 :!: 2 • 16 • 17 FA17 p 560.1 . 18 Potassium shifts SHIFTS K+INTO CEll (CAUSING HYPOKALEMIA) SHIFTS K• OUT OF CELL (CAUSING HYPERKALEMIA) • 19 Digitalis (blocks la+fK+ ATPase) • 20 llypo-osmolarity l lyperOsmolarity • 21 • a s 8 Lock Suspend End Block Item: 11 of 27 ~ 1 • M k -<:J 1>- Jil ~· !:';-~ QIO: 1781 ..L a r Previous Next Labfli!llues Notes Calculat o r

1 & ~-adrenergic agonist (f 1 a• JK-+ ATPasc) ~-blocker & 2 Insulin (f la•"JK+ TPasc) High blood Suga r (insulin defi ciency) 3 Insulin shifts K+ into cells Succinylcholine (f risk in burns/muscle trauma) 4 l lyperkalemia? DO LA~ S 5

6 FA17 p 576.3 7 Diuretics: electrolyte changes 8 Urine NaCI f with all diuretics (strength varies based on potency of diuretic effect). Serum _ aCl may decrease 9 as a result. 10 Urine K• f especially with loop and thiazide diuretics. Serum K+ may decrease as a result. 11 Blood pH l (acidemia): carbonic anhydrase inhibitors: l IIC03- reabsorption. K+ sparing: aldosterone • 12 blockade prevents K+ secretion and tt+ secreti on. Additionally, hyperkalemia leads to K+ entering • 13 all cells (,·ia H+fK+ exchanger) in exchange for H .. exiting cells . f (alkalemia): loop diuretics - Jil ~· !:';-~ QIO: 4118 ..L a r Previous Next Labfli!llues Not es Calculat o r

1 & & A 66-year-old woman with acute renal insufficiency and a medical history significant for a chronic disease undergoes a renal biopsy that 2 reveals glomerular capilla•·y subendothelial deposits, as shown in the image. 3 4 5 6 7 8 9 10 11 • 12 • 13 • 14 • 15 • 16 • 17 • 18 • 19 might have presented earlier in the patient's life? • 2 0 Which of the following symptoms • 21 : • a s 8 Lock Suspend End Block • 12 • 13 Which of the following symptoms might have presented earlier in the patient's life? • 14 : A. Achalasia • 15 • 16 B. Aortic aneurysm

• 17 C. Esophageal reflux • 18 D. Malar rash • 19 • 20 E. Morbilliform rash • 21 • a s 8 Lock Suspend End Block Item: 12 of 27 ~ . I • M k <:] t> al ~· ~ QIO: 4118 .l. ar Previous Next lab 'lifllues Notes Calculator

1 The correct a nswer is 0. 820/o c hose t his. 2 Wire-loop lesions of the g lomeruli (see the arrows in the image) are ind icative of lupus nephritis, also known as d iffuse proliferative 3 g lomerulonephritis, a complication of systemic lupus erythematosus (S LE ) that largely d ictates the patient's prognosis. SLE classically presents 4 with a malar "butterfly" rash on the face after exposure to sunlight. Various manifestations of SLE can affect d ifferent aspects of the body, such as the cardiovascular (pericarditis), respiratory (pleural effusion), musculoskeletal (arthralgias), and hematologic (anemia, various cytopenias) 5 systems. Treatment generally consists of immunosuppressive d rugs such as prednisone. 6 Pleural effusion lupus nephritis Systemic lupus erythematosus Prednisone Glomerulus (kidney) Pericarditis Cytopenia Glomerulus Anemia Nephritis 7 Glomerulonephritis Immunosuppression Immunosuppressive drug lupus erythematosus Pleural cavity Rash Prognosis Human musculoskeletal system Circulatory system 8 A is no t correct . 4 0fo c hose t his. 9 Although achalasia is not associated with systemic lupus erythematosus, it is associated with Chagas d isease resu lting from an infection 10 with Trypanosoma cruzi. This condition is due to the lower esophageal sphincter (LES} losing the ability to relax as a resu lt of damage to the the 11 myenteric (Auerbach) plexus. The high LES opening pressure and uncoordinated peristalsis lead to symptoms of progressive dysphagia (first solids, then solids and liquids}. A barium swallow study will show a d ilated esophagus with an area of d istal stenosis g iving the "bird 's beak" 12 appearance. • 13 Trypanosoma cruzi Chagas disease Peristalsis Systemic lupus erythematosus Dysphagia Upper gastrointestinal series Achalasia Esophagus • 14 lower esophageal sphincter Cardia Sphincter Stenosis lupus erythematosus Infection • 15 B is no t correct . 50fo c hose t his . Aortic aneurysms are not particularly associated with systemic lupus erythematosus but can be associated with connective tissue d isorders like • 16 Marfan syndrome or Ehlers-Dan los syndrome. Thoracic and abdominal aneurysms are the most common locations for aortic aneurysms to occu r. • 17 Thoracic aneurysms can also be associated with tertiary syphilis and can cause the classic "tree-bark" appearance of the aorta. Abdominal aneurysms usually occu r below the renal arteries, but above the aortic bifurcation . These are usually associated with male, hypertensive smokers • 18 over the age of 60 . • 19 Marfan syndrome Ehlers-Danlos syndrome Systemic lupus erythematosus Syphilis Aorta Renal artery Connective tissue Aneurysm Abdominal aortic aneurysm • 20 Aortic bifurcation Artery Connective tissue disease lupus erythematosus Hypertension Thorax Thoracic vertebrae Kidney

• 21 C is no t correct . 4 0fo c hose t his . 6 s 0 lock Suspend End Block Item: 12 of 27 ~ . I • M k <:] t> al ~· ~ QIO: 4118 .l. ar Previous Next lab 'lifllues Notes Calculator

1 lower esophageal sphincter Cardia Sphincter Stenosis lupus erythematosus Infection 2 B is no t correct . 50fo c hose t his. 3 Aortic aneurysms are not particularly associated with systemic lupus erythematosus but can be associated with connective tissue d isorders like 4 Marfan syndrome or Ehlers-Dan los syndrome. Thoracic and abdominal aneurysms are the most common locations for aortic aneurysms to occu r. Thoracic aneurysms can also be associated with tertiary syphilis and can cause the classic "tree-bark" appearance of the aorta. Abdominal 5 aneurysms usually occu r below the renal arteries, but above the aortic bifurcation . These are usually associated with male, hypertensive smokers 6 over the age of 60. Marfan syndrome Ehlers-Danlos syndrome Systemic lupus erythematosus Syphilis Aorta Renal artery Connective tissue Aneurysm Abdominal aortic aneurysm 7 Aortic bifurcation Artery Connective tissue disease lupus erythematosus Hypertension Thorax Thoracic vertebrae Kidney 8 C is no t correct . 4 0fo c hose t his. 9 Esophageal reflux is not particularly associated with lupus. Risk factors include obesity, hiatal hernia, pregnancy, and scleroderma. This condition 10 is due to a decrease in the lower esophageal sphincter tone resu lting in gastric contents refluxing into the esophagus. It commonly presents as heartburn and regurg itation upon laying down. Other presenting symptoms may be nocturnal cough, dyspnea, or adult-onset asthma. 11 Dyspnea Scleroderma Heartburn Hiatus hernia lower esophageal sphincter Asthma Esophagus Gastroesophageal reflux disease Regurgitation {digestion) Cardia 12 Cough Obesity Hernia Pregnancy Reflux Esophageal cancer Systemic lupus erythematosus Sphincter lupus erythematosus • 13 E is no t correct . 50fo c hose t his . • 14 Morbilliform rashes refer to rashes that resemble measles. The rash consists of erythematous macular lesions usually 2-10 mm in d iameter and • 15 may be seen in many conditions includ ing viral exanthemas, Kawasaki d isease, or a d rug eruption (most commonly due to antibiotics) . Kawasaki disease Measles Morbilliform Erythema Antibiotics Rash Drug eruption • 16 • 17

• 18 Botto m Line : • 19 Lupus nephritis, also known as d iffuse proliferative g lomerulonephritis, commonly presents with wire-loop g lomerular lesions on renal biopsy . Symptoms characteristic of lupus include malar rash, arthritis, oral and nasal ulcers, fever, and malaise . • 20 lupus nephritis Glomerulonephritis Nephritis Malaise lupus erythematosus Systemic lupus erythematosus Biopsy Renal biopsy Malar rash Arthritis Glomerulus Rash Fever Glomerulus (kidney) Kidney • 21 6 s 0 lock Suspend End Block Item: 12 of 27 ~ . I • M k <:] t> al ~· ~ QIO: 4118 .l. ar Previous Next lab 'lifllues Notes Calculator

1 lower esophageal sphincter Cardia Sphincter Stenosis lupus erythematosus Infection 2 B is no t correct . 50fo c hose t his. 3 Aortic aneurysms are not particularly associated with systemic lupus erythematosus but can be associated with connective tissue d isorders like 4 Marfan syndrome or Ehlers-Dan los syndrome. Thoracic and abdominal aneurysms are the most common locations for aortic aneurysms to occu r. Thoracic aneurysms can also be associated with tertiary syphilis and can cause the classic "tree-bark" appearance of the aorta. Abdominal 5 aneurysms usually occu r below the renal arteries, but above the aortic bifurcation . These are usually associated with male, hypertensive smokers 6 over the age of 60. Marfan syndrome Ehlers-Danlos syndrome Systemic lupus erythematosus Syphilis Aorta Renal artery Connective tissue Aneurysm Abdominal aortic aneurysm 7 Aortic bifurcation Artery Connective tissue disease lupus erythematosus Hypertension Thorax Thoracic vertebrae Kidney 8 C is no t correct . 4 0fo c hose t his. 9 Esophageal reflux is not particularly associated with lupus. Risk factors include obesity, hiatal hernia, pregnancy, and scleroderma. This condition 10 is due to a decrease in the lower esophageal sphincter tone resu lting in gastric contents refluxing into the esophagus. It commonly presents as heartburn and regurg itation upon laying down. Other presenting symptoms may be nocturnal cough, dyspnea, or adult-onset asthma. 11 Dyspnea Scleroderma Heartburn Hiatus hernia lower esophageal sphincter Asthma Esophagus Gastroesophageal reflux disease Regurgitation {digestion) Cardia 12 Cough Obesity Hernia Pregnancy Reflux Esophageal cancer Systemic lupus erythematosus Sphincter lupus erythematosus • 13 E is no t correct . 50fo c hose t his . • 14 Morbilliform rashes refer to rashes that resemble measles. The rash consists of erythematous macular lesions usually 2-10 mm in d iameter and • 15 may be seen in many conditions includ ing viral exanthemas, Kawasaki d isease, or a d rug eruption (most commonly due to antibiotics) . Kawasaki disease Measles Morbilliform Erythema Antibiotics Rash Drug eruption • 16 • 17

• 18 Botto m Line : • 19 Lupus nephritis, also known as d iffuse proliferative g lomerulonephritis, commonly presents with wire-loop g lomerular lesions on renal biopsy . Symptoms characteristic of lupus include malar rash, arthritis, oral and nasal ulcers, fever, and malaise . • 20 lupus nephritis Glomerulonephritis Nephritis Malaise lupus erythematosus Systemic lupus erythematosus Biopsy Renal biopsy Malar rash Arthritis Glomerulus Rash Fever Glomerulus (kidney) Kidney • 21 6 s 0 lock Suspend End Block Item: 12 of 27 ~ 1 • M k -<:J 1>- Jil ~· !:';-~ QIO: 4118 ..L a r Previous Next Labfli!llues Not es Calculat o r

1 & &

2 FA17 p 564.1 3 Nephritic syndrome l\ephrltic syndrome= Inflammatory process. When glomeruli are inmlved, leads to hematuria 4 and RBC casts in urine. Associated with azotemia, oliguria, hypertension (due to salt retention), proteinuria. 5 Acute (J. 6 LM-glomeruli enlarged and h) percellular ~lost freq uently seen in children. Occurs post streptococcal IF' - ("starry sky") granular appearance - 2-4 weeks after group A streptococcal 7 glomerulonephritis ("lumpy-bumpy") D due to lgC, lg\ I, and C3 infection of pharynx or skin. Resolves 8 deposition along CB I and mesangium. spontaneous!). Type Ill h) persensiti' it ~ 9 E~-subepithelial immune complex (IC) reaction. 10 humps. Presents with peripheral and periorbital edema,

11 cola-colored urine, hypertension. Positi,·e strep titers/serologies, L complement 12 b·cls (C3) due to consumption . • 13 Rapidly progressive LM and IF- crescent moon s h ape ~- C rescents Poor prognosis. Rapidly deteriorating renal • 14 (crescentic) consist of fibrin and plasma proteins (eg, C3b) function (days to weeks). • 15 glomerulonephritis with glomerular parietal cells, monocytes, • 16 macrophages . • 17 Several d isease processes may result in th is pattern, in particular: . 18 • Goodpasture syndrome-type II llemaluria/hemoptysis. • 19 hypersensitivity reaction; antibodies to Treatment: emergent plasmapheresis. • 20 CB.t\1 and alveolar basement membrane • 21 -+ linear IF • a s 8 Lock Suspend End Block Item: 12 of 27 ~ 1 • M k -<:J 1>- Jil ~· !:';-~ QIO: 4118 ..L a r Previous Next Labfli!llues Not es Calculat o r

1 & & FA17 p 443.1 2 Systemic lupus erythematosus 3 SYMPTOMS Classic presentation: rash, joint pain, and fe,er, R. \ $ 11 OR P.\1 ~ : 4 most commonly in a female of reproductive Rash (malar · or discoid : ) 5 age and African-American descent. \ rthritis (nonerosive) 6 Libman-Sacks Endocarditis-nonbacteri- Jil ~· !:';-~ QIO: 1792 ..L a r Previous Next Labfli!llues Notes Calculat o r

1 & & A 54-year-old homeless woman is found unconscious on the street. On admission to the emergency department, her laboratory tests show : 2 Na+: 137 mEq/L 3 K+: 3 .3 mEq/L 4 cl-: 112 mEq/L 5 HC03-: 15 mEq/L Arterial blood gas on room air: pH 7. 28 6 Pco2: 28 mm Hg 7 Po2: 90 mm Hg 8 Which of the following most likely caused her acidosis? 9 10 : A. A late-stage aspirin overdose 11 12 B. Diabetic ketoacidosis

• 13 C. Severe diarrhea • 14 D. Severe underperfusion of her peripheral muscles • 15 E. Uremia • 16 • 17 • 18 • 19 • 20 • 21 • a s 8 Lock Suspend End Block Item: 13 of 27 ~ . I • M k <:] t> al ~· ~ QIO: 1792 .l. ar Previous Next lab 'lifllues Notes Calculator

1 • The correct answer is C. 540fo chose t his. 2 To understand this metabolic abnormality, first look at the pH and then the bicarbonate (HC03- ) and partial pressure of carbon d ioxide (Pc02). 3 Her pH is 7. 28, which ind icates a form of acidosis. Metabolic acidosis is the presence of low pH with low plasma HCo3-, in this case 15 m Eq/L (normal = 22 - 26 m Eq/L}; she is suffering from metabolic acidosis. Her lungs are blowing off more C02 in order to raise the pH . The causes of 4 metabolic acidosis are events that either increase acid levels (eg, d iabetic ketoacidosis, uremia, hypovolemic shock) or decrease the amount of 5 base present (eg, d iarrhea, ). Metabolic acidosis can be subdivided further into non-anion gap and anion gap metabolic acidosis. The anion gap can be calculated using the following equation : An ion Gap = Na+ - (CI- + HC03- ). A normal anion gap is 8 - 12. This patient's anion 6 gap is equal to 10 [ 137 - (112 + 15}). Therefore, there is no anion gap acidosis. If the primary cause of acidosis is a loss of HC03-, there will be 7 an increase in chloride and the anion gap will be normal, as is the case with severe d iarrhea. Of the answers listed, on ly d iarrhea can cause a non-anion gap acidosis. 8 Diabetic ketoacidosis Uremia Hypovolemia Metabolic acidosis PH Carbon dioxide Partial pressure Anion gap Ketoacidosis Bicarbonate Acidosis Kidney

9 Blood plasma Diarrhea Chloride Anion Diabetes mellitus lung Equivalent (chemistry) Carbon Metabolism 10 A is not correct. 120/o chose t his. 11 Late-stage salicylate overdose causes an anion gap acidosis (ingested salicylic acid is the unmeasured anion). Lab values for a patient with anion 12 gap acidosis would be: pH < 7.35, HC03- < 20 m Eq/L, Pco2 36-44 mm Hg. Also, if the anion gap were to be calculated, Na+ - (CI- + HC03- ), it would be >12 m Eq/L. 13 Salicylic acid Anion gap PH Acidosis Anion Drug overdose • 14 B is not correct. 190/o chose t his . • 15 Diabetic ketoacidosis (DKA) causes a severe anion gap acidosis (the unmeasured anions in this case are ketoacids}. In d iabetic ketoacidosis, • 16 oxaloacetate is depleted for g luconeogenesis. This stalls the tricarboxylic acid cycle, which shunts g lucose and free fatty acids toward the production of ketone bodies. Lab values for a patient with anion gap acidosis would be: pH < 7.35, HCo3- < 20 m Eq/L, Pco2 36-44 mm Hg. Also, • 17 if the anion gap were to be calculated, Na+ - (cl- + HC03- ), it would be> 12 m Eq/L. • 18 Diabetic ketoacidosis Gluconeogenesis Oxaloacetic acid Ketone bodies Citric acid cycle Ketone Anion gap PH Ketoacidosis Acidosis Anion Diabetes mellitus Fatty acid Free fatty acids Keto acid Equivalent (chemistry) Anions • 19 6 0/o • 20 0 is not correct . chose t his . The patient in this question has a non-anion gap acidosis. Un derperfusion causes anion gap acidosis (the anion in this case is lactic acid}. Lab • 21 • values for a oatient with anion aao acidosis would be: oH < 7 .35, HCo't- < 20 m Ea/L. PCO? 36-44 mm Ha. Also, if the anion aao were to be 6 s 0 lock Suspend End Block Item: 13 of 27 ~ . I • M k <:] t> al ~· ~ QIO: 1792 .l. ar Previous Next lab 'lifllues Notes Calculator

T • • - • 1 Salicylic acid Anion gap PH Acidosis Anion Drug overdose 2 B is no t correct . 190/o c hose t his. 3 Diabetic ketoacidosis (DKA) causes a severe anion gap acidosis {the unmeasured anions in this case are ketoacids}. In d iabetic ketoacidosis, 4 oxaloacetate is depleted for g luconeogenesis. This stalls the tricarboxylic acid cycle, which shunts g lucose and free fatty acids toward the prod uction of ketone bod ies. Lab values for a patient with anion gap acidosis would be : pH < 7.35, HC03- < 20 m Eq/ L, Pco2 36- 44 mm Hg. Also, 5 if the anion gap were to be calculated, Na+ - {c l- + HCo3- ), it would be> 12 m Eq/ L. 6 Diabetic ketoacidosis Gluconeogenesis Oxaloacetic acid Ketone bodies Citric acid cycle Ketone Anion gap PH Ketoacidosis Acidosis Anion Glucose Diabetes mellitus Fatty acid Free fatty acids Keto acid Equivalent (chemistry) Anions 7

8 0 is no t correct . 6 0/o c hose t his. The patient in this q uestion has a non-anion gap acidosis. Un derperfusion causes anion gap acidosis (the anion in this case is lactic acid }. Lab 9 values for a patient with anion gap acidosis would be : pH < 7 .35, HC03- < 20 m Eq/ L, Pco2 36- 44 mm Hg. Also, if the anion gap were to be 10 calculated, Na+ - {c l- + HC03- ), it would be >12 m Eq/ L. Anion gap lactic acid PH Acidosis Anion Equivalent (chemistry) 11

12 E is no t correct . 90/o c hose t his. Uremia ind icates renal failure. The inability of the kid ney to excrete organic acids leads to an anion gap acidosis. Re nal failure also causes 13 hyperkalemia, because the kid ney is unable to excrete K+. Lab values for a patient with anion gap acidosis would be : pH < 7.35, HC03- < 20 • 14 m Eq/ L, Pco2 36- 44 mm Hg. Also, if the anion gap were to be calculated, Na+ - {CI- + HC03- ), it would1 be > 2 m Eq/ L . Hyperkalemia Uremia Anion gap PH Kidney Acidosis Organic acid Anion Equivalent (chemistry) • 15

• 16 • 17 Bo tto m Line :

• 18 In patients with acidemia {pH < 7.4}, next check the partial pressure of carbon d ioxide {Pco2); a Pco2 1evel < 40 with a normal anion gap metabolic acidosis ind icates d iarrhea, renal tubular acidosis, g lue sniffing, or hyperchloremia. The eq uation used to determine the anion gap is • 19 as follows: Anion gap = Na+ - (c l- + HC03- ). • 20 Anion gap Metabolic acidosis Carbon dioxide Renal tubular acidosis PH Partial pressure Diarrhea Acidosis Intoxicative inhalant Hyperchloremia Anion Chloride Kidney Metabolism • 21 • 6 s 0 lock Suspend End Block 13 27 • M k ~ £!1}>' !!":-~ Item: of ~ 1 a r -<:J I> • QIO: 1792 ..L Previous Next Lab lues Notes Calculat o r

1 & & FA17 p 561 .2 2 Acidosis and alkalosis 3 4 Check arterial pH 5 pH< 7.35 pH> 745 6 Ac.idemia Alkalemia 7 < Hg 8 Pco2 36 mm 9 10 RespiratOf}' Respiratory MetaboUc acidos1s MetaboUc alkalOSIS acidosis alkalosis 11 12 n gap Hypowntilation Check anio Hyptrvtntilation H• loss/HC01- excess 13 =Na • - (CI- t HCO; l Airway obstruction Hysteria Loop diuretics • 14 Acute lung disease Hypoxemia (eg, high altitude) Vomiting Chronic lung disease Sallcylates (early) Antacid use • 15 Opioids, sedatives Tumor Hyperaldosteronism Weakening of respiratory • 16 Pulmonary embolism muscles • 17 . 18 > 12 mEq/L 8-12 mEq/L • 19 Pco • t 4S 40mmHg • 20 40 Resporatory 1 Anion gap Nonnal anion gap aCidosiS • 21 3S • MUDPILES: HARDASS: ~ a ------s 8 Lock Suspend End Block Item: 14 of 27 ~ 1 • M k -<:J 1>- Jil ~· !:';-~ QIO: 3520 ..L a r Previous Next Labfli!l tues Not es Calculat o r

1 & & A 36-year-old man presents for an annual flu shot. He has been treated for hypertension with an angiotensin-converting enzyme inhibitor 2 since age 30 years. He denies smoking and alcohol use. Family history is notable for his father requiring dialysis at age 50. An older brother nephrectomy to decompress intra-abdominal organs. On examination, the patient has a sitting blood pressure 3 recently underwent unilateral of 135/ 90 mm Hg. An abdominal ultrasound reveals multiple cysts within the liver. 4 5 His likely genetic renal pathology is associated with an increased incidence of which of the following? 6 : 7 A. Cerebral aneurysm 8 B. Horseshoe kidney 9 Macular cherry-red spots 10 c. 11 o. Pott disease

12 E. Potter syndrome 13 • 14

• 15

• 16

• 17

• 18 • 19

• 20

• 21 • a s 8 Lock Suspend End Block Item: 14 of 27 ~ . I • M k <:] t> al ~· ~ QIO: 3520 .l. ar Previous Next lab 'lifllues Notes Calculator

1 • The correct answer is A. 740/o chose this. 2 Autosomal dominant polycystic kidney d isease (ADPKD) is associated with berry aneurysms in the circle of Willis and with cyst formation not on ly 3 in the kidneys, but also in the liver, pancreas, and spleen. Patients with this condition are also more prone to developing aortic root aneurysms, 4 mitral valve prolapse, and early-onset hypertension, with an average age at onset of 30 years. Later in the course, kidneys can reach sizes triple their normal volumes, often resu lting in d isabling symptoms due to pressu re on intra-abdominal organs. ADPKD does not manifest at a young 5 age. Autosomal recessive polycystic kidney d isease is d iagnosed most often in ped iatric patients; it is associated less often with cyst formation in 6 multiple visceral organs and with the formation of berry aneurysms. Circle of Willis Mitral valve prolapse Polycystic kidney disease Autosomal dominant polycystic kidney disease Dominance (genetics) Mitral valve Pancreas Spleen 7 Hypertension Autosomal recessive Kidney liver Aneurysm Kidney disease Aorta Cyst Ascending aorta Organ (anatomy) 8 Autosomal recessive polycystic kidney disease 9 B is not correct. 70/o chose this. 10 Horseshoe kidney is associated with many renal d iseases, but it does not involve the extrarenal findings described in the question . The maj ority 11 of patients with a horseshoe kidney are asymptomatic and are usually d iagnosed by coincidence. If not asymptomatic, a patient may have hematuria, hydronephrosis or renal calculi. A horseshoe kidney may also be found with another urologic anomaly, genital anomalies or 12 syndromes like Turner syndrome. 13 Horseshoe kidney Turner syndrome Hydronephrosis Hematuria Kidney stone Kidney disease Kidney Urology 14 C is not correct. 60/o chose this. • 15 Ch ild ren with inborn errors of metabolism can present with ophthalmolog ic findings such as cherry-red spots in the macula. An example is Niemann-Pick d isease, a lysosomal storage d isease in which a sphingomyelinase deficiency resu lts in accumulation of sphingomyelin. This d isease • 16 may manifest at d ifferent ages with varying degrees of splenomegaly and neurologic deficits. • 17 lysosomal storage disease Inborn error of metabolism Splenomegaly Sphingomyelin Niemann-Pick disease Sphingomyelin phosphodiesterase lysosome

• 18 Metabolism Ophthalmology Neurology Macula of retina • 19 0 is not correct. 50/o chose this . Pott d isease is a tuberculosis infection of the vertebrae, most commonly the lower thoracic and upper lumbar. In some patients, Pott d isease, • 20 manifests with fever and weight loss. The most common symptom is local pain, which increases in severity over weeks to months. Muscle spasm • 21 and rig id ity may cause a characteristic erect posture and "aldermanic" gait. This gait is described as an ind ividual that walks with short steps to • 6 s 0 lock Suspend End Block Item: 14 of 27 ~ . I • M k <:] t> al ~· ~ QIO: 3520 .l. ar Previous Next lab 'lifllues Notes Calculator

1 • Niemann-Pick d isease, a lysosomal storage d isease in which a sphingomyelinase deficiency resu lts in accumulation of sphingomyelin. This d isease • may manifest at d ifferent ages with varying degrees of splenomegaly and neurologic deficits. 2 lysosomal storage disease Inborn error of metabolism Splenomegaly Sphingomyelin Niemann-Pick disease Sphingomyelin phosphodiesterase lysosome 3 Metabolism Ophthalmology Neurology Macula of retina

4 0 is no t correct. 50/o c hose this. 5 Pott d isease is a tuberculosis infection of the vertebrae, most commonly the lower thoracic and upper lumbar. In some patients, Pott d isease, manifests with fever and weight loss. The most common symptom is local pain, which increases in severity over weeks to months. Muscle spasm 6 and rig id ity may cause a characteristic erect posture and "aldermanic" gait. This gait is described as an ind ividual that walks with short steps to 7 avoid shaking the spine. Pott disease Tuberculosis Vertebra Gait (human) Weight loss Thorax Thoracic vertebrae lumbar Fever Vertebral column Infection lumbar vertebrae Muscle 8 9 E is no t correct. S O/o c hose this. Potter syndrome is an in utero condition that resu lts from conditions associated with oligohydramnios, such as renal agenesis or autosomal 10 recessive polycystic kidney d isease. It is not associated with adults afflicted with autosomal dominant polycystic kidney d isease. The 11 oligohydramnios leads to compression of the developing fetus, lead ing to limb deformities and facial anomalies (low-set ears and retrognathia). The chest also becomes compressed and will lead to pulmonary hypoplasia, which is the cause of death for an ind ividual with Potter syndrome. 12 The features of Potter syndrome may be remembered by using the mnemonic POTTER. Pu lmonary hypoplasia, O ligohydramnios, Twisted face, 13 Twisted skin, Extremity defects, Renal failure (in utero). 14 Oligohydramnios Pulmonary hypoplasia Polycystic kidney disease Renal agenesis Autosomal recessive Dominance (genetics) Hypoplasia Autosomal recessive polycystic kidney disease In utero Potter sequence Kidney Fetus Autosomal dominant polycystic kidney disease Agenesis Kidney disease • 15 Autosome • 16 • 17 Bo tto m Line : • 18 Autosomal dominant polycystic kidney d isease typically manifests in adulthood with hypertension and cysts in the kidney. It is associated with • 19 other systemic pathologies, such as berry aneurysms, liver cysts, d iverticulosis, and mitral valve prolapse. • 20 Mitral valve prolapse Polycystic kidney disease Dominance (genetics) Autosomal dominant polycystic kidney disease Mitral valve Hypertension Kidney liver Kidney disease Cyst Autosome Aneurysm Pathology • 21 • 6 s 0 lock Suspend End Block Item: 14 of 27 ~ . I • M k <:] t> al ~· ~ QIO: 3520 .l. ar Previous Next lab 'lifllues Notes Calculator

1 FA17 p 573.1 2 Renal cyst disorders 3 Autosomal dominant lumerous cysts in cortex and medull a a causing bilateral enlarged kidneys ultimately destroy 4 polycystic kidney kidney parenchyma. Presents with Aank pain, hematuria, hypertension, urinary infection, 5 disease progressi,·e renal fai lure in - 50% of individuals. 6 Mutation in PKDl (85% of cases, chromosome 16) or PKD2 (15% of cases, chromosome 4). Death 7 from complications of chronic kidney disease or hypertension (caused by t renin production). Associated with berry aneurysms, mitral ,·alve prolapse, benign hepatic cysts, diverticulosis. 8 Treatment: ACE inhibitors or ARBs. 9 Autosomal recessive Cystic di lation of collecting ducts [i]. Often presents in infancy. ssociated with congenital 10 polycystic kidney hepatic fibrosis. Significant oliguric renal fai lure in utero can lead to Potter sequence. Concerns 11 disease beyond neonatal period include systemic hypertension, progressive renal insufficiency, and portal 12 hypertension from congenita I hepatic fibros is. 13 Medullary cystic Inherited disease causing tubulointerstitial fibrosis and progressive renal insufficiency with inability 14 disease to concentrate urine. Medullary cysts usually not visualized; shrun ken kidneys on ultrasound. • 15 Poor prognosis . • 16 Simple vs complex Simple cysts are filled with ultra filtrate (anechoic on ul trasound [!t). Very common and account for renal cysts majority of all renal masses. Found incidentally and typica ll y asymptomatic. • 17 Complex cysts, including those that arc septated, enhanced, or ha ve solid components on imaging • 18 req uire follow-up or removal due to risk of renal cell carcinoma . • 19 • 20 • 21 6 s 0 lock Suspend End Block Item: 14 of 27 ~ 1 • M k -<:J 1>- Jil ~· !:';-~ QIO: 3520 ..L a r Previous Next Labfli!l tues Not es Calculat o r

1 & FA17 p 56.1 & 2 Autosomal dominant Achondroplasia, autosomal dominant polycystic kidney disease, familial adenomatous polyposis, 3 diseases familial hypercholesterolemia, hereditar) hemorrhagic telangiectasia, hereditary spherOC) tosis, 4 Huntington disease, Li-Fraumeni S) ndrome, .1\ larfan syndrome, multiple endocrine neoplasias, neurofibromatosis type I (von Recklinghausen disease), neurofibromatosis type 2, tuberous 5 sclerosis, von Hippel-Lindau disease. 6 7 FA17 p 486.2 8 Aneurysms AbnormaJ dilation of an artery due to weakening of 'esse! "·all. 9 Saccular (berry) Occurs at bifurcations in the circle ofWillb. \1ost common site is junction of ACom and 10 aneurysm J\CA. Associated with ADPKD, Ehlers-Danlos S) ndrome. Other risk factors: advanced age, 11 hypertension, smokin g, race (f risk in African-Americans). 12 Usually clinically si lent until rupture (most common complication) .... subarachnoid hemorrhage ("worst headache of my life" or "thunderclap headache") .... focal neurologic deficits. Can also 13 cause symptoms via direct compression on surrounding structures by growing aneurysm. 14 • ACorn-compression .... bitempora l hemianopia (compression of optic chiasm); visual acuity • 15 deficits; rupture .... ischemia in CA distribution .... contralateral lower extremity hemiparesis, • 16 sensory deficits. • 17 MCA- rupture .... ischemia in MCA distribution .... contralateral upper extremity and facial hemiparesis, sensory deficits . . 18 PCom-comprcssion .... ipsilateral Ct Ill p:llsy .... mydriasis ("blown pupil"}; may also see • 19 ptosis, "down and out" eye . • 20 Charcot-Bouchard Common, associated with chronic hypertension; affects small vessels (eg, lenticulostriate arteries in • 21 • microaneurysm basal ganglia. thalamus). ot visible on aneiograohv. a s 8 Lock Suspend End Block Item: 15 of 27 ~ 1 • M k -<:J 1>- Jil ~· !:';-~ QIO: 3488 ..L a r Previous Next Labfli!l tues Not es Calculat o r

1 & & A 33-year-old woman presents to her physician complaining of fatigue and joint aches. On physical examination, her doctor notices 2 abnormalities in her skin (see image). Laboratory studies show antinuclear and anti-Smith antibodies. Urinalysis shows RBC casts and 2 .1 g renal biopsy is performed, which shows subendothelial deposition of immune complexes. 3 protein per 24 hours. A 4

5

6

7 8 9 10

11

12

13 14

0 15

0 16

0 17

0 18 Image courtesy of Wikimedia Commons • 19 Which of the following conditions has a m echanism similar to that underlying this patient's renal dysfunction? 0 20

• 21 : • . . a s 8 Lock Suspend End Block Image courtesy of Wikimedia Commons 11 12 Which of the following conditions has a mechanism similar to that underlying this patient's renal dysfunction?

13 : 14 A. Acute serum sickness

0 15 B. Cellulitis

0 16 c. Cervical carcinoma 0 17 D. Contact dermatitis 0 18 • 19 E . Goodpasture disease

0 20 F . Graves disease . 21 • a s 8 Lock Suspend End Block Item: 15 of 27 ~ 1 • M k -<:J 1>- Jil ~· !:';-~ QIO: 3488 ..L a r Prev ious Next Lab fli!ltues Not es Calculat or

1 & & The correct a ns wer is A. 480/o chose this. 2 The patient presents with symptoms of systemic lupus erythematosus (SLE), among which is the 3 malar, or "butterfly," rash on the face. SLE primarily affects women of childbearing age. Laboratory studies with a positive anti-Smith antibody and suggestion of lupus nephritis confirm the diagnosis of 4 SLE. Up to 85% of patients with SLE have renal involvement. Damage to glomeruli occurs as a result 5 of the subendothelial deposition of immune complexes (shown in this image). Acute serum sickness is also a type IIJ hypersensitivity reaction. It occurs when antibody-antigen complexes deposit into 6 reaction. It is often caused I tissue, activating an inflammatory response and the subsequent immune 7 by a medication acting as the hapten. The list of drugs that can cause serum sickness is vast and includes but is not limited to certain ant ibiot ics, antitoxins and antivenins, and biologics. 8 Lupus nephritis Systemic lupus erythematosus LSm Hapten Serum sicloness -ype Ill hypt. sensitivity 9 Hypersensitivity Antibody Glomerulus Inflammation Immune complex Rash Immune system Antibiotics 10 Lupus erythematosus Blood plasma Serum {blood)

11 Image copyright© 2009 Kamdar eta/.; 12 licensee Cases Network Ltd. 13 B is not co rre ct. 60fo c hose this. 14 Cellulitis is caused by a bacterial infect ion, wit h Staphylococcus and Streptococcus being the most common offenders. 15 Cellulitis Streptococcus Pathogenic bacteria Staphylococcus Infection • 16 C is not co rre ct. JO/o c hose this . • 17 Cervical carcinoma is typically the r esult of an infection by human papillomavirus types 16 or 18. • 18 Human papillomavirus Cervical cancer Neck Carcinoma Cervix

• 19 D is not corre ct . J Ofo chose this .

• 20 Contact dermatitis is a type IV delayed hypersensitivity mediated by memory T lymphocytes in the skin. The T cells are responding to an exogenous agent that directly or indirectly injures the skin . • 21 Exogeny Memory T cell • Contact dermatitis Dermatitis T cell Lymphocyte Hypersensitivity a s 8 Lock Suspend End Block Item: 15 of 27 ~ 1 • M k -<:J 1>- Jil ~· !:';-~ QIO: 3488 ..L a r Previous Next Labfli!l tues Not es Calculat o r

1 & &

2 B is n o t correct. 60fo chose this. Cellulitis is caused by a bacterial infection, with Staphylococcus and Streptococcus being the most common offenders. 3 Cellulitis Streptococcus Pathogenic bacteria Staphylococcus Infection 4 C is n o t correct. J O/o chose this. 5 Cervical carcinoma is typically the result of an infection by human papillomavirus types 16 or 18. 6 Human pa loma~ rus Cervical cancer Neck carcinoma Cerviv 7 D is not correct. 7 % chose this. I 8 Contact dermatitis is a type IV delayed hypersensitivity mediated by memory T lymphocytes in the skin. The T cells are responding to an exogenous agent that directly or indirect ly injures the skin. 9 Contact dermatitis Dermatitis T cell Lymphocyte Hypersensitivity EJCogeny Memory T eel. 10 E is not correct. 260/o chose this. 11 Goodpasture syndrome is a type II hypersensitivity with autoantibodies against type IV collagen, which is present in t he glomerular and alveolar 12 basement membranes. Collagen Type II hypersensitivity Type IV collagen Autoantibody Goodpasture syndrome Hypersensitivity Basement membrane Glomerulus Pulmonary alveolus 13 F is not correc t. 100/o c hose t his. 14 Graves disease is a type II hypersensit iv ity d isease. Its pathophysiology involves autoantibod ies against t he t hyroid-stimulating hormone 15 receptor, leading to its activation and ult imately to hyperthyroid ism. • 16 Graves' disease Thyroid-stimulating hormone Hyperthyroidism Type II hypersensitivity Autoantibody Hypersensitivity Hormone Pathophysiology • 17

• 18 Bottom Line:

• 19 Systemic lupus erythematosus and acute serum sickness are examples of type III hypersensitivity reactions (antibody- antigen immune • 20 complex disease) . System c lupus erythematosus Immune complex Serum sickness Type III hypersensitivity Hypersensitivity Blood plasma Lupus erythematosus Serum (blood} • 21 • a s 8 Lock Suspend End Block Item: 15 of 27 ~ 1 • M k -<:J 1>- Jil ~· !:';-~ QIO: 3488 ..L a r Previous Next Labfli!l tues Not es Calculat o r

1 & & FA17 p 443.1 2 Systemic lupus erythematosus 3 SYMPTOMS Classic presentation: rash, joint pain, and fe,er, Jt\ Sll OR PA l ~ : 4 most commonly in a female of reproductive Rash (malar · or discoid : ) 5 age and African-American descent. \ rthritis (nonerosi,·e) 6 Libman-Sacks Endocarditis-nonbacteri- Jil ~· !:';-~ QIO: 3488 ..L a r Previous Next Labfli!l tues Not es Calculat o r

1 & & FA17 p 564.1 2 Nephritic syndrome J\ephrltic syndrome= InA ammatory process. When glomeruli are involved, leads to hematuria 3 and RBC casts in urine. Associated with azotemia, oliguria, hypertension (due to salt retention), 4 proteinuria. 5 Acute u.,J- glomerul i enlarged and h) percellular f'J. lost frequently seen in children. Occurs 6 poststreptococca I IF' - ("starry sky") granular appearance - 2-4 weeks after group A streptococcal 7 glomerulonephritis ("lumpy-bumpy") [lJ due to lgC, lg \ I, and C3 infection of pharynx or skin. Resolves deposition along GB I and mesangium. spontaneous!). Type Ill h) persensiti' it ~ 8 E~1 -s ubepi t hel ia l immune complex (IC) reaction. 9 humps. Presents with peripheral and periorbital edema, 10 cola-colored urine, hypertension. 11 Positi,·e strep titers/serologies, ~ complement 12 b ·cls (C3) due to consumption. 13 Rapidly progressive LM and IF- crescent moon shape [j. C rescents Poor prognosis. Rapidly deteriorating renal 14 (c rescentic) consist of fibrin and plasma proteins (eg, C3b) function (days to weeks). glomerulonephritis with glomerular parietal cel ls, monocytes, 15 macro phages. • 16 Several d isease processes may result in this • 17 pallern, in particular: . 18 • Goodpasture syndrome-type II llematuria/hemoptysis. I • 19 hrpersensitivity reaction; antibodies to Treatment: emergent plasmapheresis. CBM and alveolar basement membra ne • 20 -+ linear IF • 21 • Granulomatosis with oolvanl!i itis (Weeener) PR3-ANCA/c-AJ CA. Pauci-immune (no [f!/C3 a s 8 Lock Suspend End Block Item: 16 of 27 ~ 1 • M k -<:J 1>- Jil ~· !:';-~ QIO: 5000 ..L a r Previous Next Labfli!l tues Notes Calculat o r

1 & & An 11-year-old boy is brought to the doctor with complaints of a sore throat and difficulty swallowing for the past couple of days. Physical 2 examination is significant for a temperature of 39.0°C {102.2°F), erythematous tonsils, and white patches in his throat. 3 4 If the child were tested 3-6 weeks later, which of the following would be considered a sequela of this illness?

5 : 6 A. Blood pressure of 103/ 71 mm Hg

7 B. Serum albumin level of 2.1 g/ dl 8 C. Urinalysis yielding 4+ protein 9 D. Urinalysis yielding 5-7 RBCs/ mm3 10 11 E. Urine output of 3 L per 24 hours 12 13 14 15 • 16 • 17 • 18 • 19 • 20 • 21 • a s 8 Lock Suspend End Block Item: 16 of 27 ~ 1 • M k -<:J 1>- Jil ~· !:';-~ QIO: 5000 ..L a r Prev ious Next Lab fli!ltues Not es Calculat or

& 1 Th e correct an swer is D. 720/o chose this. & 2 The child is presenting with a classic case of strep throat, caused by group A 13-hemolytic streptococci. His doctor should recognize the signs, confirm the diagnosis with a strep test, and begin the child on penicillin. Regardless of treatment, some patients may go on to develop 3 poststreptococcal glomerulonephritis (PSGN) 3- 6 weeks after pharyngitis (this can also happen after strep-related impetigo). Poststreptococcal 4 glomerulonephritis is the most common secondary cause of nephritic syndrome. It manifests with hematuria +/- red blood cell casts, hypertension, azotemia, and oliguria. Renal biopsy is not necessary for diagnosis if there is a clear clinical picture, but one can test for 5 antistreptolysin 0 antibodies, which will be positive in the majority of patients. IgA nephropathy is "synpharyngitic" with hematuria occurring 6 concurrently with pharyngitis or upper respiratory tract infection. PSGN is, as the name implies, "postpharyngitic," occurring 3-6 weeks after the group A beta-hemolytic strep pharyngitis. An important distinction to make is the time from pharyngitis to hematuria when differentiating these 7 two entities. 8 Red blood ce 0 guria Hematuria Nephritic syndrome Penicillin Impetigo Streptococcal pharyngitis Upper respiratory tract infection Azotemia IgA nephropathy 9 Renal biopsy Pharyngitis Glomerulonephritis Urinary cast Streptococcus Anti-streptolysin 0 Hypertension Biopsy Immunoglobulin A Antibody Nephritis Respiratory tract Kidney disease Kidney 10 11 A is not correct. 30/o chose this. If there is any change in blood pressure, one would expect to see hypertension with nephritic syndrome rather than hypotension. 12 Nephritic syndrome Hypotension Hypertension Blood pressure Nephron Nephritis 13 B is not correct. 60/o c hose this. 14 Hypoalbuminemia (a value <3.4 g/dL) is a symptom of nephrotic syndrome, along with proteinuria, edema, and hyperlipidemia. 15 Poststreptococcal glomerulonephritis causes a nephritic synd rome. Nephrotic syndrome Hypoalbuminemia Nephritic syndrome Proteinuria Hyperlipidemia Glomerulonephritis Edema Nephritis Nephron Streptococcus Symptom 16 C is not correct. 170/o chose this • • 17 Proteinuria is typically a symptom of nephrotic syndrome, along with hypoalbuminemia, edema, and hyperlipidemia. There can be some protein • 18 in the urine with nephritic syndrome, but it is unusual to see 4+ protein; that would be far more common for nephrotic syndrome. • 19 Poststreptococcal glomerulonephritis causes a nephritic syndrome . Nephrotic syndrome Hypoalbuminemia Nephritic syndrome Proteinuria Hyperlipidemia Glomerulonephritis Urine Edema Nephritis Nephron Protein Streptococcus • 20 Symptom • 21 • .- =- ··~- .. ------• "n'- _., ___ .. 1.. : ..... a s 8 Lock Suspend End Block Item: 16 of 27 ~ . I • M k <:] t> al ~· ~ QIO: 5000 .l. ar Previous Next lab 'lifllues Notes Calculator

1 A is no t correct . 30fo c hose t his. 2 If there is any change in blood pressu re, one would expect to see hypertension with nephritic syndrome rather than hypotension. 3 Nephritic syndrome Hypotension Hypertension Blood pressure Nephron Nephritis

4 B is no t correct . 6 0/o c hose t his. 5 Hypoalbum inemia (a value <3.4 g/ d l } is a symptom of nephrotic syndrome, along with proteinuria, edema, and hyperlipidemia. Poststreptococcal g lomerulonephritis causes a nephritic syndrome. 6 Nephrotic syndrome Hypoalbuminemia Nephritic syndrome Proteinuria Hyperlipidemia Glomerulonephritis Edema Nephritis Nephron Streptococcus Symptom 7 C is no t correct . 1 70/o c hose t his. 8 Proteinuria is typically a symptom of nephrotic syndrome, along with hypoalbuminemia, edema, and hyperlipidemia. There can be some protein 9 in the urine with nephritic syndrome, but it is unusual to see 4 + protein ; that would be far more common for nephrotic syndrome. Poststreptococcal g lomerulonephritis causes a nephritic syndrome. 10 Nephrotic syndrome Hypoalbuminemia Nephritic syndrome Proteinuria Hyperlipidemia Glomerulonephritis Urine Edema Nephritis Nephron Protein Streptococcus 11 Symptom

12 E is no t correct . 20/o c hose t his. 13 If there is any change in urine output, one would expect to see decreased urine output (oliguria) with nephritic syndrome rather than increased 14 urine output. Nephritic syndrome Oliguria Urine Nephron Nephritis 15 16 Bo tto m Line : • 17 Poststreptococcal g lomerulonephritis is the most common secondary cause of nephritic syndrome. It manifests with hematuria, hypertension, • 18 azotemia, and oliguria 3-6 weeks after an infection by g rou p A 13-hemolytic streptococci. lgA nephropathy, the most common primary cause of • 19 nephritic syndrome, is "synpharyngitic"- hematuria occu rrs concurrently with pharyng itis or upper respiratory tract infection . Oliguria Hematuria Nephritic syndrome Azotemia IgA nephropathy Upper respiratory tract infection Pharyngitis Glomerulonephritis Hypertension Streptococcus • 20 Kidney disease Immunoglobulin A Nephritis Respiratory tract Infection • 21 6 s 0 lock Suspend End Block Item: 16 of 27 ~ 1 • M k -<:J 1>- Jil ~· !:';-~ QIO: 5000 ..L a r Previous Next Labfli!l tues Notes Calculat o r

1 & & FA17 p 564.1 2 Nephritic syndrome J\ephrltic syndrome= InAammatory process. When glomeruli are inmlved, leads to hematuria 3 and RBC casts in urine. Associated with azotemia, oliguria, hypertension (due to salt retention), 4 proteinuria. 5 Acute LM -glomeruli enlarged and h) percellular f'J. lost frequently seen in children. Occurs 6 post streptococcal IF'-("starry sky") granular appearance - 2-4 weeks after group A streptococcal 7 glomerulonephritis ("lumpy-bumpy") [lJ due to lgC, lg\ I, and C3 infection of pharynx or skin. Resolves 8 deposition along CB I and mesangium. spontaneous!). Type Ill h) persensiti' it~ E~1-subep i thelial immune complex (IC) reaction. 9 humps. Presents with peripheral and periorbital edema, 10 cola-colored urine, hypertension. 11 Positi,·e strep titers/serologies, l complement 12 b ·cls (C3) due to consumption. 13 Rapidly progressive LM and IF- crescent moon shape [!ll. Crescents Poor prognosis. Rapidly deteriorating renal 14 (crescentic) consist of fibrin and plasma proteins (eg, C3b) function (days to weeks). glomerulonephrit is with glomerular parietal cells, monocytes, 15 macro phages. 16 Several d isease processes may result in this • 17 pattern, in particular: . 18 • Goodpasture syndrome-type II llematuria/hemoptysis. • 19 hypersensitivity reaction; antibodies to Treatment: emergent plasmapheresis. • 20 CBM and alveolar basement membrane -+ linear IF • 21 • r.r!lnnlnrn!ltnnaiiti< ( \ ,\IPaP n P r) PR ~-A I r.A Ir-A 1 r.A P:mri-irnrnnnP fnn l air.~ a s 8 Lock Suspend End Block Item: 16 of 27 ~ 1 • Ma rk -<:J 1>- Jil ~· !:';-~ QIO: 5000 ..L Previous Next Labfli!l tues Notes Calculat o r

1 FA17 p 132.4 2 Streptococcus Gram<±> cocci in chains rJ. Group A strep J• N ES (major criteria for acute rheumatic 3 pyogenes (group A cause: fever): 4 streptococci) • Pyogenic-pharyngitis, cellulitis, impetigo Joints-polyarthritis 5 ("honey-crusted" lesions), er)sipelas • -carditis 6 • Toxigenic-scarlet fc,·er, toxic shock-like :\odules (subcutaneous) syndrome. necrotizing fasciitis tr}thema marginatum 7 ~ Immunologic-rheumatic fever, Srdenham chorea 8 glomerulonephritis Pharyngitis can result in rheumatic "phe,er" 9 Bacitracin sensitive, ~ - hemolytic, pyrrolidon} I and glomerulonephritis. 10 arylamidase (PYR) $. Hyaluronic acid capsule Impetigo usually precedes glomerulonephrit is. 11 inhibits phagocytosis. ntibodies to Yl protein Scarlet fever-blanching, sandpaper-like body enhance host defenses against S pyogenes but rash, strawberry tongue, and circumoral 12 can give rise to rheumatic fe,·er. pallor in the setting of group A streptococcal 13 ASO titer or anti-DNasc B antibodies indicate pharyngitis (erythrogenic toxin$). 14 recent S pyogenes infection. 15 16 FA17 p 183.2 • 17 Penicillin G, V Penicillin G (I and li\ 1 form), penicillin V (oral). Prototype ~-lactam antibiotics. . 18 MECHANISM 0-Ala-D-Ala structural analog. Bind penici llin-binding proteins (transpeptidases). • 19 Block transpeptidase cross- l in~ing of peptidogl)can in cell wall. • 20 Activate autolytic enzymes . • 21 Cli NIC AL USE J\ lostly used for gram<±> organisms (S pneumonioe, S pyogenes, Actinomyces). Also used for gram e • a s 8 Lock Suspend End Block Item: 17 of 27 ~ 1 • M k -<:J 1>- Jil ~· !:';-~ QIO: 1795 ..L a r Previous Next Labfli!llues Notes Calculat o r

1 & & An 18-year-old woman presents to the em ergency department because of a 3-day history of worsening fever and a painful right foot. Physical i•A] 2 examination reveals the patient has erythema around a puncture wound in her foot. She is admitted to the hospital and given penicillin and A fever and reduce the erythema, pain, and swelling. Ten days later the patient develops new onset of 3 acetaminophen, which break her weakness, fatigue, diffuse rash, and fever . Significant laboratory findings include a blood urea nitrogen of 45 mg/dl and a creatinine of 3 .1 mg/dl. 4 Urinalysis shows WBC casts, eosinophils, and a fractional excretion of sodium of 2 .5%. 5 6 The new onset of illness is most likely attributable to which of these potential sources?

7 : 8 A. Acetaminophen

9 B. Penicillin 10 C. Poststreptococcal glomerulonephrit is 11 D. Septic shock 12 13 E. Urolithiasis 14 15 16 • 17 • 18 • 19 • 20 • 21 • a s 8 Lock Suspend End Block Item: 17 of 27 ~ 1 • M k -<:J 1>- Jil ~· !:';-~ QIO: 1795 ..L a r Prev ious Next Labfli!llues Notes Calculat or

1 & & Th e correct an swer is B. 61 Ofo ch ose this. 2 The patient's urinalysis indicates that her renal failure is intrarenal in origin {shown by her fractional 3 excretion of sodium of between 2% and 4%) and that interstitial nephritis is occurring (as seen by 4 the WBC casts, similar to those in the image). Allergic interstitial nephritis is an inflammatory process that results in infiltration of the interstitium of the kidney with polymorphonuclear leukocytes and 5 lymphocytes. The cause of interstitial nephritis is usually an allergic reaction to medications, but 6 infections and immunologic disorders occasionally precipitate the disorder. Medications commonly associated with allergic reaction include penicillins, cephalosporins, and sulfonamides. Clinical 7 findings include fever, rash, and eosinophilia. The most likely culprit in this case is the penicillin she 8 was given for her foot infection. Other rare but serious reactions associated with penicillin include Image copyright© Dustri-Verlag Dr. K. hemolytic anemia and anaphylaxis. Feistle 9 Anaphyla is Penici lin Urinalysis Eosinophilia Interstitial nephritis Hemolytic anemia Anemia White blood cell 10 Sulfonamide (medicine) Hemolysis Granulocyte Kidney Cephalosporin Rash Nephritis Sodium Lymphocyte Sulfonamide Immune system Fever Allergy Inflammation Excretion Infection Interstitial fluid 11 Immunology 12 A i s not correct. 120/o chose this. 13 Although there is suggestive evidence that chronic use of acetaminophen may cause long-term nephropathy, this medication is rarely acutely 14 toxic to the kidneys in the absence of an overdose and concurrent liver damage. There is no information in the vignette to suggest that this patient took more than the recommended dose of acetaminophen. Furthermore, when acute renal failure does occur secondary to acetaminophen 15 use, urinalysis generally reveals granular and epithelial cell casts. The most common causes of allergy-mediated acute renal failure are ~-lactam 16 antibiotics and NSAIDs; as such, the antibiotic therapy is much more likely to be the instigating factor. kidney injury Antibiotics Urinalysis Paracetamol Epithelium Nonsteroidal anti-inflammatory drug Kidney disease Liver Hepatotoxicity Nephrotoxicity Kidney 17 Acute Drug overdose Pharmaceutical drug Toxicity • 18 C is n ot correct. 140/o ch ose this • • 19 Poststreptococcal glomerulonephritis is a type of immune-complex glomerulonephritis. This condition usually presents about 10 days after • 20 pharyngitis or 2 weeks after skin infection and is usually seen in children. Most patients test positive for antistreptolysin 0, which can be helpful in • 21 making the diagnosis. Although poststreptococcal glomerulonephritis is possible, given her skin infection, it classically manifests with a nephritic • a s 8 Lock Suspend End Block Item: 17 of 27 ~ . I • M k <:] t> al ~· ~ QIO: 1795 .l. ar Previous Next lab 'lifllues Notes Calculator

1 C is no t correct . 140/o c hose t his. 2 Poststreptococcal g lomerulonephritis is a type of immune-complex g lomerulonephritis. This condition usually presents about 10 days after 3 pharyngitis or 2 weeks after skin infection and is usually seen in children. Most patients test positive for antistreptolysin 0 , which can be helpful in making the d iagnosis. Although poststreptococcal g lomerulonephritis is possible, g iven her skin infection, it classically manifests with a nephritic 4 picture. Eosinophils and rash would not be seen. 5 Pharyngitis Anti-streptolysin 0 Glomerulonephritis Streptococcus Rash Eosinophil Nephritis Skin infection Infection Nephritic syndrome

6 0 is no t correct . 90fo c hose t his. 7 Although it is theoretically possible that the infection in this patient's puncture wound entered the bloodstream and led to septic shock, causing hypoperfusion of the kidney and prerenal acute renal failure, this is unlikely. First, the patient's infection responded well to antibiotics. Second, 8 the pattern of acute renal failure suggests a parenchymal rather than prerenal orig in . In prerenal failure, typically the ratio of BUN to creatinine is 9 >20 and the fractional excretion of sodium is <%1, ind icating volume depletion with normally functioning g lomeruli and tubules. In addition, prerenal failure is not associated with eosinophilia. In general, septic shock can be considered in a hypotensive patient who is not responding to 10 fluid resuscitation . 11 BUN-to-creatinine ratio Acute kidney injury Septic shock Creatinine Glomerulus (kidney) Glomerulus Blood urea nitrogen Hypotension Kidney Hypovolemia 12 Shock (circulatory) Sodium Fluid replacement Fractional sodium excretion Antibiotics Infection Parenchyma Eosinophilia Circulatory system 13 E is no t correct . 4 0fo c hose t his. 14 Urolithiasis (kidney stones) will manifest with postrenal failure and a fractional excretion of sodium of > 4% . The complications of urolithiasis are hydronephrosis due to obstruction and interstitial nephritis if the stone is secondary to a bacterial infection . On clinical presentation, some 15 patients present with excruciating flank pain. 16 Hydronephrosis Interstitial nephritis Urolithiasis Nephritis Sodium Fractional sodium excretion Abdominal pain Infection 17 • 18 Botto m Line : • 19 Interstitial nephritis can resu lt from an allergic reaction to penicillins (particularly methicillin and nafcillin), cephalosporins, and sulfonamides. One would expect to see WBC casts and eosinophils in the urine as well as a FE Na between 2% and 4% . • 20 Interstitial nephritis Nafcillin Sulfonamide (medicine) Nephritis Penicillin Cephalosporin Eosinophil Allergy Meticillin Urine • 21 6 s 0 lock Suspend End Block Item: 17 of 27 ~ 1 • M k -<:J 1>- Jil ~· !:';-~ QIO: 1795 ..L a r Previous Next Labfli!llues Notes Calculat o r

1 & & FA17p572.1 2 Acute interstitial Acute interstitial renal inAammation. P} uri a Associated with fever, rash, hematuria, and 3 nephritis (classically eosinophils) and a.totemia costovertebral angle tenderness, but can be 4 (tubuloi nterstitial occurring after administration of drugs that asymptomatic. 5 nephritis) act as haptens, inducing h) pcrscnsitivity (eg, Remember these P's: 6 diuretics, penicillin dcri\·ati\CS, proton pump Pee (diuretics) inhibitors. sulfonamidcs, rifarnpin, 'SAl Ds). Pain-free (r\SAlDs) 7 Less commonly mar be zoto other processes Penicillins and cephalosporins 8 such as systemic infections (eg, lll)Coplasma) or Proton pump inhibitors 9 autoimmune diseases (eg, Sjogren S} ndrome, RifamPin 10 SLE, sarcoidosis). 11

12 FA17 p 571 .2 13 Acute kidney injury Acute kidney injury is defined as an abrupt decline in renal function as measured by t creatinine 14 (acute renal failure) and t BUI or by oligu ri<1/anuria. 15 Prerenal azotemia Due to l RBF (eg, hypotension) - l CFR. a+Jl-1 20 and BU reta ined by kidn ey in an attempt to 16 conserve volume - t BU /creatin ine ratio (BU I is reabso rbed, creatinine is not) and l FE a· 17 Intrinsic renal failure Generally clue to acute tubular necrosis or ischemia/toxins; less commonly due to acute . 18 glomerulonephritis (cg, RPC , hemolytic uremic syndrome) or acute interstitial nephritis . • 19 In AT , patchy necrosis - debris obstructing tubule and Auid backAow across necrotic tubule - l GFR. Urine has epithelial/granular casts. BU reabsorption is impaired - l BU /creatinine • 20 ratio and t FE'Ja· • 21 • a s 8 Lock Suspend End Block Item: 17 of 27 ~ 1 • M k -<:J 1>- Jil ~· !:';-~ QIO: 1795 ..L a r Previous Next Labfli!llues Notes Calculat o r

1 & &

2 FA17 p 564.1 3 Nephritic syndrome J\ephrltic syndrome= Inflammatory process. When glomeruli are inml\'ed, leads to hematuria 4 and RBC casts in urine. Associated with azotemia, oliguria, hypertension (due to salt retention), proteinuria. 5 Acute LM-glomeruli enlarged and h) percellular ~lost frequently seen in children. Occurs 6 post streptococcal IF' - ("starry sky") granular appearance - 2-4 weeks after group A streptococcal 7 glomerulonephritis ("lumpy-bumpy") D due to lgC, lg\ I, and C3 infection of pharynx or skin. Resolves 8 deposition along CB I and mesangium. spontaneous!). Type 111 h) persensiti' it ~ 9 E~-subepithelial immune complex (IC) reaction. 10 humps. Presents \\'ith peripheral and periorbital edema, cola-colored urine, hypertension. 11 Positi,·e strep titers/serologies, L complement 12 b ·els (C3) due to consumption. 13 Rapidly progressive LM and IF- crescent moon shape ~- Crescents Poor prognosis. Rapidly deteriorating renal 14 (crescentic) consist of fibrin and plasma proteins (eg, C3b) function (days to weeks). 15 glomerulonephrit is with glomerular parietal cells, monocytes, 16 macro phages. 17 Several d isease processes may result in th is pallern, in particular: . 18 • Goodpasture syndro me-type 11 llemaluria/hemoptysis. • 19 hypersensiti\'ity reaction; antibodies to Treatment: emergent plasmapheresis. • 20 CB.t\1 and alveolar basement membrane

• 21 -+ linear IF • a s 8 Lock Suspend End Block Item: 18 of 27 ~ 1 • M k -<:J 1>- Jil ~· !:';-~ QIO: 5 051 ..L a r Previous Next Labfli!l tues Not es Calculat o r

1 & & A 9-year-old boy presents to his pediatrician for his annual check-up. He is accompanied by his mother, whose only concern is a 4.5-kg (10- 2 lb) weight gain over the past 2 months. On examination his lungs are clear and his pulse is regular. However, he has 2+ pitting edema in his Urinalysis demonstrates significant proteinuria, and routine blood work reveals hyperlipidemia. 3 lower extremities bilaterally. 4 What is the most appropriate next step in management? 5

6 A. Follow up in 3 months 7 8 B. Immediate treatment with a statin

9 C. Immediate treatment with cyclophosphamide 10 D. Immediate treatment with steroids 11 E. Renal biopsy 12

13 14

15

16

17 • 18 • 19

• 20

• 21 • a s 8 Lock Suspend End Block Item: 18 of 27 ~ . I • M k <:] t> al ~· ~ QIO: 5051 .l. ar Previous Next lab 'lifllues Notes Calculator

1 The correct answer is 0. 660/o chose t his. 2 This patient is presenting with signs of nephrotic syndrome characterized by proteinuria, hypoalbuminemia, hyperlipidemia, and edema. In the ped iatric population, nephrotic syndrome is most commonly caused by m inimal change d isease (MCD}. MCD is definitively d iagnosed on renal 3 biopsy, which shows no obvious g lomerular lesion by light m icroscopy and is negative for deposits by immunofluorescent m icroscopy. Electron 4 m icroscopy, however, consistently demonstrates an effacement of the foot process supporting the epithelial podocytes with weakening of slit­ pore membranes. The pathophysiology of this lesion is uncertain, although most agree there is a circulating cytokine (perhaps related to a T­ 5 lymphocyte response) that alters g lomerular capillary charge and podocyte integ rity. MCD presents with the abrupt onset of edema and weight 6 gain accompanied by significant proteinuria on urinalysis. Empirical treatment with steroids is ind icated based on clinical suspicion of MCD; renal 7 biopsy is not necessary before treatment is initiated. Although up to 30% of children haves spontaneous remission, all children should be treated with steroids. On ly those who are nonresponders to steroids should undergo biopsy. The long-term prognosis is generally favorable, with a good 8 response to steroids in most ped iatric patients. 9 Podocyte Nephrotic syndrome Hypoalbuminemia Cytokine Minimal change disease Hyperlipidemia Proteinuria Urinalysis Edema Immunofluorescence Biopsy Renal biopsy T cell Steroid Electron microscope Glomerulus Optical microscope Epithelium Remission (medicine) Prognosis Capillary Kidney Microscopy 10 light microscopy Glomerulus (kidney) lesion 11 A is not correct. J Ofo chose t his. 12 This child has typical features of MCD. Because untreated nephrotic state is associated with several complications, all patients with MCD should be 13 treated immed iately. Hence, expectant management at this time is inappropriate. 14 Mini CD MCD Productions Municipal Corporation of Delhi Maxi single 15 B is not correct. 4 0fo chose t his. Although patients with MCD often have associated hyperlipidemia, treatment with a statin is not ind icated at this time. The appropriate initial 16 management of a patient with suspected MCD is steroids. The hyperlipidemia observed in this patient should resolve once the MCD resolves with 17 appropriate therapy. Statin Hyperlipidemia Steroid 18 • 19 C is not correct . 5 0fo chose t his . Steroids are the first-line treatment for MCD. Cyclophosphamide is an immunosu ppressant that is used as second -line treatment in those patients • 20 who do not respond to steroids . • 21 Cyclophosphamide Immunosuppression Steroid 6 s 0 lock Suspend End Block Item: 18 of 27 ~ . I • M k <:] t> al ~· ~ QIO: 5051 .l. ar Previous Next lab 'lifllues Notes Calculator

1 light microscopy Glomerulus (kidney) lesion

2 A is not correct. JOfo c hose t his. 3 This child has typical features of MCD. Because untreated nephrotic state is associated with several complications, all patients with MCD should be 4 treated immediately. Hence, expectant management at this time is inappropriate. Mini CD MCD Productions Municipal Corporation of Delhi Maxi single 5 B is not correct. 4 0fo c hose t his. 6 Although patients with MCD often have associated hyperlipidemia, treatment with a statin is not ind icated at this time. The appropriate initial 7 management of a patient with suspected MCD is steroids. The hyperlipidemia observed in this patient should resolve once the MCD resolves with 8 appropriate therapy. Statin Hyperlipidemia Steroid 9 C is not correct. 50fo c hose t his. 10 Steroids are the first-line treatment for MCD. Cyclophosphamide is an immunosuppressant that is used as second -line treatment in those patients 11 who do not respond to steroids. 12 Cyclophosphamide Immunosuppression Steroid 13 E is not correct. 180/o c hose t his. Although MCD is definitively d iagnosed by renal biopsy, histologic evidence of the d isease is not necessary prior to the initiation of treatment with 14 steroids in children. Renal biopsy is ind icated on ly in patients who fail an initial trial of steroid treatment; it would not be the most appropriate 15 next step in management for this patient. Renal biopsy Biopsy Steroid Histology Kidney 16 17 18 Bottom Line : • 19 Minimal change d isease presents with the abrupt onset of edema and proteinuria. Empiric treatment with steroids is ind icated based on clinical suspicion . • 20 Proteinuria Minimal change disease Edema Steroid • 21 6 s 0 lock Suspend End Block Item: 18 of 27 ~ 1 • M k -<:J 1>- Jil ~· !:';-~ QIO: 5 051 ..L a r Previous Next Labfli!l tues Not es Calculat o r

& & 1 FA17 p 566.1 2 Nephrotic syndrome f'\eph rO ti c syndrome-massi\ e p rO teinuri a (> 3.5 g/day) with hypoalbuminemia, resulting 3 edema, hyperl ipidemia. Frothy urine " ith fa tly casts. Due to podocyte damage disrupting 4 glomerular filtration charge barrier. \llay be 1° (eg, d irect sclerosis of podocrtes) or 2° (systemic 5 process [eg, diabetes] secondarily damages podocytes). Associated with hypercoagulable state (eg, thromboembolism) due to antithrombin ( T ) Ill loss in urine and t risk of infection (due to loss of 6 immunoglobulins in urine and soft tissue compromise by edema). 7 Se,·ere nephritic syndrome may present with nephrotic S}lldrome features (nephritic-nephrotic 8 syndrome) if damage to C B I is se,·crc enough to damage charge barrier. 9 Minimal change L\11 - normal glomeruli (lipid may be seen in \!l ost common cause of nephrotic S)lldrome 10 disease (lipoid PCT cells). in children. Often 1° (idiopathic) and ma) be 11 nephrosis) IF 8 . triggered by recent infection, immunization, EM-effacement of foot processes []. immune stimulus. Rarely, may be to 12 zo lymphoma (eg, cytokine-mediated damage). I0 13 disease has excellent response to corticosteroids. 14 Focal segmental LM-segmental sclerosis and hya linosis : . M o~ t common cause of nephrotic sy ndrome in 15 glomerulosclerosis IF - often 8 , but may be (f) for nonspeci fi e focal African Americans and Hispanics. Can be ]0 16 deposits of Ig r, C3, C l. (idiopathic) or zo to other conditions (eg, H IV 17 E If -effacement of foot process similar to infecti on, sickle cell disease, heroin abuse, 18 minimal change disease. massive obesity, interferon treatment, chronic kidney disease due to congenitalmalfo m1ati ons) . • 19 0 ) disease has inconsistent response to steroids . • 20 :\1ay progress to chronic renal disease. • 21 • Membranous L.\11- diffuse caoillarv and G B\11 thickeninP \llost common cause of 1° ncohroti c S\'nclromc a s 8 Lock Suspend End Block Item: 18 of 27 ~ 1 • M k -<:J 1>- Jil ~· !:';-~ QIO: 5 051 ..L a r Previous Next Labfli!l tues Not es Calculat o r

1 & & FA17 p 564.1 2 Nephritic syndrome I\ephrltic syndrome= Inflammatory process. When glomeruli are inmh-ed, leads to hematuria 3 and RBC casts in urine. Associated with a:Lolemia, oliguria, hypertension (due to salt retention), 4 proteinuria. 5 Acute L~ l -glomerul i enlarged and h) percellular f'J. lost frequently seen in children. Occurs 6 poststreptococcal IF' - (''starry sky") granular appearance - 2-4 weeks after group A streptococcal 7 glomerulonephritis ("lumpy-bumpy") [lJ due to lgC, lg\ I, and C3 infection of pharynx or skin. Resolves 8 deposition along GB I and mesangium. spontaneous!). Type Il l h) persensiti' it ~ 9 E~1-subepithelial immune complex (IC) reaction. humps. Presents with peripheral and periorbital edema, 10 cola-colored urine, hypertension. 11 Positi,·e strep titers/serologies, ~ complement 12 b ·cls (C3) due to consumption. 13 Rapidly progressive LM and IF- crescent moon s hape~- C rescents Poor prognosis. Rapidly deteriorating renal 14 (crescentic) consist of fibrin and plasma proteins (eg, C3b) function (days to weeks). 15 glomerulonephritis with glomerular parietal cells, monocytes, macro phages. 16 Several d isease processes may result in th is 17 pallern, in particular: 18 • Goodpasture syndrome-type II llematuria/hemoptysis. • 19 hrpersensitivity reaction; antibodies to Treatment: emergent plasmapheresis. • 20 CB.t\1 and alveolar basement membrane -+ linear IF • 21 • • ~'"'---···---•- -=- ...:.! ... --···--..::.:... 1\11... --·--- \ 001 \ '-11'"' \ 1- A '-'1'"' \ D-··-: :--··-- ' ··- 1-11'"'1 a s 8 Lock Suspend End Block Item: 19 of 27 ~ 1 • M k -<:J 1>- Jil ~· !:';-~ QIO: 4117 ..L a r Previous Next Labfli!llues Not es Calculat o r

1 & & A renal biopsy is performed on a 25-year-old man. Light microscopy reveals large, hypercellular glomeruli with mesangial proliferation and 2 thickening and splitting of the basement membrane. Immunofluorescent staining shows granular deposits of C3 and IgG. Electron subendothelial deposits. 3 microscopy shows 4 Which of the following is the most likely diagnosis? 5 6 A. Diabetic glomerulosclerosis 7 8 B. Goodpasture syndrome

9 C. Membranoproliferative glomerulonephritis 10 D. Membranous nephropathy 11 E. Minimal change disease 12 13 14 15 16 17 18 . 19 • 20 . 21 • a s 8 Lock Suspend End Block Item: 19 of 27 ~ . I • M k <:] t> al ~· ~ QIO: 4117 .l. ar Previous Next Lab'lifllues Notes Calculator

1 • The correct answer is C. 740fo chose this. 2 This patient most likely has membranoproliferative g lomerulonephritis {MPGN} type 1. The d isease is classified as a nephritic syndrome but often 3 presents with nephrotic syndrome as welL Light m icroscopy shows mesangial proliferation and thickening and splitting of the basement 4 membrane. Immunofluorescence shows g ranular subendothelial deposits of C3 and lgG. Type 2 MPGN is caused by C3 nephritic factor, which stabilizes C3 convertase lead ing to dense deposit d isease. 5 Membranoproliferative glomerulonephritis Nephrotic syndrome Nephritic syndrome Immunofluorescence Basement membrane Glomerulonephritis C3-convertase

6 Nephron Nephritis Mesangium light microscopy Optical microscope

7 A is not correct. 30fo chose this. 8 Diabetic g lomerulosclerosis is characterized by d ramatic thickening of the capillary basement membrane. Its immunofluorescence pattern is nonspecific. 9 Diabetic nephropathy Basement membrane Immunofluorescence Capillary Diabetes mellitus Glomerulosclerosis 10 B is not correct. 100/o chose this. 11 Goodpasture syndrome usually presents with nephritic syndrome and has linear immunofluorescent deposits. 12 Nephritic syndrome Immunofluorescence Goodpasture syndrome Nephritis Nephron

13 0 is not correct. 11 Ofo chose this. 14 Membranous nephropathy, also known as membranous g lomerulonephritis, does not usually have g lomerular hypercellularity and mesangial proliferation. Light microscopy usually reveals d iffuse thickening of the capillary waiL 15 Membranous glomerulonephritis Glomerulonephritis Capillary light microscopy Kidney disease Glomerulus (kidney) Mesangium Glomerulus Optical microscope 16 E is not correct. 20/o chose this. 17 Minimal change d isease usually is negative for immunofluorescence, and g lomeruli appear normal under light m icroscopy. Retraction of 18 podocytes is seen under electron m icroscopy. This d isease is the most common cause of nephrotic syndrome in children but rarely affects adults. Nephrotic syndrome Glomerulus (kidney) Minimal change disease Immunofluorescence Glomerulus Electron microscope Podocyte Optical microscope 19 light microscopy Microscopy

0 20

0 21 .,. n_...... __. __ ,: ___ _ 6 s 0 lock Suspend End Block Item: 19 of 27 ~ . I • M k <:] t> al ~· ~ QIO: 4117 .l. ar Previous Next Lab'lifllues Notes Calculator

1 • Membranoproliferative glomerulonephritis Nephrotic syndrome Nephritic syndrome Immunofluorescence Basement membrane Glomerulonephritis C3-convertase

2 Nephron Nephritis Mesangium light microscopy Optical microscope

3 A is no t correct . 30fo c hose t his. Diabetic g lomerulosclerosis is characterized by d ramatic thickening of the capillary basement membrane. Its immunofluorescence pattern is 4 nonspecific. 5 Diabetic nephropathy Basement membrane Immunofluorescence Capillary Diabetes mellitus Glomerulosclerosis

6 B is no t correct . 100/o c hose t his. 7 Goodpasture syndrome usually presents with nephritic syndrome and has linear immunofluorescent deposits. Nephritic syndrome Immunofluorescence Goodpasture syndrome Nephritis Nephron 8 9 0 is no t correct . 11 Ofo c hose t his. Membranous nephropathy, also known as membranous g lomerulonephritis, does not usually have g lomerular hypercellularity and mesangial 10 proliferation . Light microscopy usually reveals d iffuse thickening of the capillary waiL 11 Membranous glomerulonephritis Glomerulonephritis Capillary light microscopy Kidney disease Glomerulus (kidney) Mesangium Glomerulus Optical microscope

12 E is no t correct . 20/o c hose t his. 13 Minimal change d isease usually is negative for immunofluorescence, and g lomeruli appear normal under light microscopy. Retraction of podocytes is seen under electron microscopy. This d isease is the most common cause of nephrotic syndrome in children but rarely affects adults. 14 Nephrotic syndrome Glomerulus (kidney) Minimal change disease Immunofluorescence Glomerulus Electron microscope Podocyte Optical microscope 15 light microscopy Microscopy 16 17 Bo tto m Line : 18 Membranoproliferative g lomerulonephritis {MPGN} can cause either nephritic or nephrotic syndrome. In type 1 MPGN, immunoflorescence 19 shows subendothelial deposits of C3 and lgG. Type 2 MPGN is caused by C3 nephritic factor, which stabilizes C3 convertase lead ing to dense deposit d isease. 0 20 Membranoproliferative glomerulonephritis Nephrotic syndrome Glomerulonephritis Nephron C3-convertase Nephritis Nephritic syndrome 0 21 • 6 s 0 lock Suspend End Block Item: 19 of 27 ~ 1 • M k -<:J 1>- Jil ~· !:';-~ QIO: 4117 ..L a r Previous Next Labfli!llues Not es Calculat o r

1 & & FA17p564.1 2 3 Nephritic syndrome 1\ephrltic syndrome= Inflammatory process. When glomeruli are inm lved, leads to hematuria and RBC casts in urine. Associated with a~o t emia, oliguria, hypertension (due to salt retention), 4 proteinuria. 5 Acute L~vl-g l ome rul i enlarged and h) percellular fJ. ~los t frequently seen in children. Occurs 6 post streptococca I IF' - (.. s tarry sky") granular appearance - 2 -4 weeks after group A streptococcal 7 glomerulonephritis ("lumpy-bumpy") g due to lgC, lg\ I, and C3 infection of pharynx or skin. Resolves 8 deposition along CB I and mesangium. spontaneous!). Type Ill h) persensiti' it ~ 9 E~1 -subepi t heli a l immune complex (IC) reaction. ps. ripheral and periorbital edema, 10 hum Presents with pe cola-colored urine, hypertension. 11 Positi,·e strep titers/serologies, ! complement 12 b ·cls (C3) due to consumption. 13 Rapidly progressive LM and IF- crescent moon shape Bl. C rescents Poor prognosis. Rapidly deteriorating renal 14 (crescentic) consist of fibrin and plasma proteins (eg, C3b) function (days to weeks). 15 glomerulonephrit is with glomerular parietal cel ls, monocytes, 16 macro phages. Several d isease processes may result in this 17 pallern, in particular: 18 • Goodpasture syndrome-type II llematuria/hemoptysis. 19 hypersensiti,·ity reaction; antibodies to Treatment: emergent plasmapheresis. • 20 CB~I and alveolar basement membmne . 21 -+ linear IF \ ,,..., • ...... ,, , nn "'' • ' , ,....., • 1 ... ' , ,...... _ • n . a s 8 Lock Suspend End Block Item: 19 of 27 ~ 1 • M k -<:J 1>- Jil ~· !:';-~ QIO: 4117 ..L a r Previous Next Labfli!llues Not es Calculat o r

1 & & FA17 p 566.1 2 Nephrotic syndrome 1'\ephrO tic syndrome-massi\e prO teinuria (> 3.5 g/day) with hypoalbuminemia, resulting 3 edema, hyperlipidemia. Frothy urine" ith fatly casts. Due to podocyte damage disrupting 4 glomerular filtration charge barrier. \!Ia)' be 1° (eg, direct sclerosis of podocytes) or 2° (systemic 5 process [eg, diabetes] secondarily damages podocytes). Associated with hypercoagulable sta te (cg, 6 thromboembolism) due to antithrombin ( T ) Ill loss in urine and t risk of infection (due to loss of ). 7 immunoglobulins in urine and soft tissue compromise by edema Se,·ere nephritic syndrome may present with nephrotic S}lldrome features {nephritic-nephrotic 8 syndrome) if damage to C Bt\ I is severe enough to damage charge barrier. 9 Minimal change LM-normal glomeruli (lipid may be seen in \!lost common cause of nephrotic sp1drome 10 disease (lipoid PCT cells). in children. Often 1° (idiopathic) and ma) be 11 nephrosis) we. triggered by recent infection, immunization, 12 EM -effacement of foot processes Fl. immune stimulus. Rarely, may be zoto 0 13 lymphoma (eg, cytokine-mediated damage). I disease has excellent response to corticosteroids. 14 Focal segmental LM-segmental sclerosis and hyalinosis : . Mo~t common cause of nephrotic syndrome in 15 glomerulosclerosis IF - often 8, but may be (f) for nonspeci fie focal African Americans and Hispanics. Can be ]0 16 deposits of Ig r, C3, C l. (idiopathic) or zo to other conditions (eg, I-I IV 17 EM -effacement offoot process simila r to infection, sickle cell disease, heroin abuse, 18 minimal change disease. massive obesity, interferon treatment, chronic 19 kidney disease due to congenital malfom1ations). 0 • 20 ) disease has inconsistent response to steroids . ~ lay progress to chronic renal disease . • 21 •- T , s •·rr ... 1 """"' ••• • 1 r•o • . . ... - - - ' . a s 8 Lock Suspend End Block Item: 19 of 27 ~ 1 • M k -<:J 1>- Jil ~· !:';-~ QIO: 4117 ..L a r Previous Next Labfli!llues Not es Calculat o r

1 & & FA17 p 566.1 2 Nephrotic syndrome 1'\ephrO tic syndrome-massi'e prO teinuria (> 3.5 glday) with hypoalbuminemia, resulting 3 edema, hyperlipidemia. Frothy urine" ith fatly casts. Due to podocyte damage disrupting 4 glomerular filtration charge barrier. \!Ia)' be 1° (eg, direct sclerosis of podocytes) or 2° (systemic 5 process [eg, diabetes] secondarily damages podocytes). Associated " ·ith hypercoagulable state (eg, 6 thromboembolism) due to antithrombin ( T ) Ill loss in urine and t risk of infection (due to loss of 7 immunoglobulins in urine and soft tissue compromise by edema). Se,·ere nephritic syndrome may present with nephrotic S}lldrome features {nephritic-nephrotic 8 syndrome) if damage to CBl\1 is severe enough to damage charge barrier. 9 Minimal change LM-normal glomeruli (lipid may be seen in \!lost common cause of nephrotic sp1drome 10 disease (lipoid PCT cells). in children. Often 1° (idiopathic) and ma) be 11 nephrosis) we. triggered by recent infection, immunization, 12 EM -effacement of foot processes Fl. immune stimulus. Rarely, may be 2° to 0 13 lymphoma (eg, cytokine-mediated damage). I 14 disease has excellent response to corticosteroids. 15 Focal segmental LM-segmental sclerosis and hyalinosis : . Mo~t common cause of nephrotic syndrome in glomerulosclerosis IF - often 8, but may be® for nonspeci fie focal African Americans and Hispanics. Can be 1° 16 deposits of Ig r, C3, C l. (idiopathic) or zo to other conditions (eg, I-I IV 17 E If-effacement offoot process simila r to infection, sickle cell disease, heroin abuse, 18 minimal change disease. massive obesity, interferon treatment, chronic I 19 kidney disease due to congenitalmalfom1ations). 0 • 20 ) disease has inconsistent response to steroids . ~ lay progress to chronic renal disease . • 21 • a s 8 Lock Suspend End Block Item: 20 of 27 ~ 1 • M k -<:J 1>- Jil ~· !:';-~ QIO: 4120 ..L a r Previous Next Labfli!llues Not es Calculat o r

1 & & An 8-year-old boy is brought to the emergency department by his parents because his face appears swollen. The parents state that their son 2 has been feeling weak during the last 2 d ays and has been complaining of a headache and that this morning his face looked puffy. The boy surgical history, but the parents do say that their son had an episode of pharyngitis 3 weeks ago. Vital 3 has no significant past medical or signs reveal a heart rate of 78/min, respiratory rate of 16/min, and a blood pressure of 140/90 mm Hg. Physical examination reveals swelling 4 around the eyes but no rash. 5 6 Given the patient's condition and past medical history, which one of the following would be present on urinalysis?

7 : 8 A. Crystal casts

9 B. Epithelial cell casts 10 C. Fatty casts 11 D. Hyaline casts 12 13 E. RBC casts 14 15 16 17 18 19 • 20 • 21 • a s 8 Lock Suspend End Block Item: 20 of 27 ~ 1 • M k -<:J 1>- Jil ~· !:';-~ QIO: 4120 ..L a r Previous Next Labfli!llues Not es Calculat o r

1 & & Th e correct an swer is E. 700/o chose this. 2 This patient is suffering from poststreptococcal glomerulonephritis Greatly increased glomerular cellularoty 3 (PSGN), shown in the image; RBC casts are indicative of this disease. Symptoms of PSGN usually develop 1- 3 weeks after acute infection with 4 specific nephritogenic strains of group A ~ - hemolytic streptococcus. :;.~~#Nil·:..~ Loss ol 5 PSGN can develop after streptococcal pharyngitis or skin infection, Bowman space it is more commonly seen after a skin infection. Symptoms of 6 though PSGN include facial puffiness, hypertension, and dark urine. Urinalysis 7 reveals hematuria and proteinuria; microscopic analysis shows RBC should be 8 casts and, in rare instances, WBC casts. IgA nephropathy d istinguished from PSGN as they can manifest very similarly. Time 9 course is imperative as IgA nephropathy manifests days after an 10 infection, whereas PSGN occurs weeks afterwards. 11 Treatment for PSGN is supportive therapy and not antibiotics. Most patients experience a complete recovery with supportive therapy alone. 12 It is important to note that unlike PSGN, rheumatic fever may be Image courtesy of Y. Chu, MD 13 prevented with antibiotic therapy after a group A ~-hemolytic streptococcal infection. 14 Streptococcal pharyngitis Rheumatic fever Hematuria Proteinuria IgA nephropathy Urinalysis Pharyngitis Glomerulonephritis Antibiotics Hypertension 15 Kidney disease Streptococcus Urine Immunoglobulin A Red blood cell Fever Urinary cast Infection 16 A i s not correct. JO/o chose this. 17 Crystal casts-mixing of c•·ystallized urinary solutes like oxalates, urates, and sulfonamides with other materials- are not found in 18 poststreptococcal glomerulonephritis. These mixed casts are found incidentally and are clinically insignificant. Uric acid Oxalate Oxalic acid Sulfonamide 19 Sulfonamide (medicine) Glomerulonephritis Streptococcus 20 B is n ot correct . 7 0fo ch ose this. These casts can be seen in acute tubular necrosis (ATN) and toxic . 21 Desquamated epithelial cells of the tubule lining may form epithelial casts. • innPc::tinn (pn mPrr11rv rtiPthviPnP nlvrnl nr <:;=!llir-vl~tp.) Althn11nh c::nmP vir11C::PC:: fpn rvtnmPnA~Invirllc;) rAin fnrm thPc:P r:.c::tc:: thP n:.tiPnt'c:: a s 8 Lock Suspend End Block Item: 20 of 27 ~ . I • M k <:] t> al ~· ~ QIO: 4120 .l. ar Previous Next lab 'lifllues Notes Calculator • 1 Crystal casts- m ix ing of crystallized urinary solutes like oxalates, urates, and sulfonamides with other materials- are not found in 2 poststreptococcal g lomerulonephritis. These m ixed casts are found incidentally and are clinically insignificant. Sulfonamide (medicine) Glomerulonephritis Streptococcus Uric acid Oxalate Oxalic acid Sulfonamide 3 B is not correct. JOfo c hose t his. 4 Desquamated epithelial cells of the tubule lining may form epithelial casts. These casts can be seen in acute tubular necrosis (ATN} and toxic 5 ingestion (eg, mercury, d iethylene g lycol, or salicylate). Although some viruses (eg, cytomegalovirus) can form these casts, the patient's 6 symptoms are more ind icative of pharyngitis caused by Streptococcus pyogenes. Streptococcus pyogenes Cytomegalovirus Acute tubular necrosis Diethylene glycol Pharyngitis Epithelium Streptococcus Salicylic acid Mercury (element) Necrosis 7 Virus Desquamation 8 C is not correct. 1 20/o c hose t his. 9 Breakdown of lipid-rich epithelial cells may form fatty casts. These casts are found in various d isorders, includ ing nephrotic syndrome and 10 d iabetic or lupus nephropathy. Minimal change d isease is common in children after an infection, but one should g ive g reater consideration to poststreptococcal g lomerulonephritis for a patient recovering from pharyngitis. 11 Nephrotic syndrome Minimal change disease Pharyngitis Glomerulonephritis Epithelium Kidney disease lupus erythematosus Systemic lupus erythematosus 12 Diabetes mellitus Streptococcus

13 0 is not correct. SO/o c hose t his. 14 Fo rmed from the tubular epihtelial cells of ind ividual , hyaline casts are the most common type of cast. Conditions that may lead to their 15 formation include low urine flow, concentrated urine, or an acidic environment. Consequently, they may be seen in normal ind ividuals during vigorous exercise or when dehydrated. 16 Hyaline Urine Nephron Dehydration 17 18 Bottom Line : 19 RBC casts ind icate bleed ing into the nephrons and this occu rs to a significant degree in g lomerulonephritis. Consequently, RBC casts are 20 considered a hallmark, but not pathognomonic finding for g lomerulonephritis. • 21 Urinary cast Pathognomonic Nephron Glomerulonephritis Red blood cell • 6 s 0 lock Suspend End Block Item: 20 of 27 ~ 1 • M k -<:J 1>- Jil ~· !:';-~ QIO: 4120 ..L a r Previous Next Labfli!llues Not es Calculat o r

1 & & FA17 p 564.1 2 Nephritic syndrome J\ephrltic syndrome= InAammatory process. When glomeruli are involved, leads to hematuria 3 and RBC casts in urine. Associated with azotemia, oliguria, hypertension (due to salt retention), 4 proteinuria. 5 Acute L~v l -glomerul i enlarged and h) percellular f'J. lost frequently seen in children. Occurs 6 poststreptococcal IF- ("starry sky") granular appearance - 2-4 weeks after group A streptococcal 7 glomerulonephritis ("lumpy-bumpy") [lJ due to lgC, lg\ I, and C3 infection of pharynx or skin. Resolves deposition along GB I and mesangium. Type Ill h) persensiti' it~ 8 spontaneous!). E~1-subep i thelial immune complex (IC) reaction. 9 humps. Presents with peripheral and periorbital edema, 10 cola-colored urine, hypertension. 11 Positi,·e strep titers/serologies, l complement 12 b ·cls (C3) due to consumption. 13 Rapidly progressive LM and IF- crescent moon shape ~- Crescents Poor prognosis. Rapidly deteriorating renal 14 (crescentic) consist of fibrin and plasma proteins (eg, C3b) function (days to weeks). glomerulonephrit is with glomerular parietal cells, monocytes, 15 macro phages. 16 Several d isease processes may result in this 17 pallern, in particular: 18 • Goodpasture syndro me -type II llematuria/hemoptysis. 19 hrpersensitivity reaction; antibodies to Treatment: emergent plasmapheresis. CBt\1 and alveolar basement membrane 20 -+ linear IF • 21 • Granulomatosis with nolv:ml7iiti~ (\MePener) PR 3-ANCA/c-A• CA. Pauci-immune (no (p/C3 a s 8 Lock Suspend End Block Item: 20 of 27 ~ 1 • M k -<:J 1>- Jil ~· !:';-~ QIO: 4120 ..L a r Previous Next Labfli!ltues Not es Calculat o r

& & 1 FA17 p 563.2 2 Glomerular diseases 3 4 5 Nephritic syndrome-due to GBM desruption Nephrotic syndrome-podocyte disruption Hypertension. "' BUN and creatinine. oliguria. -+ charge barrier impaired Massive proteinuria 6 hematuria. RBC casts in urine Proteinuria often (> 3 5 g/day) with hypoalbuminemia. 7 in the subnephrotic range(< 3.5 g/day) but in hyperbpidemea. edema May be 1" (eg. direct severe cases may be in nephrotic range podocyte damage) or 2" (podocyte damage 8 from systemiC process (eg. diabetesll o Acute poststreptococcal glomerulonephntis 9 o Rapedly progressive glomerulonephritis o Focal segmental glomerulosclerosis (1" or 2") o lgA nephropathy (Berger disease) • M1nemal change d•sease (1' or 2") 10 o Alport syndrome • Membranous nephropathy (1' or 2") o Membranoproliferative glomerulonephritis • AmyloidoSIS (2, 11 • Diabetic glomerulonephropathy (2') 12 13 14 Nephritic-nephrotic syndrome-severe nephritic syndrome 15 with profound GBM damage that damages the glomerular filtration charge barner -+ nephrOtiC-range proteinuria(> 3.5 16 g/day) and concomitant features of nephrotec syndrome. Can 17 occur with any form of nephritiCsyndrome. but IS most commonly seen with 18 • D1Huse prohferateve glomerulonephntis 19 • Membranoprohferalive glomerulonephnlls 20 • 21 • a s 8 Lock Suspend End Block Item: 21 of 27 ~ 1 • M k -<:J 1>- Jil ~· !:';-~ QIO: 4115 ..L a r Previous Next Labfli!llues Not es Calculat o r

1 & & A 27- year-old HIV-positive man presents to the emergency department with swollen legs. He states that his urine has been very foamy. 2 Urinalysis shows massive proteinuria. Treatment with corticosteroids is initiated but is not successful. Immunofluorescence of the kidney glomeruli. 3 shows deposition of IgM in the affected 4 Which of the following is most likely to be seen on kidney biopsy? 5 6 A. Apple-green birefringence with Congo red stain under polarized light 7 8 B. Extensive glomerular crescent formation

9 C. IgA deposition in the glomerular mesangium 10 D. Immune complex deposition in subepithelial space 11 E. Segmental sclerosis and hyalinosis 12 13 14 15 16 17 18 19 20 . 21 • a s 8 Lock Suspend End Block Item: 21 of 27 ~ 1 • M k -<:J 1>- Jil ~· !:';-~ QIO: 4115 ..L a r Prev ious Next Labfli!ltues Not es Calculat or

1 & & Th e c o rrect a n sw er i s E. 5 2 0/o ch ose this. 2 Focal sclerosis and hyaline deposits are characteristic of focal segmental glomerulosclerosis (FSGS). FSGS presents as nephrotic syndrome 3 (massive proteinuria, hypoalbuminemia, generalized edema, hyperlipidemia, and lipiduria) and often occurs in patients with HIV infection. It is 4 also characterized by either a lack of deposits on immunofluorescence or nonspecific deposition of IgM in sclerotic glomeruli. Similar to minimal change disease, effacement of podocyte foot processes may also be observed. However, the distinguishing feature of FSGS is the hyaline 5 deposition. 6 Podocyte lllep otic syndrome Hypoalbuminemia Focal segmental glomerulosclerosis Proteinuna Minimal change disease Immunofluorescence Hyper! pidemla Glomerulosclerosis Glomerulus (kidney) 7 Imm 1nog 1bu ·- M Hya "ne Edema HIV Glomerulus 8 A is not correct . s olo chose this. Renal amyloidosis presents as nephrot ic syndrome and is the most common manifestation of systemic amyloidosis. Amyloid deposits are visible 9 under light microscopy using Congo red stain and polarized light. Renal amyloidosis can be caused by many types of protein deposits, including 10 Ig light chains in systemic amyloidosis, Ig heavy chains, amyloid A in secondary amyloidosis, 13 amyloid in Alzheimer disease and Down syndrome, and 132-microglobulin in dialysis-related amyloidosis. 11 Congo red Down syndrome Nephrotic syndrome Amyloid Amyloidosis Protein Alzheimer·s disease Microscopy Immunoglobulin light chain 12 Immunoglobulin heavy chain Optical microscope Polarization (waves) 13 B is not co rrec t. 140/o c hose this. 14 Rapidly progressive glomerulonephritis is characterized by the presence of crescents in most g lomeruli. The description of this patient indicates 15 that many glomeruli are spared, making t his synd rome unlikely. Glomerulonephritis Rapidly progressive glomerulonephritis Glomerulus Glomerulus (kidney) Glomerulus (olfaction) 16 C is not co rrect. 4 0fo c hose this. 17 IgA nephropathy (Berger disease) is characterized by the presence of prominent IgA, not IgM, deposits in the m esangial region of the 18 glomerulus. As it causes the nephritic syndrome, mild proteinuria usually is present, and hematuria usually is found . 19 IgA nephropathy Hematuria Nephritic syndrome Proteinuria Immunoglobulin M Immunoglobulin A Kidney disease Glomerulus Nephron Nephritis Glomerulus (kidney) Mesangium 20 21 D is n o t correct . 220/o ch ose this. • a s 8 Lock Suspend End Block Item: 21 of 27 ~ . I • M k <:] t> al ~· ~ QIO: 4115 .l. ar Previous Next lab 'lifllues Notes Calculator • 1 Renal amyloidosis presents as nephrotic syndrome and is the most common manifestation of systemic amyloidosis. Amyloid deposits are visible 2 under light microscopy using Congo red stain and polarized light. Renal amyloidosis can be caused by many types of protein deposits, includ ing lg light chains in systemic amyloidosis, lg heavy chains, amyloid A in secondary amyloidosis, 13 amyloid in Alzheimer d isease and Down 3 syndrome, and 132-microglobulin in d ialysis-related amyloidosis. 4 Congo red Down syndrome Nephrotic syndrome Amyloid Amyloidosis Protein Alzheimer's disease Microscopy Immunoglobulin light chain 5 Immunoglobulin heavy chain Optical microscope Polarization (waves) 6 B is not correct. 140/o chose this. Ra pid ly prog ressive g lomerulonephritis is characterized by the presence of crescents in most g lomeruli. The description of this patient ind icates 7 that many g lomeruli are spared, making this syndrome unlikely. 8 Glomerulonephritis Rapidly progressive glomerulonephritis Glomerulus Glomerulus (kidney) Glomerulus (olfaction)

9 C is not correct. 4 0fo chose this. 10 lgA nephropathy (Berger d isease) is characterized by the presence of prominent lgA, not lgM, deposits in the mesangial reg ion of the g lomerulus. As it causes the nephritic syndrome, mild proteinuria usually is present, and hematuria usually is found. 11 IgA nephropathy Hematuria Nephritic syndrome Proteinuria Immunoglobulin M Immunoglobulin A Kidney disease Glomerulus Nephron Nephritis 12 Glomerulus (kidney) Mesangium

13 0 is not correct. 220/o chose this. 14 Although membranous nephropathy was previously the most common cause of nephrotic syndrome, focal g lomerular sclerosis has since 15 surpassed it in frequency. It is characterized by d iffuse thickening of capillary walls and deposits of lgG in the g lomerular basement membrane. Thus it is an unlikely the cause of this patient's symptoms. 16 Nephrotic syndrome Glomerular basement membrane Basement membrane Kidney disease Immunoglobulin G Glomerulus Capillary Glomerulus (kidney) 17 18 Bottom Line: 19 FSGS occu rs often in HIV-positive patients and presents as nephrotic syndrome. It shows large deposits of hyaline in some of the g lomeruli. 20 Nephrotic syndrome Glomerulus Glomerulus (kidney) Hyaline Focal segmental glomerulosclerosis HIV Glomerulus (olfaction) 21 • 6 s 0 lock Suspend End Block Item: 21 of 27 ~ 1 • M k -<:J 1>- Jil ~· !:';-~ QIO: 4115 ..L a r Previous Next Labfli!llues Not es Calculat o r

1 & & FA17 p 566.1 2 Nephrotic syndrome t\ephrO tic syndrome-massi'e prO teinuria (> 3.5 g/day) with hypoalbuminemia, resulting 3 edema, hyperlipidemia. Frothy urine " ith fatty casts. Due to podocyte damage disrupti ng 4 glomerular filtration charge barrier. \llay be 1° (eg, direct sclerosis of podocytes) or 2° (systemic 5 process [eg, diabetes] secondarily damages podocytes). Associated with hypercoagulable stale (eg, 6 thromboembolism) due to antithrombin ( T ) Ill loss in urine and t risk of infection (due to loss of immunoglobulins in urine and soft tissue compromise by edema). 7 Se,·ere nephritic syndrome may present with nephrotic S}lldrome features (nephritic-nephrotic 8 syndrome) if damage to C B I is severe enough to damage charge barrier. 9 Minimal change LM-normal glomeruli (lipid may be seen in \!lost common cause of nephrotic S)11drome 10 disease (lipoid PCT cells). in children. Often 1° (idiopathic) and ma) be 11 nephrosis) IF 8 . triggered by recent infection, immunization, 12 EM-effacement of foot processes[]. immune stimulus. Rarely, may be zoto lymphoma (eg, cytokine-medialed damage). I0 13 disease has excellent response to corticosteroids. 14 Focal segmental LM-segmental sclerosis and hyalinosis : . Mo~l common cause of nephrotic syndrome in 15 glomerulosclerosis IF - often 8, but may be <±> for nonspeci fie focal African Americans and Hispanics. Can be ]0 16 deposits of Ig r, C3, C l. (idiopathic) or zo to other conditions (eg, H IV 17 EM -effacement of foot process similar lo infection, sickle cell disease, heroin abuse, 18 minimal change disease. massive obesity, interferon treatment, chronic 19 kidney disease due to congenital malformations). 0 ) disease has inconsistent response to steroids. 20 May progress to chronic renal disease. 21 • ••--a...... - ...... ,. \tf .... ,._._ ,.....,__..._...... - ...... ,...... C 10 ...... ~_ ... .._; ...... ,.,...... ,t_...... __ .... a s 8 Lock Suspend End Block Item: 22 of 27 ~ 1 • M k -<:J 1>- Jil ~· !:';-~ QIO: 1796 ..L a r Previous Next Labfli!llues Not es Calculat o r

A A 7 A 67-year-old man with a hist01·y of mild hypertension has a severe heart attack while walking to his car. When he arrives at the emergency 8 department he is pale, cold, and diaphoretic. On physical examination he is tachycardic and hypotensive, and his ECG shows ST-segment elevations. He is treated with morphine, oxygen, and aspirin and is sent to the cardiac catheterization laboratory. The next day he has low 9 urine output, his blood urea nitrogen level is 35 mg/dl, his creatinine level is 1.3 mg/dl, and his blood pressure is 85/55 mm Hg. FeNa is 0.5%. 10 Urinalysis reveals few hyaline casts. 11 likely cause of his low urine output? 12 Which of the following is the most

13 : A. A blockage of the ureters or urethra 14

15 B. Acute tubular necrosis

16 C. Interstitial nephritis 17 D. Low urine output is normal in t imes of stress 18 19 E. Poor perfusion of the kidneys

20

21 • 22

• 23 • 24

• 25

• 26

• 27 • a s 8 Lock Suspend End Block Item: 22 of 27 ~ . I • M k <:] t> al ~· ~ QIO: 1796 .l. ar Previous Next lab 'lifllues Notes Calculator

7 The correct answer is E. 650/o chose this. This patient has experienced cardiogenic shock, as evidenced by his hypotension (systolic blood pressure <90 mm Hg } after a myocard ial 8 infarction . This has led to decreased renal perfusion, which has, in turn, led to prerenal acute kidney injury. When the g lomerular filtration rate 9 d rops, there is an increase in sodium and water reabsorption in the proximal tubule. This leads to an increase in tubular urea concentration, 10 which favors increased reabsorption of urea. This will raise the BUN (remember, this is urea in the blood} and, therefore, the BUN :Cr ratio will rise. Keep in mind that the most common cause of acute tubular necrosis is prerenal d isease. Therefore in any volume-depleted patient it is 11 important to know (a) when the initial insult occu rred and (b) what the BUN :Cr ratio is. Finally, one can also d ifferentiate prerenal from intrinsic 12 renal failure on the basis of the fractional excretion of sodium {FeNA}. A FeN A < 1% is ind icative of prerenal causes, whereas a FeN A > 2% ind icates intrinsic renal failure. 13 Cardiogenic shock Hypotension Blood pressure Systole Heart Kidney 14 A is not correct. 30fo chose this. 15 Blockage of the ureters is a postrenal cause of renal failure. It would manifest with pain on urination, lower abdominal d iscomfort and bloating, 16 and is unlikely in the setting of cardiogenic shock. Cardiogenic shock Urination Bloating Heart Kidney Ureter 17 B is not correct. 200/o chose this. 18 This patient's BUN :Cr ratio of >20:1 and FeNa <1% ind icates that this is a prerenal process, not an intrarenal one. Acute tubular necrosis (ATN} 19 is an intrarenal process, which can be caused by exposure to nephrotoxic d rugs (eg, aminoglycosides, rad iocontrast media, amphotericin B, 20 cisplatin), endogenous nephrotoxins (eg, myoglobin), or prolonged ischemia. The laboratory findings are very important for d ifferentiating ATN from other renal pathologies causing acute kidney inj ury. ATN would manifest with renal failure in the setting of a BUN :Cr ratio of 10- 15:1 and 21 FeN a > 2% with muddy brown g ranular and epithelial cell casts, as well as free renal tubular epithelial cells in the urine. 22 Epithelium Urine Kidney • 23 C is not correct. JOfo chose this . • 24 Although interstitial nephritis can occu r from the use of nonsteroidal anti-inflammatory d rugs, urinalysis would classically show WBCs in the urine and a FeNa >2%. Signs and symptoms of interstitial nephritis include rash and fever. It is possible to also have eosinophilia, , and • 25 elevated lg E levels in certain cases of interstitial nephritis. • 26 Interstitial nephritis Nephritis Urinalysis Eosinophilia Immunoglobulin E Urine Nonsteroidal anti-inflammatory drug Anti-inflammatory Rash Interstitial fluid Fever • 27 0 is not correct. 50fo chose this . 6 s 0 lock Suspend End Block Item: 22 of 27 ~ . I • M k <:] t> al ~· ~ QIO: 1796 .l. ar Previous Next lab 'lifllues Notes Calculator • 7 and is unlikely in the setting of cardiogenic shock. Cardiogenic shock Urination Bloating Heart Kidney Ureter 8 B is not correct. 200/o c hose t his. 9 This patient's BUN :Cr ratio of >20:1 and FeNa <1% ind icates that this is a prerenal process, not an intrarenal one. Acute tubular necrosis (ATN} 10 is an intrarenal process, which can be caused by exposure to nephrotoxic d rugs (eg, aminoglycosides, rad iocontrast media, amphotericin B, 11 cisplatin), endogenous nephrotoxins (eg, myoglobin), or prolonged ischemia. The laboratory findings are very important for d ifferentiating ATN from other renal pathologies causing acute kidney inj ury. ATN would manifest with renal failure in the setting of a BUN :Cr ratio of 10-15: 1 and 12 FeN a > 2% with muddy brown g ranular and epithelial cell casts, as well as free renal tubular epithelial cells in the urine. 13 Epithelium Urine Kidney 14 C is not correct. JOfo c hose t his. 15 Although interstitial nephritis can occu r from the use of nonsteroidal anti-inflammatory d rugs, urinalysis would classically show WBCs in the urine and a FeNa >2%. Signs and symptoms of interstitial nephritis include rash and fever. It is possible to also have eosinophilia, eosinophiluria, and 16 elevated lgE levels in certain cases of interstitial nephritis. 17 Interstitial nephritis Nephritis Urinalysis Eosinophilia Immunoglobulin E Urine Nonsteroidal anti-inflammatory drug Anti-inflammatory Rash Interstitial fluid Fever 18 0 is not correct. 50/o c hose t his. 19 Urine output is controlled mainly by two factors: the hydration state of the body and the level of kidney function. Therefore, low urine output is seen on ly in the setting of dehydration or kidney dysfunction . Stress by itself will not cause low urine output unless it is coupled with dehydration 20 or an acute renal d isease process. 21 Kidney disease Dehydration Urine Oliguria Kidney Renal function Management of dehydration 22 • 23 Bottom Li ne : • 24 Poo r perfusion of the kidneys can resu lt in prerenal acute renal failure. Laboratory values suggestive of prerenal etiology include blood urea • 25 nitrogen/ creatinine (BUN :Cr} ratio of >20:1 and FeNa < 1% . Urea Creatinine Acute kidney injury Perfusion Kidney Etiology • 26 • 27 ~ 6 s 0 lock Suspend End Block Item: 22 of 27 ~ . I • M k <:] t> al ~· ~ QIO: 1796 .l. ar Previous Next lab 'lifllues Notes Calculator • • 7 FA17p299.1 8 Shock Inadequate orga n perfus ion and delive ry of nutrients necessary for normal ti ss ue and cellular 9 fu nction. Initially may be reversible but life threatening if not trea ted promptly. 10 PCWP SVR 11 CAUSED BY SKIN (PRELOAD) co (AFTERLOAD) TREATMENT 12 Hypovolemic Hemorrhage, dehydration, Cold, u ! t lV fluids 13 burns clammy 14 Cardiogenic Acute Ml, HF, valvular lnotropes, diuresis dysfun ction, arrhythmia 15 Cold, Obstructive Cardiac tamponade, t orl u t Reii eve obstruction 16 clammy pulmonary embolism, 17 tension pneumothora x 18 Distributive Sepsis, anaphylaxis Wa rm ! f u l V Au ids, pressors 19 CNS injury Dry ! ! u 20 21 FA17 p 571 .2 22 • 23 Acute kidney injury Acute kidney injury is defin ed as an abrupt decli ne in renal fun ction as measured by f creatinine (acute renal failure) and f BU N or by oliguria/anuria. • 24 Prerenal azotemia Due to l RBF (eg, hypotension) -+ l GFR. Na+JJ-J 0 and BU retained by kidney in an attempt to • 25 2 conserve volume .... t BU /creatin ine rati o (BU 1 is reabsorbed, creatinine is not) and l FENa· • 26 Intrinsic renal failure Generally due to acute tubular necrosis or ischemia/toxins; less commonly due to acute • 27 • • ,...),.. ._ ...... , )..,. .,.,..,... ),. ..: .. : ,./...,_ 000 1\.. J l..,..., • ..,, ..,.. ) •• l.: ...... , ... : ..., ...... ,) ...... _...,..\ -. .. "'""""L"" : • .., 1- ,. ....,. 1:1: .., 1 ...... t ...: a.: ~o 6 s 0 lock Suspend End Block Item: 22 of 27 ~ . I • M k <:] t> al ~· ~ QIO: 1796 .l. ar Previous Next lab 'lifllues Notes Calculator • 7 FA17 p296.1 8 Myocardial complications 9 Cardiac arrhythmia Occurs within the first few days after MI. Important cause of death before reaching the hospital 10 and within the first 24 hours post-MI. 11 Postinfarction Occurs l-3 days after MI. Friction rub. 12 fibrinous pericarditis 13 Papillary muscle Occurs 2-7 days after Ml. Posteromedial papillary muscle rupture f'J t risk due to single blood 14 rupture supply from posterior descending artery. Can result in severe mitral regurgitation. 15 Interventricular septal Occurs 3-5 days after MI. l\ilacrophagc-mediated degradation - VSD. 16 rupture 17 Ventricular Occurs 3-14 days after MI. Contained free wall rupture rn; l CO, risk of arrhythmia, embolus 18 pseudoaneurysm from mural thrombus. 19 formation 20 Ventricular free wall Occurs 5-14 days after Ml. Free wa ll rupture - cardiac tamponade. L hypertrophy and rupture previous Ml protect against free wall rupture. 21 22 True ventricular Occurs 2 weeks to several months after MI. Outward bulge with contraction ("dyskinesia"), aneurysm associated with fibrosis. • 23 Dressler syndrome Occurs several weeks after Ml. Au toimmune phenomenon resulting in fibr inous pericarditis. • 24 • 25 LV failure and Can occur 2° to LV infarction, VSD, free wall rupture, papillary muscle rupture with mitral pulmonary edema regurgitation. • 26 • 27 ~ 6 s 0 lock Suspend End Block Item: 23 of 27 ~ 1 • M k -<:J 1>- Jil ~· !:';-~ QIO: 1783 ..L a r Previous Next Labfli!llues Notes Calculat o r

A A 7 A 28-year-old woman who received no prenatal care gives birth at 37 weeks' gestation. Her medical history is significant for essential 8 hypertension, which was well controlled with benazepril throughout her pregnancy. There are no known genetic disorders in her family, but she is unsure who the father of the baby is. Unfortunately, the fetus is stillborn. The obstetrician notes a number of anomalies, including a 9 Aattened face, large and low-set ears, and clubbed feet. He explains to the patient that the baby did not develop normally because there was not 10 enough amniotic Auid. 11 oligohydramnios seen in this patient? 12 What is the most likely cause of the

13 : A. Anencephaly 14 15 B. Drug effect

16 C. Duodenal atresia 17 D. Maternal diabetes 18 19 E. Trisomy 18 20 21 22 • 23 • 24 • 25 • 26 • 27 • a s 8 Lock Suspend End Block Item: 23 of 27 ~ . , . M k <:] t> al ~· ~ QIO: 1783 .l. ar Previous Next lab 'lifllues Notes Calculator • 7 The correct a nswer is B. 54 0/o c hose t his. 8 Angiotensin-converting enzyme inhibitors {ACE inhibitors) such as benazepril are contraindicated in pregnancy as they may lead to renal agenesis by interfering with normal upregulation of the ren in -angiotensin system during development. Amniotic fluid volume depends on the 9 balance between production via fetal urination and elim ination via fetal swallowing. Conditions that reduce urine output, such as renal agenesis, 10 resu lt in oligohydramnios. This leads to a sequence of events called Potter sequence or syndrome, in which fetuses develop flattened facies and limb deformities due to physical compression, as well as pulmonary hypoplasia that can resu lt in fetal demise, as seen in this case. Maternal 11 hypertension is best managed with labetalol or hydralazine. 12 labetalol Benazepril Oligohydramnios Hydralazine Renin-angiotensin system Pulmonary hypoplasia Angiotensin-converting enzyme Amniotic fluid Enzyme 13 Renal agenesis ACE inhibitor Hypertension Potter sequence Kidney Pregnancy Enzyme inhibitor Urine Hypoplasia 14 A is no t correct . 70/o c hose t his. 15 Anencephaly is a common neural tube defect. It is defined as partial or complete absence of the fetal brain or cranial vault. This condition is lethal, and most fetuses are stillborn. It can be d iagnosed using ultrasound to identify the neural tube defect. Po lyhyd ramnios also is commonly 16 found in anencephaly, because the fetus is unable to swallow amniotic fluid. 17 Polyhydramnios Neural tube defect Anencephaly Amniotic fluid Fetus Medical ultrasound Ultrasound Stillbirth Neural tube Skull Brain

18 C is no t correct . 11 Ofo c hose t his. 19 Since amniotic fluid is eliminated by fetal swallowing, duodenal atresia would lead to polyhydramnios, or excessive amniotic fluid volumes. Other fetal malformations, such as esophageal atresia and anencephaly, that impair swallowing can also cause polyhydramnios. 20 Polyhydramnios Anencephaly Esophageal atresia Duodenal atresia Amniotic fluid Atresia Duodenum Congenital disorder 21 0 is no t correct . 11 Ofo c hose t his. 22 Maternal d iabetes has been associated with polyhydramnios. The pathophysiology behind this phenomenon is that increased serum g lucose in 23 the mother is transmitted to the fetus. Just as in any other d iabetic patient, increased blood g lucose acts as an osmotic d iuretic, causing an increased production of urine. Any cause of increased urination in the fetus will lead to polyhydramnios . • 24 Polyhydramnios Diuretic Polyuria Diabetes mellitus Glucose Urine Blood plasma Blood sugar Gestational diabetes Urination Fetus Pathophysiology Serum (blood) • 25 E is no t correct . 1 70/o c hose t his . • 26 Trisomy 18, or Edwards syndrome, can also manifest with facial deformities (eg, low-set ears) and limb deformities (eg, clenched hands}. It is, • 27 however, not associated with a flattened facies. Instead, the pathognomonic features of this syndrome are micrognathia and a prominent occiput, ~ 6 s 0 lock Suspend End Block Item: 23 of 27 ~ . , . M k <:] t> al ~· ~ QIO: 1783 .l. ar Previous Next lab 'lifllues Notes Calculator • Anencephaly is a common neural tube defect. It is defined as partial or complete absence of the fetal brain or cranial vault. This condition is • 7 lethal, and most fetuses are stillborn. It can be d iagnosed using ultrasound to identify the neural tube defect. Po lyhyd ramnios also is commonly 8 found in anencephaly, because the fetus is unable to swallow amniotic fluid. Polyhydramnios Neural tube defect Anencephaly Amniotic fluid Fetus Medical ultrasound Ultrasound Stillbirth Neural tube Skull Brain 9

10 C is no t correct . 11 Ofo c hose t his. Since amniotic fluid is elim inated by fetal swallowing, duodenal atresia would lead to polyhydramnios, or excessive amniotic fluid volumes. Other 11 fetal malformations, such as esophageal atresia and anencephaly, that impair swallowing can also cause polyhydramnios. 12 Polyhydramnios Anencephaly Esophageal atresia Duodenal atresia Amniotic fluid Atresia Duodenum Congenital disorder

13 0 is no t correct . 11 Ofo c hose t his. 14 Maternal d iabetes has been associated with polyhydramnios. The pathophysiology behind this phenomenon is that increased serum g lucose in the mother is transmitted to the fetus. Just as in any other d iabetic patient, increased blood g lucose acts as an osmotic d iuretic, causing an 15 increased production of urine. Any cause of increased urination in the fetus will lead to polyhydramnios. 16 Polyhydramnios Diuretic Polyuria Diabetes mellitus Glucose Urine Blood plasma Blood sugar Gestational diabetes Urination Fetus Pathophysiology Serum (blood)

17 E is no t correct . 1 70/o c hose t his. 18 Trisomy 18, or Edwards syndrome, can also manifest with facial deformities (eg, low-set ears) and limb deformities (eg, clenched hands}. It is, however, not associated with a flattened facies. Instead, the pathognomonic features of this syndrome are micrognathia and a prominent occiput, 19 along with the aforementioned anomalies in the ears and limbs. Edwards syndrome is the second most common trisomy resu lting in live birth 20 after Down syndrome and has not been associated with oligohydramnios in utero. Oligohydramnios Down syndrome Micrognathism Occiput Edwards syndrome Trisomy In utero 21

22

23 Botto m Line : • 24 Potter sequence refers to a set of events secondary to reduced fetal urine output in which oligohydramnios causes flattened facies, limb deformities, and pulmonary hypoplasia. For this reason, ACE inhibitors, which cause bilateral renal agenesis, are contraindicated in pregnancy . • 25 Conversely, polyhydramnios occu rs when the fetus either cannot swallow the amniotic fluid or produces too much urine. • 26 Polyhydramnios Oligohydramnios Pulmonary hypoplasia Potter sequence Amniotic fluid Fetus Renal agenesis ACE inhibitor Urine Hypoplasia Pregnancy Kidney • 27 ~ 6 s 0 lock Suspend End Block Item: 23 of 27 ~ 1 • M k -<:J 1>- Jil ~· !:';-~ QIO: 1783 ..L a r Previous Next Labfli!llues Notes Calculat o r

A A 7 FA17 p 548.2 8 Potter sequence Oligohydramnios - compression of Babies who can't "Pee" in utero dc,·clop Potter 9 (syndrome) developing fetus - limb deformities, sequence. 10 facial anomalies {eg, low-set ears and P

• 26 FA17 p 577.1 • 27 Anqiotensin- Caotooril. enalaoril. lisinooril. ramioril. a s 8 Lock Suspend End Block Item: 24 of 27 ~ 1 • M k -<:J 1>- Jil ~· !:';-~ QIO: 1797 ..L a r Previous Next Labfli!llues Not es Calculat o r

A A 7 A 49-year-old alcoholic consumes antifreeze and is subsequently found unconscious on the sidewalk. He is taken to a nearby hospital, where 8 he is found to be in Aorid renal failure. Despite emergency dialysis, he ultimately goes into torsade de pointes and dies. Postmortem renal biopsy shows diffuse tubular necrosis. 9 Url no Urine Na' Serum I 10 o smolarity FEN. (mEq/L) BUN/ Cr (mOsml kg) 11 A 800 10 -r- < 1% 30 12 B 200 50 > 2% I 10 c I 275 60 > 1% _! 40 13 0 I 550 20 < 1% 30 E I 275 20 <1% I 40 14 ' 15 Which of the laboratory findings in the table above would you expect to see in the patient on admission? 16 : 17 A 18 B 19 20 c 21 D 22 E 23 . 24 • 25 . 26 . 27 • a s 8 Lock Suspend End Block Item: 24 of 27 ~ . I • M k <:] t> al ~· ~ QIO: 1797 .l. ar Previous Next lab 'lifllues Notes Calculator • 7 The correct a nswer is B. 560/o c hose t his. 8 The im portant point of this question is that this patient d ied in renal failure, which is ind icated by the d iffuse tubular necrosis seen on autopsy. 9 Tubular necrosis ind icates acute renal failure, which can be secondary to ischemia, crush inj uries (myoglobinuria), or a variety of toxins (ethylene g lycol, lead ing to metabolic anion gap acidosis). If the remaining organ systems are supported, ind ividuals with ATN (acute tubular necrosis) can 10 recover in 2-3 weeks with supportive d ialysis. To pred ict this patient's lab findings, one has to figure out what type of renal failure he had : 11 prerenal, intrarenal,p or ostrenaL ATN is an intrarenal process, and intrarenal damage is marked by a decline in g lomerular filtration rate. BUN and plasma Cr concentrations increase proportionally, so the ratio is approximately 10-15: 1. Because the kidney is damaged in ATN, it will not 12 reabsorb sodium and water to concentrate the urine. Therefore, the urine osmolarity is low/normal ( <500 mOsm/L}, the fraction of excreted 13 sodium is >2%, and the urine sodium is >20 m Eq/ L. Lastly, a decrease in renal blood flow is associated with a decrease in urine output. Myoglobinuria Acute tubular necrosis Renal function Acute kidney injury Urine Ischemia Osmotic concentration Blood urea nitrogen Kidney Acidosis Dialysis 14 Glomerulus Sodium Necrosis Autopsy Crush syndrome Renal blood flow Metabolism Glomerulus (kidney) 15 A is no t correct . JOfo c hose t his. 16 The urine osmolarity, urine sodium, fraction of excreted sodium, and BUN :Cr ratio point to a prerenal cause of g lomerular inj ury. 17 Osmotic concentration Sodium Urine Glomerulus Glomerulus (kidney) Acute kidney injury

18 C is no t correct . 180/o c hose t his. 19 These findings are consistent with postrenal azotemia. Postrenal azotemia is a resu lt of obstruction of urine flow from the kidneys to the bladder. For example, prostatic d isease and urethral stones can cause urinary obstruction, lead ing to acute renal failure. The BUN :Cr ratio is initially> 15, 20 although it can decrease as the problem persists, due to epithelial damage in the tubules. Because there is no intrinsic renal problem, the urine 21 osmolality is normal ( <500 mOsm/L}, along with the fraction of excreted sodium {>2%}, and the urine sodium {>20 m Eq/ L}. 22 Azotemia Acute kidney injury Urinary retention Urine Prostate Sodium Urinary bladder Kidney Epithelium

23 0 is no t correct . 80/o c hose t his. 24 These findings are consistent with postrenal azotemia, with one exception . With postrenal azotemia, the fraction of excreted sodium is >2%. Azotemia Sodium • 25 E is no t correct . 11 Ofo c hose t his . • 26 A BUN : Cr ratio above 20: 1 is ind icative of prerenal d isease. Prerenal d isease is due to conditions that are associated with decreased blood flow to • 27 the kidneys, includ ing hemorrhage, hypovolemic shock, and congestive heart failure. Renal biopsy will show normal parenchyma. The kidneys, ~ ~ 6 s 0 lock Suspend End Block Item: 24 of 27 ~ 1 • M k -<:J 1>- Jil ~· !:';-~ QIO: 1797 ..L a r Previous Next Labfli!llues Not es Calculat o r

A A 7 FA17 p 572.2 8 Acute tubular necrosis to- lost common cause of acute kidney injur) in hospitali zed patients. Spontaneously resokes in 9 many cases. Can be fatal, especially during initial oliguric phase. t FENa. 10 Key finding: granular (""muddy brown") casts 11 3 stages: I. Inciting e\·ent 12 2. yfaintenance phase-oliguric; la ~ t s 1-3 " eeks; risk of hyperkalemia, metabolic acidosis, 13 urem1a 14 3. Reco,·ery phase-polyuric; BUN" and serum creatinine fall; risk of hypokalemia 15 Can be caused by ischemic or nephrotoxic injury: 16 lschemic- 2° to l renal blood Aow (eg, h) potension, shock, sepsis, hemorrhage, H F'). Results in death of tubular cells that may slough into tubular lumen : (PCT and thick ascending limb 17 are highl) susceptible to injury). 18 • 1 ephrotoxic-2° to injury resulting from toxic substances (eg, aminoglycosides, radiocontrast 19 agents, lead, cisplatin, ethylene glycol), crush injury (myoglobinuria), hemoglobinuria. PCT is 20 particularly susceptible to injury. 21 22

23 FA17 p 571 .2 24 Acute kidney injury Acute kidney injury is defined as an abrupt decline in renal function as measured by t creatinine • 25 (acute renal failure) and t BUN or by oliguria/anuria. • 26 Prerenal azotemia Due to l RBF (eg, hypotension) - l CFR. a+/11 20 and BU1 retained by kidn ey in an attempt to • 27 consene \'Oiume - t BU '/creatinine ratio (BU l is reabsorbed, creatinine is not) and l FE"• . a s 8 Lock Suspend End Block Item: 25 of 27 ~ 1 • M k -<:J 1>- Jil ~· !:';-~ QIO: 1774 ..L a r Previous Next Labfli!llues Not es Calculat o r

A A 7 A 67-year-old man is diagnosed with acute lymphocytic leukemia and is administered induction chemotherapy. Three days after his course is 8 complete, he presents to the emergency department with sharp, sudden-onset, sporadic lower back pain. His physical examination is unremarkable, and x-ray of his abdomen shows no calcifications; however, further work-up with sonography reveals a stone in his right 9 ureter. 10

11 Which of the following is the most likely cause of this patient's kidney stone? 12 : 13 A. Colonization of his urinary tract with urease-positive bacteria 14 B. Cystinuria 15 C. Hyperparathyroidism 16 17 0. Increased breakdown of nucleic acid

18 E. Increased destruction of bone by osteoclasts 19 20 21 22 23 24 • 25 . 26 . 27 • a s 8 Lock Suspend End Block Item: 25 of 27 ~ 1 • M k -<:J 1>- Jil ~· !:';-~ QIO: 1774 ..L a r Prev ious Next Labfli!llues Not es Calculat or

A A 7 The correct a ns wer is D. 680/o chose this. 8 Uric acid is a metabolite of nucleic acid turnover, which is heightened in the setting of cell destruction. As a result, hyperuricemia, and subsequent uric acid stones, are seen in diseases with increased cell proliferation and turnover, such as leukemia and myeloproliferative 9 disorders. Induction chemotherapy can lead to massive cell necrosis and release of purines, resulting in an increase in uric acid levels and stone 10 formation. Uric acid stones are radiolucent, so they are not visible on x-ray. They can be detected through CT scan and ultrasound. The differences in imaging modality between radiopaque and radiolucent stones is shown in the image. 11 Hype u icemoa Uric acod Nucleic acid CT scan Chemotherapy Leukemia Myelop o ifeo •tive neoplasm Ultrasound Metabolite Purine Radiodensity Necrosos 12 Medical ultrasound X-ray Cell growth 13 14 15 ephrollthlasls 16

17 R~d lo luc«ot stones 18 ·-" / 1l'·"-'"'"' l ~ 19 Uric. acid calcium Struvite 20 t ~ / t 21 ViSible on: VIsible on: Visible on:

22 1. CT scan 1. x-ray 1. CT scan 2. U ftrasound 2. CT scan 2. Uttrasound 23 3. Ukrasound 24 " Cystine stones are considered radloopaque due to their suHur content, but are rarely visible on x-ray. 25 • 26 Image courtesy of Holly Villa magna

• 27 A is no t correct. 8 0/o chose this. a s 8 Lock Suspend End Block Item: 25 of 27 ~ . , . M k <:] t> al ~· ~ QIO: 1774 .l. ar Previous Next lab'lifllues Notes Calculator

7 A is not correct. 80fo c hose t his. Urinary tract infection with urease-positive microorganisms such as Proteus vulgaris and Staphylococcus saprophyticus can lead to large struvite 8 calculi (ammonium magnesiu m phosphate), which are rad iopaque. These stones are more common in women. 9 Urinary tract infection Proteus vulgaris Struvite Staphylococcus saprophyticus Urinary system Staphylococcus Radiodensity Proteus (bacterium) Magnesium 10 Infection Phosphate Microorganism

11 B is not correct. 100/o c hose t his. 12 Cystine stones are formed secondary to cystinuria, a rare autosomal recessive condition characterized by an inability of the proximal renal tubule to reabsorb d ibasic amino acids such as cystine. As a resu lt, there is an excessive amount of insoluble cysteine in the urine, and stones can 13 precipitate. Cysteine stones are faintly rad iopaque and generally not visible on abdominal x-ray, but they become rad iolucent with intravenous 14 contrast. The j ej unum is also unable to absorb cystine in this d isease, but because it is not an essential amino acid, this is not of clinical significance. 15 Cystinuria Essential amino acid Jejunum Amino acid Cysteine Proximal convoluted tubule Cystine Dominance (genetics) Autosomal recessive Renal tubule Urine 16 Autosome Radiodensity Intravenous therapy X-ray Recessive Nephron Kidney Precipitation (chemistry)

17 C is not correct. 60/o c hose t his. 18 Hyperparathyroid ism would cause hypercalcemia, which can lead to calcium -based kidney stones that are rad iopaque, not rad iolucent. This patient does not have symptoms of hypercalcemia, which include bone pain, abdominal d iscomfort, and depression . 19 Hyperparathyroidism Hypercalcaemia Kidney Kidney stone Radiodensity Bone pain Bone Abdominal pain 20 E is not correct. 80fo c hose t his. 21 Destructive bone d iseases cause hypercalcemia and lead to the production of rad iopaque (not rad iolucent) calcium -based stones. Calciu m-based 22 stones are the most common cause of kidney stones (80%-85% ), and the stones are most commonly made of calcium oxalate or calcium phosphate. Other risk factors are increased vitamin D levels and m ilk-alkali syndrome. Given this patient's history of acute lymphocytic leu kemia, 23 it is more likely that he is experiencing uric acid stones. 24 Milk-alkali syndrome Uric acid Calcium oxalate Acute lymphoblastic leukemia Hypercalcaemia Vitamin D Calcium phosphate leukemia Kidney stone Calcium 25 Oxalate Kidney Radiodensity Phosphate Vitamin Bone • 26 • 27 Bottom Line : 6 s 0 lock Suspend End Block Item: 25 of 27 ~ . , . M k <:] t> al ~· ~ QIO: 1774 .l. ar Previous Next lab'lifllues Notes Calculator

7 Infection Phosphate Microorganism

8 B is no t correct. 100/o c hose this. Cystine stones are formed secondary to cystinuria, a rare autosomal recessive condition characterized by an inability of the proximal renal tubule 9 to reabsorb d ibasic amino acids such as cystine. As a resu lt, there is an excessive amount of insoluble cysteine in the urine, and stones can 10 precipitate. Cysteine stones are faintly rad iopaque and generally not visible on abdominal x-ray, but they become rad iolucent with intravenous 11 contrast. The j ej unum is also unable to absorb cystine in this d isease, but because it is not an essential amino acid, this is not of clinical significance. 12 Cystinuria Essential amino acid Jejunum Amino acid Cysteine Proximal convoluted tubule Cystine Dominance (genetics) Autosomal recessive Renal tubule Urine 13 Autosome Radiodensity Intravenous therapy X-ray Recessive Nephron Kidney Precipitation (chemistry)

14 C is no t correct. 6 0/o c hose this. 15 Hyperparathyroid ism would cause hypercalcemia, which can lead to calcium-based kidney stones that are rad iopaque, not rad iolucent. This patient does not have symptoms of hypercalcemia, which include bone pain, abdominal d iscomfort, and depression . 16 Hyperparathyroidism Hypercalcaemia Kidney Kidney stone Radiodensity Bone pain Bone Abdominal pain 17 E is no t correct. S O/o c hose this. 18 Destructive bone d iseases cause hypercalcemia and lead to the production of rad iopaque (not rad iolucent) calcium-based stones. Calcium-based 19 stones are the most common cause of kidney stones {80%-85% ), and the stones are most commonly made of calcium oxalate or calcium phosphate. Other risk factors are increased vitamin D levels and milk-alkali syndrome. Given this patient's history of acute lymphocytic leu kemia, 20 it is more likely that he is experiencing uric acid stones. 21 Milk-alkali syndrome Uric acid Calcium oxalate Acute lymphoblastic leukemia Hypercalcaemia Vitamin D Calcium phosphate leukemia Kidney stone Calcium Oxalate Kidney Radiodensity Phosphate Vitamin Bone 22 23 24 Bo tto m Line : 25 Increased cell proliferation and turnover can resu lt in rad iolucent kidney stones secondary to increased free uric acid released during cell breakdown . • 26 Uric acid Radiodensity Kidney Kidney stone Cell growth • 27 6 s 0 lock Suspend End Block Item: 25 of 27 ~ . , . M k <:] t> al ~· ~ QIO: 1774 .l. ar Previous Next Lab'lifllues Notes Calculator • • 7 8 FA17 p 567.1 9 Kidney Can lead to se,·ere compl ications such as hyd ronephrosis, pyelonephritis. Presents with unilateral Aank stones 10 tenderness, colicky pain radiating to groin, hematuria. Treat and prevent by encouraging Auid intake. Most common kidney stone presentation: calcium oxa late stone in patient with hypercalciuria and 11 normoca lcem ia. 12 CONTENT PRECIPITATES WITH X-RAY FINDINGS CT FINDINGS URINEC RYSTAL NOTES 13 Calcium Calcium Radiopaque Radiopaque Shaped like Calcium stones most common (80%); calcium 14 oxalate: envelope rJ oxalate more common than calcium 15 hypocitraturia or dumbbell phosphate stones. Hypocitraturia often associated with ! urine pH. 16 Can result from ethylene glycol (antifreeze) 17 ingestion, vitamin C abuse, hypocitraturia, 18 malabsorption (eg, Crohn disease). 19 Treatment: th iazides, citrate, low-sodium diet. 20 Calcium Radiopaque Radiopaque Wedge- Treatment: low-sodium diet, thiazides. 21 phosphate: shaped t pH pnsm 22 Ammonium t pH Radiopaque Radiopaque Coffin lid I] Also known as struvite; account for 15% of 23 magnesium stones. Caused by infection with urease® 24 phosphate bugs (eg, Proteus mirabilis, Staphylococcus 25 saprophyticus, Klebsiella) that hydrolyze • 26 urea to ammonia ... urine alkalinization . Commonly form staghorn ca l c uli ~ . • 27 • ,.....,_ ___ .._ ____ .._ ____ ,: __ , : __ _ c ___ .J _ _ J•• : __ :_ c __ .. : __ • 6 s 0 lock Suspend End Block Item: 25 of 27 ~ . , . M k <:] t> al ~· ~ QIO: 1774 .l. ar Previous Next Lab'lifllues Notes Calculator • • 7 8 FA17 p 567.1 9 Kidney Can lead to se,·ere compl ications such as hyd ronephrosis, pyelonephritis. Presents with unilateral Aank stones 10 tenderness, colicky pain radiating to groin, hematuria. Treat and prevent by encouraging Auid intake. Most common kidney stone presentation: calcium oxa late stone in patient with hypercalciuria and 11 normoca lcem ia. 12 CONTENT PRECIPITATES WITH X-RAY FINDINGS CT FINDINGS URINEC RYSTAL NOTES 13 Calcium Calcium Radiopaque Radiopaque Shaped like Calcium stones most common (80%); calcium 14 oxalate: envelope rJ oxalate more common than calcium 15 hypocitraturia or dumbbell phosphate stones. Hypocitraturia often associated with ! urine pH. 16 Can result from ethylene glycol (antifreeze) 17 ingestion, vitamin C abuse, hypocitraturia, 18 malabsorption (eg, Crohn disease). 19 Treatment: th iazides, citrate, low-sodium diet. 20 Calcium Radiopaque Radiopaque Wedge- Treatment: low-sodium diet, thiazides. 21 phosphate: shaped t pH pnsm 22 Ammonium t pH Radiopaque Radiopaque Coffin lid I] Also known as struvite; account for 15% of 23 magnesium stones. Caused by infection with urease® 24 phosphate bugs (eg, Proteus mirabilis, Staphylococcus 25 saprophyticus, Klebsiella) that hydrolyze • 26 urea to ammonia ... urine alkalinization . Commonly form staghorn ca l c uli ~ . • 27 • ,.....,_ ___ .._ ____ .._ ____ ,: __ , : __ _ c ___ .J _ _ J•• : __ :_ c __ .. : __ • 6 s 0 lock Suspend End Block Item: 25 of 27 ~ 1 • M k -<:J 1>- Jil ~· !:';-~ QIO: 1774 ..L a r Previous Next Labfli!llues Not es Calculat o r

- ~ ,.. _ _ - - - - A A 7 in urine. Usually begins in childhood. Can 8 form staghorn calculi. Sodium cyanide 9 nitroprusside test ®. "SLXtine" stones have SIX sides. 10 Treatment: lo"· sodium diet, al kalinization of 11 urine, chelating agents if refractory. 12 13 14 15 16 17 18 19 FA1 7 p 177.1 20 Urinary tract Cystitis presents with dysuria, frequency, urgency, suprapubic pain, and WBCs (but not WBC 21 infections casts) in urine. Primarily caused by ascension of microbes from urethra to bladder. !ales­ 22 infants with congenital defects, vesicourel'eral reA ux. Elderly-enlarged prostate. Ascension to 23 kidney results in pyelonephritis, which presents with fever, ch ills, Aank pain, costovertebral angle 24 tenderness, hematuria, and WBC casts. 25 Ten times more common in \\Omen (shorter urethras colonized by fecal Aora). Other predisposing factors: obstruction, kidn ey surgery, catheterization, GU malformation, diabetes, pregnancy. • 26 • 27 a s 8 Lock Suspend End Block Item: 26 of 27 ~ 1 • M k -<:J 1>- Jil ~· !:';-~ QIO: 1773 ..L a r Previous Next Labfli!llues Notes Calculat o r

A A 7 A 40-year-old man presents to the emergency department with sharp, intermittent abdominal pain radiating into his right scrotum. He states 8 he has experienced similar symptoms in the past, but they resolved on their own. On review of systems, the patient states he's had trouble remembering things for the past 2- 3 months. He also notes constipation, occasional burning pain in his stomach, muscle weakness, and that 9 he has been urinating abnormally large volumes of urine during this time period. On physical exam there is right-sided costovertebral angle 10 tenderness. Genitourinary and rectal exam are normal. Urinalysis reveals numerous red blood cells. A plain film of the abdomen and pelvis reveals several opaque densities in the right midabdomen. A follow-up noncontrast CT scan demonstrates several large stones in the right ureter and 11 bilateral renal calyces. 12

13 Which of the following is the most likely cause of this man's symptoms? 14 : 15 A. Constipation 16 B. Crohn disease 17 C. Hyperparathyroidism 18 19 D. Hyperuricemia

20 E. Staphylococcal infection 21 22 23 24 25 . 26 . 27 • a s 8 Lock Suspend End Block Item: 26 of 27 ~ . I • M k <:] t> al ~· ~ QIO: 1773 .l. ar Previous Next lab 'lifllues Notes Calculator • 7 The correct answer is C. 560/o chose this. 8 Calcium stones are the most common type of kidney stones (80%- 85% ). Although most patients that develop calcium stones have normal levels 9 of calcium in their blood, conditions that resu lt in hypercalcemia, such as hyperparathyroid ism, cancers such as squamous cell carcinoma of the lung that produce PTH-related peptide as a paraneoplastic syndrome, or multiple myeloma, can also induce calcium stone formation. The stones 10 are made of calcium oxalate or calcium phosphate and are visible on both plain film and noncontrast CT scan . This d iffers from uric acid stones, 11 which are rad iolucent on plain film and opaque on CT scan . Other risk factors for kidney stone formation are excessive intake of vitamin D and milk-alkali syndrome. Most commonly, hyperparathyrodism is asymptomatic, as it is typically d iscovered incidentally on routine blood screening 12 and treated before symptoms develop. This patient is exhibiting signs and symptoms of hypercalcemia ("Stones, bones, groans, and 13 psychiatric overtones."). The most common symptoms include anorexia, epigastric burning pain, constipation, anxiety, depression, muscle weakness, and excessive urination due to calcium 's inhibition of antidiuretic hormone at the kidney. 14 Milk-alkali syndrome Kidney stone Paraneoplastic syndrome Multiple myeloma Uric acid Calcium oxalate Hypercalcaemia Hyperparathyroidism Vitamin D CT scan

15 Squamous-cell carcinoma Constipation Calcium phosphate Calcium Kidney Polyuria Vasopressin Anorexia (symptom) Radiodensity Phosphate Asymptomatic 16 Oxalate Hormone Peptide Muscle weakness Anxiety Vitamin lung Anorexia nervosa Radiography Carcinoma Muscle 17 A is not correct. 2 0/o chose this. 18 A large stool is unlikely to cause flank pain, and pain would not come in the waves of pain this patient is experiencing . Abdominal pain Feces Human feces 19 B is not correct. 150/o chose this. 20 Crohn d isease can resu lt in oxalate stone formation when there is ileal involvement secondary to fat malabsorption. Fat binds to calcium (rather 21 than oxalate), leaving the oxalate free to be absorbed and deposited in the kidney, where it can form into stones. This patient has some 22 epigastric burning; however, this is due to hypercalcemia. He has no other physical exam findings to suggest Crohn d isease. Hypercalcaemia Crohn's disease Oxalate Malabsorption Ileum Kidney Calcium Epigastrium Fat Physical examination 23 0 is not correct. 21 Ofo chose this. 24 Normally hyperuricemia leads to kidney stones that are rad iolucent and therefore not seen on X-ray. They would, however, be seen on 25 noncontrast CT. These stones are often seen in the setting of d iseases with increased cell proliferation and turnover, such as leu kemia and 26 myeloproliferative d isorders, and as a resu lt of tumor lysis syndrome in the treatment of these d iseases. Prevention of tumor lysis syndrome involves IV fluid hydration and allopurinol or rasburicase . • 27 ~ Rasburicase Tumor lysis syndrome Alloourinol Hyperuricemia leukemia Myeloproliferative neoolasm Kidney stone Kidney Intravenous theraoy Neoolasm X-ray 6 s 0 lock Suspend End Block Item: 26 of 27 ~ . I • M k <:] t> al ~· ~ QIO: 1773 .l. ar Previous Next lab 'lifllues Notes Calculator • . . . . . 7 Crohn d isease can resu lt in oxalate stone formation when there is ileal involvement secondary to fat malabsorption . Fat binds to calcium (rather 8 than oxalate), leaving the oxalate free to be absorbed and deposited in the kidney, where it can form into stones. This patient has some epigastric burning; however, this is due to hypercalcemia. He has no other physical exam findings to suggest Crohn d isease. 9 Hypercalcaemia Crohn's disease Oxalate Malabsorption Ileum Kidney Calcium Epigastrium Fat Physical examination 10 0 is not correct. 2 1 Ofo c hose t his. 11 Normally hyperuricemia leads to kidney stones that are rad iolucent and therefore not seen on X-ray. They would, however, be seen on 12 noncontrast CT. These stones are often seen in the setting of d iseases with increased cell proliferation and turnover, such as leu kemia and myeloproliferative d isorders, and as a resu lt of tumor lysis syndrome in the treatment of these d iseases. Prevention of tumor lysis syndrome 13 involves IV fluid hydration and allopurinol or rasburicase. 14 Rasburicase Tumor lysis syndrome Allopurinol Hyperuricemia leukemia Myeloproliferative neoplasm Kidney stone Kidney Intravenous therapy Neoplasm X-ray

15 lysis

16 E is not correct. 60/o c hose t his. Urinary tract infection with urease-positive microorganisms such as Staphylococcus saprophyticus can form large struvite calculi that are 17 rad iopaque. Urease-positive organims, includ ing Proteus mirabilis, Klebsiella, and Staphyloccocus, form ammonium magnesium phosphate 18 "struvite" stones, resu lting in staghorn calculi. These are more commonly found in women with recu rrent urinary tract infections and those with renal tract anomalies, reflux, or neurogenic bladder. This patient is exhibiting signs and symptoms of hypercalcemia, which makes this answer 19 choice less likely. 20 Urinary tract infection Hypercalcaemia Proteus mirabilis Struvite Staphylococcus Staphylococcus saprophyticus Neurogenic bladder dysfunction Klebsiella 21 Radiodensity Urinary system Urinary bladder Magnesium Microorganism Phosphate Ammonium Proteus (bacterium) Infection Kidney

22 23 Bottom Line : 24 There are four types of renal stones: calcium, struvite, uric acid, and cystine. Approximately 80% of all renal stones contain calcium. Most renal 25 stones, includ ing calcium, struvite, and cystine stones, are rad iopaque and visible on a plain X-ray film . Uric acid stones are rad iolucent, so CT must be used to visualize them. 26 Uric acid Struvite Radiodensity Kidney stone Cystine Calcium X-ray Projectional radiography Radiography Kidney • 27 ~ 6 s 0 lock Suspend End Block Item: 26 of 27 ~ . I • M k <:] t> al ~· ~ QIO: 1773 .l. ar Previous Next lab 'lifllues Notes Calculator • • 7 FA17 p 567.1 8 Kidney Can lead to se,·ere compl ications such as hyd ronephrosis, pyelonephritis. Presents with un ilateral Aank 9 stones tenderness, colicky pain radiating to groin, hematuria. Treat and prevent by encouraging Auid intake. 10 Most common kidney stone presentation: calcium oxa late stone in patient with hypercalciuria and 11 normoca lcem ia. 12 CONTENT PRECIPITATES WITH X-RAY FINDINGS CT FINDINGS URINE CRYSTAL NOTES 13 Calcium Calcium Radiopaque Radiopaque Shaped like Calcium stones most common (80%); calcium 14 oxalate: envelope rJ oxalate more common than calcium hypocitraturia or dumbbell phosphate stones. 15 Hypocitraturia often associated with l urine pH. 16 Can result from ethylene glycol (an ti freeze) 17 ingestion, vitamin C abuse, hypocitraturia, 18 malabsorption (eg, Crohn disease). Treatment: thiazides, ci trate, low-sodium diet. 19 Calcium Radiopaque Radiopaque Wedge- Treatment: low-sodium diet, thiazides. 20 phosphate: shaped 21 t pH pnsm 22 Ammonium t pH Radiopaque Radiopaque Coffin lid rn Also known as struvite; account for 15% of 23 magnesium stones. Caused by infection with urease® 24 phosphate bugs (eg, Proteus mirabilis, Staphylococcus 25 saprophyticus, Klebsiella) that hydrolyze urea to ammonia ... urine alkalinization. 26 Commonly form staghorn calculi [i. • 27 • Treatment: eradication of undcrlvine infection . • 6 s 0 lock Suspend End Block Item: 26 of 27 ~ . I • M k <:] t> al ~· ~ QIO: 1773 .l. ar Previous Next lab 'lifllues Notes Calculator • • 7 FA17 p 567.1 8 Kidney Can lead to se,·ere compl ications such as hyd ronephrosis, pyelonephritis. Presents with un ilateral Aank 9 stones tenderness, colicky pain radiating to groin, hematuria. Treat and prevent by encouraging Auid intake. 10 Most common kidney stone presentation: calcium oxa late stone in patient with hypercalciuria and 11 normoca lcem ia. 12 CONTENT PRECIPITATES WITH X-RAY FINDINGS CT FINDINGS URINE CRYSTAL NOTES 13 Calcium Calcium Radiopaque Radiopaque Shaped like Calcium stones most common (80%); calcium 14 oxalate: envelope rJ oxalate more common than calcium hypocitraturia or dumbbell phosphate stones. 15 Hypocitraturia often associated with l urine pH. 16 Can result from ethylene glycol (an ti freeze) 17 ingestion, vitamin C abuse, hypocitraturia, 18 malabsorption (eg, Crohn disease). Treatment: thiazides, ci trate, low-sodium diet. 19 Calcium Radiopaque Radiopaque Wedge- Treatment: low-sodium diet, thiazides. 20 phosphate: shaped 21 t pH pnsm 22 Ammonium t pH Radiopaque Radiopaque Coffin lid rn Also known as struvite; account for 15% of 23 magnesium stones. Caused by infection with urease® 24 phosphate bugs (eg, Proteus mirabilis, Staphylococcus 25 saprophyticus, Klebsiella) that hydrolyze urea to ammonia ... urine alkalinization. 26 Commonly form staghorn calculi [i. • 27 • Treatment: eradication of undcrlvine infection . • 6 s 0 lock Suspend End Block Item: 27 of 27 ~ 1 • M k -<:J 1>- Jil ~· !:';-~ QIO: 4997 ..L a r Previous Next Labfli!l tues Not es Calculat o r

A A 7 A 45-year-old woman with a recent Aare of Crohn disease presents with excruciating pain in her left side that radiates down to her groin. She 8 is unable to stay still for a complete physical examination, but it is noted that her left Aank is tender. A non contrast helical CT scan confirms the presence of a kidney stone. 9 10 Which of the following is the most likely make-up of this patient's stone? 11

12 A. Calcium oxalate 13 B. Calcium phosphate 14 15 C. Cystine

16 D. Magnesium ammonium phosphate 17 E. Uric acid 18 19 20 21 22 23 24 25 26 . 27 • a s 8 Lock Suspend End Block Item: 27 of 27 ~ . I • M k <:] t> al ~· ~ QIO: 4997 .l. ar Previous Next lab 'lifllues Notes Calculator • 7 The correct answer is A. 71 Ofo chose this. 8 Calcium stones are the most common type of kidney stone, accounting for 75% of cases. Typically they are caused by higher levels of calcium, due to hyperparathyroidism or increased absorption of calcium in the gut. Calcium-oxalate stones are caused by increased levels of oxalate and 9 are commonly found in patients with inflammatory bowel d isease (IBD) or chronic d iarrhea lead ing to severe dehydration . Malabsorption, as seen 10 in IBD, causes increased intraluminal fat, which will read ily bind calcium. This causes a decrease in the calcium that would normally bind oxalate, so more oxalate is absorbed in the gut. The increased level of oxalate combined with dehydration can precipitate the formation of calcium -oxalate 11 stones. 12 Kidney stone Inflammatory bowel disease Hyperparathyroidism Oxalate Diarrhea Kidney Calcium Dehydration Malabsorption Precipitation (chemistry) 13 Gastrointestinal tract lumen (anatomy) 14 B is not correct. 120/o chose this. 15 Crohn d isease does not cause calcium -phosphate stones, which are in general less common than calcium -oxalate stones overalL Calcium­ phosphate stones are seen more often in settings of hypercalcemia, which can be a resu lt of hyperparathyroidism, an increased absorption of 16 calcium in the gut, or an abnormally alkaline urine pH. Pregnancy and use of carbonic-anhydrase inhibitor are examples of patient types who may 17 have an increased urinary pH. A calcium -phosphate crystal, often described as a "wedge-shaped prism," will be rad iopaque on both x-ray and CT. 18 Hypercalcaemia Hyperparathyroidism Crohn's disease Radiodensity Alkalinity Urine Calcium Crystal X-ray Pregnancy 19 C is not correct. 50/o chose this. 20 Cystine stones are the least common, accounting for on ly 2% of kidney stones. These are caused by an intrinsic metabolic dysfunction, 21 interfering with resorption of Cystine along with O rnithine, Lysine, and A rg inine (commonly known as COLA). Cystine lysine Kidney Kidney stone Metabolism 22 0 is not correct. 50/o chose this. 23 Magnesium ammonium phosphate stones, or struvite stones, account for 15% of kidney stones. They are due to infection of the urinary tract by 24 g ram -negative rods that split urea into ammonia. The most common organisms are Proteus, Klebsiella, Providencia, Staphylococcus, and 25 Pseudomonas. Although these stones are most commonly found in women, g iven the patient's history of a recent flare of inflammatory bowel d isease (IBD), the most likely answer is calcium oxalate. 26 Ammonium phosphate Calcium oxalate Inflammatory bowel disease Struvite Urea Ammonia Gram-negative bacteria Klebsiella Magnesium Oxalate Phosphate 27 ~ Kidney stone Ammonium Kidney Urinary system Pseudomonas Staphylococcus Calcium Proteus (bacterium) Infection Gastrointestinal tract 6 s 0 lock Suspend End Block Item: 27 of 27 ~ . I • M k <:] t> al ~· ~ QIO: 4997 .l. ar Previous Next lab 'lifllues Notes Calculator • phosphate stones are seen more often in settings of hypercalcemia, which can be a resu lt of hyperparathyroidism, an increased absorption of • 7 calcium in the gut, or an abnormally alkaline urine pH. Pregnancy and use of carbonic-anhydrase inhibitor are examples of patient types who may 8 have an increased urinary pH. A calcium-phosphate crystal, often described as a "wedge-shaped prism," will be rad iopaque on both x-ray and CT. 9 Hypercalcaemia Hyperparathyroidism Crohn's disease Radiodensity Alkalinity Urine Calcium Crystal X-ray Pregnancy 10 C is no t correct. 50/o c hose this. 11 Cystine stones are the least common, accounting for on ly 2% of kidney stones. These are caused by an intrinsic metabolic dysfunction, 12 interfering with resorption of Cystine along with O rnithine, Lysine, and A rg inine (commonly known as COLA). Cystine lysine Kidney Kidney stone Metabolism 13 14 0 is no t correct. 50/o c hose this. Magnesium ammonium phosphate stones, or struvite stones, account for 15% of kidney stones. They are due to infection of the urinary tract by 15 g ram -negative rods that split urea into ammonia. The most common organisms are Proteus, Klebsiella, Providencia, Staphylococcus, and 16 Pseudomonas. Although these stones are most commonly found in women, g iven the patient's history of a recent flare of inflammatory bowel d isease (IBD), the most likely answer is calcium oxalate. 17 Ammonium phosphate Calcium oxalate Inflammatory bowel disease Struvite Urea Ammonia Gram-negative bacteria Klebsiella Magnesium Oxalate Phosphate

18 Kidney stone Ammonium Kidney Urinary system Pseudomonas Staphylococcus Calcium Proteus (bacterium) Infection Gastrointestinal tract 19 E is no t correct. 70/o c hose this. 20 Uric-acid stones account for 6% of kidney stones. These are caused by high levels of uric acid, associated with malignancy or high d ietary intake. 21 This type of stone is commonly found in patients with gout. Uric acid Gout Kidney Kidney stone Cancer Malignancy 22

23 Bo tto m Line : 24 Kidney stones are characterized by intense flank pain and, occasionally, vomiting. The most common type of stone is made of calcium and tends 25 to recu r. Any condition lead ing to hypercalcemia will pred ispose ind ividuals to stone formation. 26 Hypercalcaemia Kidney Abdominal pain Kidney stone Vomiting Calcium 27 ~ 6 s 0 lock Suspend End Block Item: 27 of 27 ~ . I • M k <:] t> al ~· ~ QIO: 4997 .l. ar Previous Next lab 'lifllues Notes Calculator • phosphate stones are seen more often in settings of hypercalcemia, which can be a resu lt of hyperparathyroidism, an increased absorption of • 7 calcium in the gut, or an abnormally alkaline urine pH. Pregnancy and use of carbonic-anhydrase inhibitor are examples of patient types who may 8 have an increased urinary pH. A calcium-phosphate crystal, often described as a "wedge-shaped prism," will be rad iopaque on both x-ray and CT. 9 Hypercalcaemia Hyperparathyroidism Crohn's disease Radiodensity Alkalinity Urine Calcium Crystal X-ray Pregnancy 10 C is no t correct. 50/o c hose this. 11 Cystine stones are the least common, accounting for on ly 2% of kidney stones. These are caused by an intrinsic metabolic dysfunction, t 12 interfering with resorption of Cystine along with O rnithine, Lysine, and A rg inine (commonly known as COLA). Cystine lysine Kidney Kidney stone Metabolism 13 14 0 is no t correct. 50/o c hose this. Magnesium ammonium phosphate stones, or struvite stones, account for 15% of kidney stones. They are due to infection of the urinary tract by 15 g ram -negative rods that split urea into ammonia. The most common organisms are Proteus, Klebsiella, Providencia, Staphylococcus, and 16 Pseudomonas. Although these stones are most commonly found in women, g iven the patient's history of a recent flare of inflammatory bowel d isease (IBD), the most likely answer is calcium oxalate. 17 Ammonium phosphate Calcium oxalate Inflammatory bowel disease Struvite Urea Ammonia Gram-negative bacteria Klebsiella Magnesium Oxalate Phosphate

18 Kidney stone Ammonium Kidney Urinary system Pseudomonas Staphylococcus Calcium Proteus (bacterium) Infection Gastrointestinal tract 19 E is no t correct. 70/o c hose this. 20 Uric-acid stones account for 6% of kidney stones. These are caused by high levels of uric acid, associated with malignancy or high d ietary intake. 21 This type of stone is commonly found in patients with gout. Uric acid Gout Kidney Kidney stone Cancer Malignancy 22

23 Bo tto m Line : 24 Kidney stones are characterized by intense flank pain and, occasionally, vomiting. The most common type of stone is made of calcium and tends 25 to recu r. Any condition lead ing to hypercalcemia will pred ispose ind ividuals to stone formation. 26 Hypercalcaemia Kidney Abdominal pain Kidney stone Vomiting Calcium 27 ~ 6 s 0 lock Suspend End Block Item: 27 of 27 ~ . I • M k <:] t> al ~· ~ QIO: 4997 .l. ar Previous Next lab 'lifllues Notes Calculator • • 7 FA17 p 567.1 8 Kidney Can lead to se,·ere compl ications such as hyd roneph rosis, pyelonephritis. Presents with unilateral Aank 9 stones tenderness, colicky pain radiating to groin, hematuria. Treat and prevent by encouraging Auid intake. 10 Most common kidney stone presentation: calcium oxa late stone in patient with hypercalciuria and normoca lcem ia. 11 CONTENT PRECIPITATES WITH X-RAY FINDINGS CT FINDINGS URINE CRYSTAL NOTES 12 Calcium Calcium Radiopaque Radiopaque Shaped like Calcium stones most common (80%); calcium 13 oxalate: envelope rJ oxalate more common than calcium 14 hypocitraturia or dumbbell phosphate stones. 15 Hypocitraturia often associated with l urine pH. Can result from ethylene glycol (an ti freeze) 16 ingestion, vitamin C abuse, hypocitraturia, 17 malabsorption (eg, Crohn disease). 18 Treatment: thiazides, ci trate, low-sodium diet. 19 Calcium Radiopaque Radiopaque Wedge- Treatment: low-sodium diet, thiazides. 20 phosphate: shaped 21 t pH pnsm 22 Ammonium t pH Radiopaque Radiopaque Coffin lid rn Also known as struvite; account for 15% of magnesium stones. Caused by infection with urease ® 23 phosphate bugs (eg, Proteus mirabilis, Staphylococcus 24 saprophyticus, Klebsiella) that hydrolyze 25 urea to ammonia ... urine alkalinization. 26 Commonly form staghorn calculi [i. Treatment: eradication of underlying infection, 27 • __ . __; __ I - ---··-' _c _.._ ___ • 6 s 0 lock Suspend End Block Item: 27 of 27 ~ 1 • M k -<:J 1>- Jil ~· !:';-~ QIO: 4997 ..L a r Previous Next Labfli!l tues Not es Calculat o r

A A 7 FA17 p365.1 8 Inflammatory bowel disease 9 Crohn disease Ulcerative colitis 10 lOCATION Any portion of the Cl tract, usually the terminal Colitis= colon inAammation. Continuous 11 ileum and colon. Slip lesions, rectal sparing. colonic lesions, always" ith rectal imolvement. 12 GROSS MORPHOLOGY Transmural inAammation -+ fi stulas. \ lucosal and submucosal inAammation onlr. 13 C obblestone mucosa, creeping fat , bowel wall Friable mucosa with superficial and/or 14 thickening (-string sign" on barium S\\a llo'' deep ulcerations (compare normal rn with 15 x-rar fJ). linear ulcers, fissures. diseased ~). Loss of haustra -+ "lead pipe" . . appearance on 1magmg. 16 MICROSCOPIC MORPHOLOGY 17 'oncaseating granulomas and 1) mphoid Crypt abscesses and ulcers, bleeding, no aggregates. Th I mediated. granulomas. Th2 mediated. 18 COMPLICATIONS 1\lalabsorption/malnutri tion, colorectal cancer (f risk with pancolitis). 19 which can ation. 20 Fistulas (eg, enterovesical fistulae, Fulminant colitis, toxic megacolon, perfor cause recurrent UTI and pneumaturia), 21 phlegmon/abscess, strictures (causi ng 22 obstruction), perianal disease. 23 INTESTINAl MANIFESTATION Diarrhea that may or may no t be bloody. Bloody diarrhea. 24 EXTRAINTESTINAlMANIFESTATIONS Rash (pyoderma gangrenosum, erythema nodosum), eye inAammation (episcleritis, u,·eitis), oral 25 ulcerations (aphthous st01mtitis), arthritis (peripheral, spondylitis). 26 Kidn ey stones (usually calcium oxalate), 1° sclerosing cholangitis. ssociatcd with 27 gallstones. ~ la\' be ~ for anti-Saccharomrces p-AN CA. a s 8 Lock Suspend End Block